Download as pdf
Download as pdf
You are on page 1of 592
2/24/2014 6:19:44 AM ‘Mark this question => Question Id : 24658 Question 1 of 30 A.60-year-old male has three day old Rkabdomyalysis. Being a patient of IHD, he takes a mmber of drugs inching Simvastatin for years. Which of the following health supplement maybe the culprit? a) Cranberry juice ) Grapefhuit juice ©) Cod liver oil capsules 4) Ginseng €) Vitamin C Anower (UEQBRERER) othe: Uses Explanation Report An Err Question Explanatio Grapetiut juice is well recognized to inhibit statin metab lism thereby potentiating the myotoxiciy. In particular, grapeiuit juice should be avoided with simvastatin and lovastatin and it also inhibits metabolism of calsium channel blockers and theophylline 2/24/2014 6:19:44 AM ‘Mark this question => Question Id : 24658 Question 1 of 30 A.60-year-old male has three day old Rkabdomyalysis. Being a patient of IHD, he takes a mmber of drugs inching Simvastatin for years. Which of the following health supplement maybe the culprit? a) Cranberry juice Y © b) Grapefiuit juice ©) Cod liver oil capsules 4) Ginseng €) Vitamin C Anower (UEQBRERER) othe: Uses Explanation Report An Err Question Explanatio Grapetiut juice is well recognized to inhibit statin metab lism thereby potentiating the myotoxiciy. In particular, grapeiuit juice should be avoided with simvastatin and lovastatin and it also inhibits metabolism of calsium channel blockers and theophylline 2/24/2014 6:20:06 AM “Maule this question & => Question 2 of 30 AAA year old male comes to the emergency department because of the sudden onsct of shortness of breath, light-headedaess, daphoresis and weakness, He is afebrile, Bibasilar rales are heard on auscultation of lungs. lectrocardiogram is shown, “Which of the following is the most likely diagnosis? a) Acute pericerdiis b) Hyperventilation syndrome ©) Pulmonary embolism 4) Spontaneous pneumothorax ©) Myocardal infarction Anewor EUNEERY) othe: user's Explanation Repost An Ero Question Explanation: ‘The electrocardiogram shows ST segment elevation in leads ¥1-V4. Furthermore, there are Q-waves forming in these leads. Although he is younger than most patients with a myocardial infarction, the symptoms and EKG are classic. As expected with an infarction of this size, he is beginring to develop pulmonary congestion as demonstrated by the bibasilar rales. 2/24/2014 6:20:06 AM “Maule this question & => Question 2 of 30 AAA year old male comes to the emergency department because of the sudden onsct of shortness of breath, light-headedaess, daphoresis and weakness, He is afebrile, Bibasilar rales are heard on auscultation of lungs. lectrocardiogram is shown, ‘Which of the following is the most likely diagnosis? a) Acute pericerdiis 'b) Hyperventiation syndrome c) Pulmonary embolism d) Spontancous pneumothorax Y © e)Myocardal infarction Anewor EQUNEERY) othe: user's Explanation Repost An Ero Question Explanation: ‘The electrocardiogram shows ST segment elevation in leads ¥1-V4. Furthermore, there are Q-waves forming in these leads. Although he is younger than most patients with a myocardial infarction, the symptoms and EKG are classic. As expected with an infarction of this size, he is beginring to develop pulmonary congestion as demonstrated by the bibasilar rales. 2/24/2014 6:20:20 AN ‘Mark this question & => Question Ta : 48691 Question 3 of 30 A.48 year old white male is under treetment for diabetes mellitus and hyperlipidemia He intially presented 1 year ago and his hemoglobin A1C was 8.0% (N3.8-64), LDL 13) mg/dL, HDL 28 mafaL, and triglycerides 450 mg/dL. After treatment with metformin (Glucophage) and hich dose Simvastatin (Zocor). his most recent laboratory evaluation revealed a hemoglobin AIC of 6.2%, LDL 95 mg/dL, HDL 32 mg/dL, and triglycerides 300 mg/4L. The patient har not had any documented corcnary or peripheral vascular disease. His family hictory ic positive for a myocardliel infarction in his father at age 55. He is a nonsmolcer. He has a body mass index (BMI) of 28 and has been mable to lose weight. His blood pressure is well controlled on Enalapril (Vasotec). The most appropriate management of his elevated triglycerides is a) No specific treatment +b) Switch from metformin to 2 sulfonylurea such as Glyburide (Micronase, DiaBeta) or Gtipiride (Giucotrol) ©) Addition ofa Bbrate such as gemibrozil (Lopid) or fenofibrate (Tricor) 4) Addition of cholestyramine (Questran) Question Explanation “Although the significance of elevated triglycerides and a low HDL in low-risk patient is somewhat uncertain, in a high risk patient such as a diabetic, improvement in these results will lower the risk of subsequent cardiac events. Tn diahetics, metformin and thiazolidinediones (e.g, rosiglitazone) are more licely to improve lipid levels than are culfonylureas. Nicotinic acid is problematic in diabetics, as it tends to cause deterioration in glucose costrol, Fibrates are good choices for this patient because they will lower the triglyceride level and raise the HDL level. Exercise and weight loss are licely to be helpfil as well. Cholestyramnine will raise triglyceride levels 2/24/2014 6:20:20 AN ‘Mark this question & => Question Ta : 48691 Question 3 of 30 A.48 year old white male is under treetment for diabetes mellitus and hyperlipidemia He intially presented 1 year ago and his hemoglobin A1C was 8.0% (N3.8-64), LDL 13) mg/dL, HDL 28 mafaL, and triglycerides 450 mg/dL. After treatment with metformin (Glucophage) and hich dose Simvastatin (Zocor). his most recent laboratory evaluation revealed a hemoglobin AIC of 6.2%, LDL 95 mg/dL, HDL 32 mg/dL, and triglycerides 300 mg/4L. The patient har not had any documented corcnary or peripheral vascular disease. His family hictory ic positive for a myocardliel infarction in his father at age 55. He is a nonsmolcer. He has a body mass index (BMI) of 28 and has been mable to lose weight. His blood pressure is well controlled on Enalapril (Vasotec). The most appropriate management of his elevated triglycerides is a) No specific treatment +b) Switch from metformin to 2 sulfonylurea such as Glyburide (Micronase, DiaBeta) or Gtipiride (Giucotrol) Y © ©) Addition ofa Bbrate such as gemibrozil (Lopid) or fenofibrate (Tricor) 4) Addition of cholestyramine (Questran) Question Explanation “Although the significance of elevated triglycerides and a low HDL in low-risk patient is somewhat uncertain, in a high risk patient such as a diabetic, improvement in these results will lower the risk of subsequent cardiac events. Tn diahetics, metformin and thiazolidinediones (e.g, rosiglitazone) are more licely to improve lipid levels than are culfonylureas. Nicotinic acid is problematic in diabetics, as it tends to cause deterioration in glucose costrol, Fibrates are good choices for this patient because they will lower the triglyceride level and raise the HDL level. Exercise and weight loss are licely to be helpfil as well. Cholestyramnine will raise triglyceride levels 2/24/2014 6:20:37 AM ‘Mat this question & => Question Td : 48903 Question 4 of 30 A 66 year old male present with cardiac arrest due to ventricular rupture. The myocardial change that is the most frequent cause of this is a) Abscess formation and tissue destruction due to nnfective endocarditis +b) Fatty change due to interaction of diphtheria exotoxin and camitine ©) Aschoff bodies associated with henmatic fever 4) Inflammation due to coxsackie B infection €) Necrosis due to coronary artery obstruction Question Explanation: ‘The most common cause of myocardial infarction is narrowing of the coronary blood vessels due to athercmatovs plaques. Plaque riphare with subsequent exposure of the basement membrane results in platzle! aggregation, thrombus formation, fibrin accumulatioa, hhemorrhage into the plague, and varying degrees of vasospasm. This can resut in partial or complete occlusion of the vessel and subsequent myocardial ischemia, Total occlusion ofthe vessel for more than 4-6 hours resuts in ireversible myocardial necrosis, but repesfusion within this period can salvage the myocardium and Lecksom enorbickts and macaie: 2/24/2014 6:20:37 AM ‘Mat this question & => Question Td : 48903 Question 4 of 30 A 66 year old male present with cardiac arrest due to ventricular rupture. The myocardial change that is the most frequent cause of this is a) Abscess formation and tissue destruction due to nnfective endocarditis +b) Fatty change due to interaction of diphtheria exotoxin and camitine ©) Aschoff bodies associated with henmatic fever 4) Inflammation due to coxsackie B infection Y © €) Necrosis due to coronary artery obstruction Question Explanation: ‘The most common cause of myocardial infarction is narrowing of the coronary blood vessels due to athercmatovs plaques. Plaque riphare with subsequent exposure of the basement membrane results in platzle! aggregation, thrombus formation, fibrin accumulatioa, hhemorrhage into the plague, and varying degrees of vasospasm. This can resut in partial or complete occlusion of the vessel and subsequent myocardial ischemia, Total occlusion ofthe vessel for more than 4-6 hours resuts in ireversible myocardial necrosis, but repesfusion within this period can salvage the myocardium and Lecksom enorbickts and macaie: 2/24/2014 6:20:51 AM ‘Marie this question <= => Question 5 of 30 Question Id : 49784 ‘A. 22 year old female comes to the emergency department because she says, "Im buming up." She is known to staff as an intravenous drug user. On physical examination a systolic heart murmur is detected over the precordiun, An expected physical finding would be a) Decreased intensity of S1 ) Increased intensity of the mumur with deep inspiration ) Increased intensty of the murmur with forced expiration 4) Positive Kussmaul sign (rise in jugular venous pulse with inspiration) ©) Right-sided gallop Question Explanation: An intrevenous drug abuser with high fevers and a cardiac murmur should be considered to have acuts bacterial endocarditis with staphylococcus until proven otnerwise. The valvular lesion most commonly seen in these patients is tricuspid regurgitation. This is a systolic murmur and since it is located on the right side of the hears, it will increase in intensity with inspiration, which increases right heart filing 2/24/2014 6:20:51 AM ‘Marie this question <= => Question 5 of 30 Question Id : 49784 ‘A. 22 year old female comes to the emergency department because she says, "Im buming up." She is known to staff as an intravenous drug user. On physical examination a systolic heart murmur is detected over the precordiun, An expected physical finding would be a) Decreased intensity of S1 Y © b) Increased intensity of the murmur with deep inspiration ) Increased intensty of the murmur with forced expiration 4) Positive Kussmaul sign (rise in jugular venous pulse with inspiration) ©) Right-sided gallop Question Explanation: An intrevenous drug abuser with high fevers and a cardiac murmur should be considered to have acuts bacterial endocarditis with staphylococcus until proven otnerwise. The valvular lesion most commonly seen in these patients is tricuspid regurgitation. This is a systolic murmur and since it is located on the right side of the hears, it will increase in intensity with inspiration, which increases right heart filing 2/24/2014 6:21:05 AM ‘Mark this question = => Question Id : 49965 Question 6 of 30 The interaction commonly seen when a patient on warfarin is given bactrim (Trimethaprim/Sulfamethoxazole) is wich of the following? a) Decreases affect of warfarin, potential for clot formation ) Increases affect of warfarin, causing bleeding ¢) Increases efficacy of bactrim & Decreases efficacy of bactrim (Question Explanation: Taking warfarin ard bactiim may cause more of a chance for bleeding. It has been reported that bactrim can prolong the prothrombin time in patients who are receiving the anticoagulant wacfarin, Using warferin together with sulfamethoxazole is usually not recommended, but may be required in some cases. Potential interaction can cause the patient to feel dizzy or lightheaded, have bloodin the urine, have bloody, black or sticky bowel movements, have unusual bleeding in the vagina, and have wusual bruising 2/24/2014 6:21:05 AM ‘Mark this question = => Question Id : 49965 Question 6 of 30 The interaction commonly seen when a patient on warfarin is given bactrim (Trimetheprin/Sulfarnethoxazole) is which of the following? a) Decreases affect of warfaria, potential fer clot formation Y © ») Increases affect of warfarin, causing bleeding ¢) Tnereases efficacy of bactrim 4d) Decreases efficacy of bactim (Question Explanation: Taking warfarin ard bactiim may cause more of a chance for bleeding. It has been reported that bactrim can prolong the prothrombin time in patients who are receiving the anticoagulant wacfarin, Using warferin together with sulfamethoxazole is usually not recommended, but may be required in some cases. Potential interaction can cause the patient to feel dizzy or lightheaded, have bloodin the urine, have bloody, black or sticky bowel movements, have unusual bleeding in the vagina, and have wusual bruising 2242014 6:21:19 AM ‘Mark this question e& => of 30 Question A70 year old retired farmer complains of difficulty walking because of poor balance and numbness in his fees for three months ‘Neurologic examination shows normal optic fundi, normal strength in all limbs, and down going plantar responses. His gaitis wide based. He is able to stand with his feet together and his eyes open, but begins to fall almost immediately when he closes his eyes, ‘Which investigation is mostlikely to give the correct diagnosis? a) Chest x-ray b) MRI scans of the spinal cord ©) Contrast enhanced brain CT scan 4) Serum Vitamin B12 level @) Blood glucose level Answer | Biolanation | Other User's Explanation Report An Error Question Explanation: Dietary vitamin B12 deficiency uaually results from inadequate absorption, but deficiency can develop in vegans who do not take vitamin supplements, Deficiency causes megaloblastic anemia, damage to the white matter of the spinal cord and brain, and peripheral neuropathy. Anemia usvally develops insidiously. Ttis often more severe than its symptoms indicate because its slow evolution allows physiologic adaptation, Occasionally, splenomegaly and hepatomegaly occur. ‘Neurologic symptoms develop independently from end often without hematologic abnermalitis. In early stages; decreased position and vibratory sensation in the extremities is accompanied by mild to moderate weakness and hyporeflexia In later stages, spasticity, extensor plantar responses, greater loss of position and vibratory sensation in the lower extremities, and ataxia emerge. Diagnosis is usualy mace by measuring serum vitamin B12 levels. The Schilling test helps determine etiology. 2242014 6:21:19 AM ‘Mark this question e& => of 30 Question A70 year old retired farmer complains of difficulty walking because of poor balance and numbness in his fees for three months ‘Neurologic examination shows normal optic fundi, normal strength in all limbs, and down going plantar responses. His gaitis wide based. He is able to stand with his feet together and his eyes open, but begins to fall almost immediately when he closes his eyes, ‘Which investigation is mostlikely to give the correct diagnosis? a) Chest x-ray b) MRI scans of the spinal cord ©) Contrast enhanced brain CT scan Y © & Serum Vitarrin B12 level @) Blood glucose level Answer | Biolanation | Other User's Explanation Report An Error Question Explanation: Dietary vitamin B12 deficiency uaually results from inadequate absorption, but deficiency can develop in vegans who do not take vitamin supplements, Deficiency causes megaloblastic anemia, damage to the white matter of the spinal cord and brain, and peripheral neuropathy. Anemia usvally develops insidiously. Ttis often more severe than its symptoms indicate because its slow evolution allows physiologic adaptation, Occasionally, splenomegaly and hepatomegaly occur. ‘Neurologic symptoms develop independently from end often without hematologic abnermalitis. In early stages; decreased position and vibratory sensation in the extremities is accompanied by mild to moderate weakness and hyporeflexia In later stages, spasticity, extensor plantar responses, greater loss of position and vibratory sensation in the lower extremities, and ataxia emerge. Diagnosis is usualy mace by measuring serum vitamin B12 levels. The Schilling test helps determine etiology. “Mark this question —& => ‘Question Id: 50679 Question 8 of 30 A. 69 year old man comes to the office because of increasing shortness of breath on exertion for the past 2 to 3 months. History of hypertension is present for which he takes hydrochlorothiazide, On physical examination his pulse is 80/min, There is an apical lit displaced to the left and a harsh diastolic murmur atthe base, Which of the following would be present on farther physical examination? a) Bifd pulse 'b) Low-amplitude puise c) Wide pulse pressure ) Pulsus alternans €) Pulsus paradoxs Question Explanation: ‘This patient most likely has acttic insufficiency. One of the hallmark physical findings of this valvular lesion is the presence ofa wide pulse pressure secondary to the diastolic rin-offback into the ventricle. Cther signs such as Quincke's pulse or Musset's sim may also be presents. A bifid pulse is seen with hypertrophic cardiomyopzthy and is best appreciated by palpation of the carotid artery. This bifid pulse occurs ac a result ofno obstruction to bloed flowing out froma the leit heart charrber in the beginaing, followed by an obstruction in the midile of rystole, and Snelly by a lessening of the obstruction atthe end of eystcle. Low amplitude pulse is ceen with peripheral acteriosclerosis. Pulsus altemans where one pulse feels large, the neat pulse fecls sinall, is appreciated with severe congestive heart failure, Pulsus paradomus is an exaggeration of a normally present fallis systolic blood pressure with inspiration Normal decrease in systolic pressure should be 10 mm Hg or less but with pulsus paradozus, it can be 15-20 mm Hg, This is most commonly seen with constrictive or restrictive diseases of the heart or pericardium. “Mark this question —& => ‘Question Id: 50679 Question 8 of 30 A. 69 year old man comes to the office because of increasing shortness of breath on exertion for the past 2 to 3 months. History of hypertension is present for which he takes hydrochlorothiazide, On physical examination his pulse is 80/min, There is an apical lit displaced to the left and a harsh diastolic murmur atthe base, Which of the following would be present on farther physical examination? a) Bifid pulse 'b) Low-amplitude puise Y © 0) Wide pulse pressure ) Pulsus alternans e) Pulus paradoms Question Explanation: ‘This patient most likely has acttic insufficiency. One of the hallmark physical findings of this valvular lesion is the presence ofa wide pulse pressure secondary to the diastolic rin-offback into the ventricle. Cther signs such as Quincke's pulse or Musset's sim may also be presents. A bifid pulse is seen with hypertrophic cardiomyopzthy and is best appreciated by palpation of the carotid artery. This bifid pulse occurs ac a result ofno obstruction to bloed flowing out froma the leit heart charrber in the beginaing, followed by an obstruction in the midile of rystole, and Snelly by a lessening of the obstruction atthe end of eystcle. Low amplitude pulse is ceen with peripheral acteriosclerosis. Pulsus altemans where one pulse feels large, the neat pulse fecls sinall, is appreciated with severe congestive heart failure, Pulsus paradomus is an exaggeration of a normally present fallis systolic blood pressure with inspiration Normal decrease in systolic pressure should be 10 mm Hg or less but with pulsus paradozus, it can be 15-20 mm Hg, This is most commonly seen with constrictive or restrictive diseases of the heart or pericardium. 2/24/2014 6:21:46 AM ‘Mat this question = => Question Td : 51619 Question 9 of 30 A 52 year old femnalz presents to her physician for follow up a fasting serun cholesterol level of 243 mg/dL. She is post menopausal since age 49, and has been not been on hormone replacement therapy. Her family history is positive for coronary astery disease and she has stoked one-half pack of cigarette per day for the past 15 years. Today she would like 1o know the sesults of her fill lipid panel. The lipid panel that would most strongly suggest the need for pharmacologic therapy in this patient is a) Total chelesterol 185 mg/dL. LDL 145 mg/dL. +b) Total cholesterol 235 mg/dL, LDL 115 g/dl, c) Total chelesterol 245 mg/dL, LDL 165 mg/dL. 4d) Total cholesterol 295 mg/dL, LDL 125 mgfdL. Anewor (NEQNSIRAN) Other Users Explanation Repost An Eror Question Explanation: For those patients in whom a fasting panel has been obtained, a stepwise approach to intervention based on the patient's LDL and nck factors may be used, A patient with 2+ nsk factors (this patent) and an LDL of greater than 160 mg/dL. warrants medical therapy. 2/24/2014 6:21:46 AM ‘Mat this question = => Question Td : 51619 Question 9 of 30 A 52 year old femnalz presents to her physician for follow up a fasting serun cholesterol level of 243 mg/dL. She is post menopausal since age 49, and has been not been on hormone replacement therapy. Her family history is positive for coronary astery disease and she has stoked one-half pack of cigarette per day for the past 15 years. Today she would like 1o know the sesults of her fill lipid panel. The lipid panel that would most strongly suggest the need for pharmacologic therapy in this patient is a) Total chelesterol 185 mg/dL. LDL 145 mg/dL. +b) Total cholesterol 235 mg/dL, LDL 115 g/dl, Y © c) Total chelesterol 245 mg/dL, LDL 165 mg/L. 4d) Total cholesterol 295 mg/dL, LDL 125 mgfdL. Anewor (NEQNSIRAN) Other Users Explanation Repost An Eror Question Explanation: For those patients in whom a fasting panel has been obtained, a stepwise approach to intervention based on the patient's LDL and nck factors may be used, A patient with 2+ nsk factors (this patent) and an LDL of greater than 160 mg/dL. warrants medical therapy. 2/24/2014 6:21:57 AM ‘Mack this question & => Question Td : 54621 Question 10 of 30 A 66-year-old obese white diabetic and hypertensive female suffering from heart and renal failure is hospitalized for urosepsis. She has completed a course of TY antibiotics. Her medications are enalapril, furosemide, labetalo, insulin and heparin for fulshing TV lines. She has had nose bleeds for past 2 days. Her platelet count is 28,000/L. Likely cause of her thrombocysopenia is a) Enalapril ) Furosemide c) Lebetalal ) Heparin ) Insulin Anewor (UEQURIRANY Other sors Explanation Report An Eror Question Explanation ‘A number of medications can cause thrombocytopenia, but heparin is a more likely cause than enalepri, furosemide, labetalal, or insulin, Even the small doses of heparin used to flush intravenous lines can be a source of thrombocytopenia. 2/24/2014 6:21:57 AM ‘Mack this question & => Question Td : 54621 Question 10 of 30 A 66-year-old obese white diabetic and hypertensive female suffering from heart and renal failure is hospitalized for urosepsis. She has completed a course of TY antibiotics. Her medications are enalapril, furosemide, labetalo, insulin and heparin for fulshing TV lines. She has had nose bleeds for past 2 days. Her platelet count is 28,000/L. Likely cause of her thrombocysopenia is a) Enalapril ) Furosemide c) Lebetalal Y © d Heparin ) Insulin Anewor (UEQURIRANY Other sors Explanation Report An Eror Question Explanation ‘A number of medications can cause thrombocytopenia, but heparin is a more likely cause than enalepri, furosemide, labetalal, or insulin, Even the small doses of heparin used to flush intravenous lines can be a source of thrombocytopenia. 2/24/2014 6:22:26 AM ‘Mark this question = => Question 11 of 30 The best method of diagnosing an aottic dissection is, a) CT scon >) MRI ¢) TEE CHR, Answer (Botaraion | Other User's Explanation Report An Error Question Explanation: Question Id : 61798 “Aortic dissection is the surging of blood through a tear in the aortic intima with reparation of the intma and media and creation of a false fumen. The intimal tear may be a primery event or secondary to hemorthage within the media. The dissection may occur anywhere along the aorta and extend proximally or distally into other arteries. Hypertension is an important contributor. Symptoms and signs include abrupt onset of tearing chest or back pain, and dissection may result in aortic regurgitation and compromised circulation in branch arteries. Diaguosis is by imaging tests (eg, transesophageal echocardiography, CT angiography, MRI, contrast aortography). MRT is the best test for diagnosis. Treatment always involves ageressive BP control and serial imaging to montor progression of dissection, surgical repair of the aorta and placement of a synthetic graft is needed for ascending aortic dissection and for certain descending aortic dissections. 2/24/2014 6:22:26 AM ‘Mark this question = => Question 11 of 30 The best method of diagnosing an aottic dissection is, a) CT scan Y © >) MRI ©) TEE &) CXR Answer (Botaraion | Other User's Explanation Report An Error Question Explanation: Question Id : 61798 “Aortic dissection is the surging of blood through a tear in the aortic intima with reparation of the intma and media and creation of a false fumen. The intimal tear may be a primery event or secondary to hemorthage within the media. The dissection may occur anywhere along the aorta and extend proximally or distally into other arteries. Hypertension is an important contributor. Symptoms and signs include abrupt onset of tearing chest or back pain, and dissection may result in aortic regurgitation and compromised circulation in branch arteries. Diaguosis is by imaging tests (eg, transesophageal echocardiography, CT angiography, MRI, contrast aortography). MRT is the best test for diagnosis. Treatment always involves ageressive BP control and serial imaging to montor progression of dissection, surgical repair of the aorta and placement of a synthetic graft is needed for ascending aortic dissection and for certain descending aortic dissections. 2/24/2014 6:22:45 AM ‘Mark this question & => Question 12 of 30 A44 year old man presents with acute pleuritic chest pain. He is siting up in bed leaned forward when you enter the examining room. Physical exam reveals afiiction rub on cardiac exam. The most appropriatz in treatment ifthis condition is a) Antarthythnic b) Naprowen c) Beta blocker 4) Albuterol Question Explanation: Pericarditis is inflammation of the pericardium, often with Quid accumulation. Pericarditis may be caused by many disorders (e.g, infection, MI, trauma, tumors, and metabolic disorders) but is often idiopathic. Symptoms include ches: pain or tightness, often worsened by deep breathing. Unlike ischemic chest pain, pain due to pericarditis is usually aggravated by thoracic motion, cough, breathing, or swalowing food; it may be relieved by sitting up and leaning forward, Diagnosis is based on symptoms, a friction rub, ECG changes; and evidence of pericardial fluid accumulation on x-ray or echocardiogram. Finding the cause requires further evaluation. Treatment depends on the cause, but general measures include analgesics (eg codeine), anti- inflammatory drugs (eg naproxea), and sometimes surgery. 2/24/2014 6:22:45 AM ‘Mark this question & => Question 12 of 30 A444 ycar old man presents with acute pleuritic chest pain. He is sitting up in bed leaned forward when you cater the examining room. Physical exam reveals a friction rub on cardiac exam. The most appropriatz in treatment if this condition is a) Antarthythmic Y © b) Naproxen <) Beta blocker d) Albuterol Question Explanation: Pericarditis is inflammation of the pericardium, often with Quid accumulation. Pericarditis may be caused by many disorders (e.g, infection, MI, trauma, tumors, and metabolic disorders) but is often idiopathic. Symptoms include ches: pain or tightness, often worsened by deep breathing. Unlike ischemic chest pain, pain due to pericarditis is usually aggravated by thoracic motion, cough, breathing, or swalowing food; it may be relieved by sitting up and leaning forward, Diagnosis is based on symptoms, a friction rub, ECG changes; and evidence of pericardial fluid accumulation on x-ray or echocardiogram. Finding the cause requires further evaluation. Treatment depends on the cause, but general measures include analgesics (eg codeine), anti- inflammatory drugs (eg naproxea), and sometimes surgery. 2/24/2014 6:22:58 AM “Mark this question & => Question 13 of 30 All of the following are causes of cardiac syncope EXCEPT a) High degree atrioventricular block, ) Perozysmal ventricular tachycardia c) Acute myocardial infarction ) Tricuspid stenosis. €) Aortic stenosis, Answer [REISIBHBNY) otner Users explanation Report An Enor Question Explanation: Cardiac syncope results from a reduction in cardiac output, most commonly caused by arrhythmias. Syacopal episodes associated with the arrhythmia of high-degree block are know es Stokes-Adams-Morgegni syndrome. Myocardial infarction, if massive, may be associated with cardiogenic shock. Aortic stenosis results in exertional syncope die to reduced cardiac output in the presence of peripheral vasodilatation. Tricuspid stenosis presents with dyspnea and fatigue associated with ascites, edema, and hepatomegaly. 2/24/2014 6:22:58 AM “Mark this question & => Question 13 of 30 All ofthe following are canses of cardiac syncope EXCEPT a) High degree atrioventricular block, ) Paroxysmal ventricular tachycardia. c) Acute myocardial infarction oH © d)Treuspid stenosis, ©) Acrtic stenosis Answer [REISIBHBNY) otner Users explanation Report An Enor Question Explanation: Cardiac syncope results from a reduction in cardiac output, most commonly caused by arrhythmias. Syacopal episodes associated with the arrhythmia of high-degree block are know es Stokes-Adams-Morgegni syndrome. Myocardial infarction, if massive, may be associated with cardiogenic shock. Aortic stenosis results in exertional syncope die to reduced cardiac output in the presence of peripheral vasodilatation. Tricuspid stenosis presents with dyspnea and fatigue associated with ascites, edema, and hepatomegaly. 2/24/2014 6:23:10 AM ‘Mare this question = => Question Td : 87563 Question 14 of 30 A44 year old diabetic pays a routine visit to his physician. Pulsatile abdominal mass is observed on physical examination. Radiographic study reveals foci of calcium related to the abdominal mass. The most likely cause that led to the development of the lesion responsible for the abnormal physical finding is a) Atherosclerosis b) Congenital weakness c) Cystic medial necrosis 4) Syphilis ©) Vasculits Answer | Explanation Other User's Explanation Report An Error Question Explanation: This is a typical presentation of an abdominal aortic anewrysm, which is almost always due to severe atherosclerosis. The foci of calcification described occur within the etherosclerotic plaques, and indicate severe atherosclerotic disease. Congenital weakness of vessels can produce berry aneurysins, especially in cerebral vessels in the circle of Wills. Cystic medial necrosis can produce dissecting aneurysms, especially in Marfan syncrome. Syphiltic aneurysms typically involve the aortic root as it leaves the heart. Vascultis can produce aneurysms in small arteries. 2/24/2014 6:23:10 AM ‘Mare this question = => Question Td : 87563 Question 14 of 30 4.44 year old diabetic pays a routine visit to his physician. Pulsatile abdominal mass is observed on physical examination. Radiographic study reveals foci of calcium relaied to the abdominal mass. The most likely cause that led to the development of the lesion responsible for the abnosmal physical finding is Y © a) Atherosclerosis b) Congenital weakness c) Cystic medial necrosis 4) Syphilis 2) Vasculits Answer | Explanation Other User's Explanation Report An Error Question Explanation: This is a typical presentation of an abdominal aortic anewrysm, which is almost always due to severe atherosclerosis. The foci of calcification described occur within the etherosclerotic plaques, and indicate severe atherosclerotic disease. Congenital weakness of vessels can produce berry aneurysins, especially in cerebral vessels in the circle of Wills. Cystic medial necrosis can produce dissecting aneurysms, especially in Marfan syncrome. Syphiltic aneurysms typically involve the aortic root as it leaves the heart. Vascultis can produce aneurysms in small arteries. 2/24/2014 6:23:25 AM ‘Mark this question & => Question 15 of 30 All are advanteges of the internal thoracic artery as coronary astery bypass grafts, EXCEPT a) Vessel size ) In situ connection to the subclavian artery. c) Improved long-term patency. 4) Rapidlty in which proximal anastomosis can be completed, €) Improved long-term survival in persons with left anterior descending coronary artery lesions Question Explanation: Tn coronary artery bypass surgery, a conduit is sumured distal to the obstructing lesion and then connected proximally either naturally (in situ) or to the aorta. Saphenous veins and the intemal thoracic artery are the most common vessels used, Advantages of the internal thoracic artery include its in stu connection to the subclavian artery (B), obviating a proximal anastomosis (D) ‘There is a better size relationship with the coronary artery, especially with the left antenior descending (LAD) artery (A). Long-term patency is better than that with saphenous veins, as is long-term survival in persons with LAD lesions (C). 2/24/2014 6:23:25 AM ‘Mark this question & => Question 15 of 30 All are advantages oftthe internal thoracic attery as coronary astery bypass grafts, EXCEPT a) Vessel size ) In sit connection to the subclavian artery. c) Improved long-term patency. Y © &) Ragidity in which proximal anastomosis can be completed. €) Improved long-term survival in persons with left anterior descending coronary artery lesions Question Explanation: Tn coronary artery bypass surgery, a conduit is sumured distal to the obstructing lesion and then connected proximally either naturally (in situ) or to the aorta. Saphenous veins and the intemal thoracic artery are the most common vessels used, Advantages of the internal thoracic artery include its in stu connection to the subclavian artery (B), obviating a proximal anastomosis (D) ‘There is a better size relationship with the coronary artery, especially with the left antenior descending (LAD) artery (A). Long-term patency is better than that with saphenous veins, as is long-term survival in persons with LAD lesions (C). 2/24/2014 6:23:35 AM ‘Mark this question & => Question Id: 94272 Question 16 of 30 Patients with Wolff Parkinson White syndrome who have episodic symptomatic supraventricular tachycardia or atrial fbrilation benefit most from which of the following? a) Episode intravenous digoxin b) Long-term oral digitalis c) Radioffequency catheter ablation of bypass tracts 4d) Episodic Beta-biockers Question Explanation: Radiofrequency catheter ablation of bypass tracts is possible in over 90% of patients and is safer and more cost effective than surgery, with a similar success sate. Intravenous and oral digowin can shorten the refractory period of the accessory pathway, and increase the ventricular rate, causing ventricular fibrillation, Beta- blockers will not control the ventricular response dunng atrial fibrillation whea conduckon proceeds over the bypass tract. 2/24/2014 6:23:35 AM ‘Mark this question & => Question Id: 94272 Question 16 of 30 Patients with Wollf Parkinson White syndrome who have episodic symptomatic supravextricutar tachycardia or atrial ibrilation ‘benefit most from which of the following? a) Episode intravenous digoxin b) Long-term oral digitalis JY © 2) Radiofrequency catheter ablation of bypass tracts 4) Episodic Beta-blockers Question Explanation: Radiofrequency catheter ablation of bypass tracts is possible in over 90% of patients and is safer and more cost effective than surgery, with a similar success sate. Intravenous and oral digowin can shorten the refractory period of the accessory pathway, and increase the ventricular rate, causing ventricular fibrillation, Beta- blockers will not control the ventricular response dunng atrial fibrillation whea conduckon proceeds over the bypass tract. Ott AM ‘Mark this question & => Question 17 of 30 ‘Which one of the following myocardial pressure and volume relationships is FALSE? a) Volume overload produces dilation, followed by hypertrophy. ») Myocardial diseace produces hypertrophy. c) Pressure overload initially produces concentric hypestrophy. 4) Myocardial disease produces dilatation ©) Pressure overload simultaneously produces diatation and concentric hyperiraphy Answer | Bplanation Other User's Explanation Report An Error Question Explanation: ‘Myocardial adaptation to a volume overload produces dlatation, followed by hypertrophy. Pressure overload produces concentric hypertrophy. Only late in the disease course does dilatation supervene, Primary diseases of the myocardium sirmultencously produce hypertrophy and dilatation, Ott AM ‘Mark this question & => Question 17 of 30 ‘Which one of the following myocardial pressure and volume relationships is FALSE? a) Volume overload produces dilation, followed by hypertrophy. ») Myocardial diseace produces hypertrophy. c) Pressure overload initially produces concentric hypestrophy. 4) Myocardial disease produces dilatation Y © 2) Pressure overload simultaneously produces dilatation and concentric hyperrophy. Answer | Bplanation Other User's Explanation Report An Error Question Explanation: ‘Myocardial adaptation to a volume overload produces dlatation, followed by hypertrophy. Pressure overload produces concentric hypertrophy. Only late in the disease course does dilatation supervene, Primary diseases of the myocardium sirmultencously produce hypertrophy and dilatation, 2/24/2014 6:24:00 AM “Mavk this question & => Question Td 97599 Question 18 of 30 The presence ofa dissecting aneurysm is most closely associated with a) Atherosclerosis b) Marfarn’s disease ©) Ehlers Danlos TV disease 9) Syphiltic aneurysms €) Congenital aneurysms newer (PERERA) otertissr< Explanation Report An For Question Explanation: “Although each of the diseases involved may produce aneurysms, only Marfan's disease is associeted with a dissecting aneurysm. 2/24/2014 6:24:00 AM “Mavk this question & => Question Td 97599 Question 18 of 30 ‘The presence of a dissecting ancurysm is most closely associated with a) Atherosclerosis Y © b)Marfarn’s disease c) Ehlers Danlos IVF disease d) Syphilitic ancurysms e) Congenital eneurysms newer (PERERA) otertissr< Explanation Report An For Question Explanation: “Although each of the diseases involved may produce aneurysms, only Marfan's disease is associeted with a dissecting aneurysm. 2/24/2014 AM ‘Mark this question & => Question 19 of 30 The term "gold standard! refers to a definitive ciagnosis obtained by some independent ineans when interpreting a diagnostic test “Which of the following would not be example of gold standards usefil in clinical practice? a) Coronary angiography in evaluating a cardiac stress test ) Bacteriologic culture in testing a rapid diamostic kit for gonorshea ©) Bronckoscopy with biopsy in evaluating eputum cytelogy 4) Autopsy results in evaluating electrocardiography ¢) Results oflong-term follow-up in a test for early sheumatoid arthritis Avowor (ENGEINI tne veore Explanation Report An Ever Question Explanation: Coronary angiography in evaluating a cardiac stress test, bacteriologic culture in testing a rapid diagnostic kit for gonomhea and bronchoscopy with biopsy in evaluating sputum cytology are all gold standards in diagnosis. Diagnostic stadies for sheumatoid arttis (BA) are hampered by the lack of an independent gold standard for RA. Since the most important clinical features of RA are the persistence of the arthritis and the development of ercsions, arthritis outcome is a clinically relevant gold standard, Diagnosis is @ phased, mutivariable process in which the probablity ofthe presence of disease is updated cortinuously when new diagnostic fedoemaliog in-added by the patient orofile 2/24/2014 AM ‘Mark this question & => Question 19 of 30 The term "gold standard! refers to a definitive ciagnosis obtained by some independent ineans when interpreting a diagnostic test “Which of the following would not be example of gold standards usefil in clinical practice? a) Coronary angiography in evaluating a cardiac stress test ) Bacteriologic culture in testing a rapid diamostic kit for gonorshea ©) Bronckoscopy with biopsy in evaluating eputum cytelogy Y¥ © AD) Autopsy results in cvaluating electrocardiography ¢) Results oflong-term follow-up in a test for early sheumatoid arthritis Avowor (ENGEINI tne veore Explanation Report An Ever Question Explanation: Coronary angiography in evaluating a cardiac stress test, bacteriologic culture in testing a rapid diagnostic kit for gonomhea and bronchoscopy with biopsy in evaluating sputum cytology are all gold standards in diagnosis. Diagnostic stadies for sheumatoid arttis (BA) are hampered by the lack of an independent gold standard for RA. Since the most important clinical features of RA are the persistence of the arthritis and the development of ercsions, arthritis outcome is a clinically relevant gold standard, Diagnosis is @ phased, mutivariable process in which the probablity ofthe presence of disease is updated cortinuously when new diagnostic fedoemaliog in-added by the patient orofile ‘Mark this question = => Question Td: 109889 Question 20 of 30 Which of the following is NOT a clinical feature of pure mural stenosis? a) Dizstolic sumble. b) Dyspnea ©) Right ventricular faire d) Left ventricular failure. €) Graham Stell murmur. Question Explanation: The left ventricle is usuelly protected! in mitral stenosis and therefore dots not hypertrophy or dilate in pure mitral stenosis. A low pitched diastolic rumble with an opening snap is the classic ausculatory finding of mitral stenosis (MS). When MS is severe, it may cause pulmonic regurgitation and the consequent murmur (tnown as the Graham-Steell). Dyspnea and right heart failure are featares of MS, ‘Mark this question = => Question Td: 109889 Question 20 of 30 ‘Which of the following is NOT a clinical feature of pure mitral stenosis? a) Dizstolic sumble. b) Dyspnea ©) Right ventricular fature V © 4) Left ventricular failure ©) Graham Steell murmur Question Explanation: The left ventricle is usuelly protected! in mitral stenosis and therefore dots not hypertrophy or dilate in pure mitral stenosis. A low pitched diastolic rumble with an opening snap is the classic ausculatory finding of mitral stenosis (MS). When MS is severe, it may cause pulmonic regurgitation and the consequent murmur (tnown as the Graham-Steell). Dyspnea and right heart failure are featares of MS, 2242014 AN “Mark this queston & => Question 21 of 30 A.21 year old boy presents with palpitations and his EKG shows a corrected QT interval of 0.7 seconds. The boy is most likely suffering from which one of the following? a) Sheehan's syndrome ) Phummner Vinson syndrome c) Romano Ward smndrome 4) Wolf Parkinson White syndrome €) Tumer’s syndrome Question Explanation Phunmner Vinson syndrome describes an association between severe iron deficiency anemia and espohageal webs. Romano Ward is a prolonged QT syndrome, producing severe ventricular dysrhythmias. Wolff Parkinson White syndrome is caused by an accessory bypass tract (Bundle of Kent) and may cause supraventricular tachycardias, a delta wave is seen, along with a shortened PR interval (nonmal is .12-.20 seconds). Tumer's syndrome is produced by nondisjunction resulting in only one sex chromosome. Sheehan's syndrome is postpartum panhypopitutarism. 2/24/2014 6:24:45 AM “Mark this queston & => Question 21 of 30 A.21 year old boy presents with palpitations and his EKG shows a corrected QT interval of 0.7 seconds. The boy is most likely suffering from which onc of the following? a) Sheehan's syndrome 'b) Plummer Vinson syndrome Y © c) Romano Ward smndrome d) Wolf Parkinson White syndrome ) Turner's syndrome Question Explanation: Phunmner Vinson syndrome describes an association between severe iron deficiency anemia and espohageal webs. Romano Ward is a prolonged QT syndrome, producing severe ventricular dysrhythmias. Wolff Parkinson White syndrome is caused by an accessory bypass tract (Bundle of Kent) and may cause supraventricular tachycardias, a delta wave is seen, along with a shortened PR interval (nonmal is .12-.20 seconds). Tumer's syndrome is produced by nondisjunction resulting in only one sex chromosome. Sheehan's syndrome is postpartum panhypopitutarism. 272472014 6:24:56 AM ‘Mark this question e => Question Td : 136829 Question 22 of 30 ‘With which one of the following is the presence of endavteritis obliterans is most closely associated? a) Osler Weber Renda disease b) Marfan's disease c) Atherosclerosis 4) Beny aneurysms €) Syphilis Question Explanation: Syplliic aneurysms, as seen in tertiary syphilis, occur fom vasculitis that involves the vasa vasora of the aorta, This causes endastents obiterans. None of the other concitions result in endarteritis obliterans, However, Marfan's syndrome is associated with dissecting aneurysms and atherosclerosis, and berry aneurysms also occur 272472014 6:24:56 AM ‘Mark this question e => Question Td : 136829 Question 22 of 30 “With which one of the following isthe presence of endavteritis obliterans is mest closely associated? a) Osler Webder Renda dsease ) Marfan's disease ©) Atherosclerosis &) Beny aneurysms Y © 2) Syphilis Question Explanation: Syplliic aneurysms, as seen in tertiary syphilis, occur fom vasculitis that involves the vasa vasora of the aorta, This causes endastents obiterans. None of the other concitions result in endarteritis obliterans, However, Marfan's syndrome is associated with dissecting aneurysms and atherosclerosis, and berry aneurysms also occur 014 6:25:22 AM ‘Mark this question & => Question 23 of 30 A. woman aged 35 years ded suddenly. Autopsy findings are within nosmal limits except for the heart, which is shown n the accompanying photograph, Which of the followirg is the most likely diggnosis? © tiertace Medios Ecucaton a) Acute endocarditis ) Calcification of the mitral anmulus ©) Merantic endocarditis 4) Mitral valve prolapse e) Myzoma Question Explanation: ‘The floppy valve seen in the photo is characteristic of mitral valve prolapse. Tn this sil idiopathic condition, myxomatous degeneration of the zona fibrosa of the valve structurally weakens the valve leaflets. The disorder has a female predominance, and a midsystolic click is typically heard on auscultation. Most individuals are acymptomatic through life, but complications can include infective eadocardits, valwular insufficiency, errhythmias, and, rarely, sudden death (as in this patient). In acuie endocarditis, valves typicelly show large vegetations. Celcificaiion of the mitral annulus would appear as calcified nodules around the ring’ surrounding the valve. ‘Marantic endocardits causes smal, sterile vegetations along the line of closure of the valve. Cardiac myomasinvolve the chambers of the heart, with the leit atrium being the most commoa site. 014 6:25:22 AM ‘Mark this question & => Question 23 of 30 A. woman aged 35 years ded suddenly. Autopsy findings are within nosmal limits except for the heart, which is shown n the accompanying photograph, Which of the followirg is the most likely diggnosis? © tiertace Medios Ecucaton a) Acute endocarditis ) Calcification of the mitral annulus ©) Marastic endocarditis Y © 4) Mival valve prolapse e) Myxoma Question Explanation: ‘The floppy valve seen in the photo is characteristic of mitral valve prolapse. Tn this sil idiopathic condition, myxomatous degeneration of the zona fibrosa of the valve structurally weakens the valve leaflets. The disorder has a female predominance, and a midsystolic click is typically heard on auscultation. Most individuals are acymptomatic through life, but complications can include infective eadocardits, valwular insufficiency, errhythmias, and, rarely, sudden death (as in this patient). In acuie endocarditis, valves typicelly show large vegetations. Celcificaiion of the mitral annulus would appear as calcified nodules around the ring’ surrounding the valve. ‘Marantic endocardits causes smal, sterile vegetations along the line of closure of the valve. Cardiac myomasinvolve the chambers of the heart, with the leit atrium being the most commoa site. ‘Mark this question = => Question Td : 175264 Question 24 of 30 Examination of a 66 year old man reveals blood pressure of 180/60 mm Hg. Further exam reveals bounding pulses and a high pitched blowing dizstolic mumur, heard best along the left siernal border. What is the most likely diagnosis? a) Aortic regurgitation b) Aortic valve obstruction ©) Cardice tamponade 4) Heart failure €) Hypovolemia Question Explanation: The case described is typical for aortic regurgitation. Tn aortic regurgitation, blood flows backward through the aortic valve during diastole when the valve is closed. The arterial pulse pressure is widened often to over 100 min Hg (nomal pulse pressure is 30 10 50 min Hg), and in extreme cases the systolic pressure can be elevated to over 250 mm Hg with a depression in diastolic pressure. Note that the pulse pressure is 180-60 = 120 mm Hg in this case. The aortic pressure falls greatly during diastole because blood from the aorta regurgitetes backward into the ventricle through the leaky cortic valve. The increase in systolic pressure most likely results from the large increase in stroke volume, which is secondary to an increase in the end diastolic volume; The diastolic murmur of aortic regurgitation is typically a high pitched, blowing, decrescendo murmur, best heard along the left stemal border over the third intercostal space. The pulse pressure is decreased in acrtic valve obstruction, cardiac tamponade, heart failure and hypovolemia ‘Mark this question = => Question Td : 175264 Question 24 of 30 Examination of a 66 year old man reveals blood pressure of 180/60 mm Hg. Further exam reveals bounding pulses and a high pitched ‘blowing diastolic murmur, heard best along the left sternal border. What is the most likely diagnosis? Y © a) Aortic regurgitation 6) Aortic valve obstruction ©) Cardie tamponade 4) Heart failure ©) Hypovolemia Question Explanation: The case described is typical for aortic regurgitation. Tn aortic regurgitation, blood flows backward through the aortic valve during diastole when the valve is closed. The arterial pulse pressure is widened often to over 100 min Hg (nomal pulse pressure is 30 10 50 min Hg), and in extreme cases the systolic pressure can be elevated to over 250 mm Hg with a depression in diastolic pressure. Note that the pulse pressure is 180-60 = 120 mm Hg in this case. The aortic pressure falls greatly during diastole because blood from the aorta regurgitetes backward into the ventricle through the leaky cortic valve. The increase in systolic pressure most likely results from the large increase in stroke volume, which is secondary to an increase in the end diastolic volume; The diastolic murmur of aortic regurgitation is typically a high pitched, blowing, decrescendo murmur, best heard along the left stemal border over the third intercostal space. The pulse pressure is decreased in acrtic valve obstruction, cardiac tamponade, heart failure and hypovolemia ‘Marie this question = => Question Td: 179890 Question 25 of 30 A74 year oldman with angina has recurrent episodes of atrial tachycardia (240/min). A rapid sequence of normal QRS waves is seen on ECG. The episodes are controllable by performance of vagal maneuvers. ‘The most litely etiology of this arrhythmia ss a) Atvial reeatry ) Attomatic atrial conduction, ©) AV dissociation 4) AV nodal reenty e) Wandering atrial pacemaker Question Explanation: This patient has paroxysmal supraventricular tachycardia SVT), which is a regular, rapid (150-250/min) arrhythmia originating in the atria or AV node, AV nodal sentry is the most common cause of this arrhythmia (about 70% of patients). In this condition, the AV node is pathologically divided into two fimctional pathways. The electrical impulse usually proceeds anterograde down the slow pathway and retrograde up the fast pathway. The P waves ate recorded neatly simultaneously with the QRS complexes (which occur in rapid sequence) and are therefore obscured on EKG. This arrhythmia is commorly seen in older patients, about half of whom have underlying heart disease, Reentry PSVTs can be reverted to normal sinus shythm by interrupting the reentry pathway. For example, the performance of vagal maneuvers offen improves the condition by insreasing AV nodal ref-actoriners. Atrial reentry causes about 10% of all PSVTs. The reentrant pathways in the atria. The P wave is recorded before the QRS complex. This arrhythmia is frequently associated with organic heart disease. Automatic airial conduction results in about 5% of PSVTs. Often, they are not parcaysmal, lest days to years, and are resistant to treatment, The P wave has an abnormal configuration, AV dissociation is caused bby accelerated juactional rhythms, in which a normally latent pacsmaker in the AV nodal area depolarizes at a regular, accelerated rate of 60-150/min, When the rate of the accelerated pacemaker approaches the rate of the SA node, the ectopic impulses are often conducted to the ventricles orly, while the atria remain under the control of the SA node, producing AV dissociation. Wandering avrial pacemaker is a usually benign arrhythmia, The dominant pacemaker ‘wanders’ from one atrial focus to another and in and out of the SA node, ‘Marie this question = => Question Td: 179890 Question 25 of 30 A74 year old men with angina has recurrent episodes of atrial tachycardia (24Q/min). A rapid sequence of normal QRS waves is seen on ECG. The episodes are controllable by performance of vagal maneuvers. The most liely etiology ofthis amraythinia is a) Atrial seeatry ) Attomatic atrial conduction, ©) AV dissociation Y¥ © 4) AV nodal reentry ¢) Wandering atriel pacemaker Question Explanation: This patient has paroxysmal supraventricular tachycardia SVT), which is a regular, rapid (150-250/min) arrhythmia originating in the atria or AV node, AV nodal sentry is the most common cause of this arrhythmia (about 70% of patients). In this condition, the AV node is pathologically divided into two fimctional pathways. The electrical impulse usually proceeds anterograde down the slow pathway and retrograde up the fast pathway. The P waves ate recorded neatly simultaneously with the QRS complexes (which occur in rapid sequence) and are therefore obscured on EKG. This arrhythmia is commorly seen in older patients, about half of whom have underlying heart disease, Reentry PSVTs can be reverted to normal sinus shythm by interrupting the reentry pathway. For example, the performance of vagal maneuvers offen improves the condition by insreasing AV nodal ref-actoriners. Atrial reentry causes about 10% of all PSVTs. The reentrant pathways in the atria. The P wave is recorded before the QRS complex. This arrhythmia is frequently associated with organic heart disease. Automatic airial conduction results in about 5% of PSVTs. Often, they are not parcaysmal, lest days to years, and are resistant to treatment, The P wave has an abnormal configuration, AV dissociation is caused bby accelerated juactional rhythms, in which a normally latent pacsmaker in the AV nodal area depolarizes at a regular, accelerated rate of 60-150/min, When the rate of the accelerated pacemaker approaches the rate of the SA node, the ectopic impulses are often conducted to the ventricles orly, while the atria remain under the control of the SA node, producing AV dissociation. Wandering avrial pacemaker is a usually benign arrhythmia, The dominant pacemaker ‘wanders’ from one atrial focus to another and in and out of the SA node, 2/24/2014 6:26:08 AM ‘Mark this question & => Question 26 of 30 A30 year old male undergoes surgery be replacement of a defective aortic valve. The surgery specimen shows bicuspid aortic valve “Which of the following is the likely auscuttatory sign associated with this ebnormality? a) Continuous murmur b) Diatonic nurtnur beginning with a snap ©) Midsystolic cick 4) Midsystolic murmur radiating to the neck. ¢) Pansystolic rnurmur loudest at the apex Answer | Boplanaion Other User's Explanation Report An Error Question Explanation: Bicuspid acrtic valves are clinically silent in childhood or adolescence, but gradually undergo progressive calification and stenosis. By the third or fourth decade, stenosis becomes functionally significant, resulking in left ventricular hypertrophy and progressive pulmonery congestion, On chest auscultation, aortic stenosis manifests with a harsh systolic murmur, which is “diamond shaped" ie, it is more intense in midsystole and is sometiines preceded by an opening click. Radiation of the rmurmur is along the vessels that branch offthe aortic arch, and thus the murmur is audible in the left cervical region. A machinery like continuous murmur is the classic description of the mumur arising from a patent ductus arteriosus. Ibis due to constant shunting of blood ftom the aorta to the pulmonery artery. A diastolic murmur beginning with a snap is bighly characteristic of mitral valve stenosis The snapping sound is due to the sudden opening of the rigid (fibrotic and calcified) leaflets. The murmur has 2 low pitched, rumbiing character and is localized near the apex. Rheumatic disease and infective endocarditis are the two most common causes. Mitral valve prolapse is the most common cause of midsystolc click. Regurgtation occurs in the most severe cases and gives rise to a systolic murmur following the click. The abnormal sound is due to a "floppy" valve, which is pushed back into the atrium (prolapse) by the force of the left ventncular contraction. Pansystolic murmur, loudest at the apestis most likely due to mitral valve insufficiency. ‘This differs from the systolic murmur of aortic stenosis because ofits pansystolic duration, its location near the apex, and absence of accompanying clicks. Again, rheumatic disease and infective endocarditis are the most common underlying etiologies. 2/24/2014 6:26:08 AM ‘Mark this question & => Question 26 of 30 A.30 year old male uadergees surgery be replacement of a defective aortic valve. The surgery specimen shows bicuspid aortic valve. “Which of the fellowing is the likely auscuitatory sign associated with this ebnormality? ) Continous murmur ) Diatonic murmaur beginning with a snap ©) Midsystolic cick Y © 4) Midsystolic murmur radiating to the neck ¢) Pansystolic rmurmur loudest at the apex Answer | Boplanaion Other User's Explanation Report An Error Question Explanation: Bicuspid acrtic valves are clinically silent in childhood or adolescence, but gradually undergo progressive calification and stenosis. By the third or fourth decade, stenosis becomes functionally significant, resulking in left ventricular hypertrophy and progressive pulmonery congestion, On chest auscultation, aortic stenosis manifests with a harsh systolic murmur, which is “diamond shaped" ie, it is more intense in midsystole and is sometiines preceded by an opening click. Radiation of the rmurmur is along the vessels that branch offthe aortic arch, and thus the murmur is audible in the left cervical region. A machinery like continuous murmur is the classic description of the mumur arising from a patent ductus arteriosus. Ibis due to constant shunting of blood ftom the aorta to the pulmonery artery. A diastolic murmur beginning with a snap is bighly characteristic of mitral valve stenosis The snapping sound is due to the sudden opening of the rigid (fibrotic and calcified) leaflets. The murmur has 2 low pitched, rumbiing character and is localized near the apex. Rheumatic disease and infective endocarditis are the two most common causes. Mitral valve prolapse is the most common cause of midsystolc click. Regurgtation occurs in the most severe cases and gives rise to a systolic murmur following the click. The abnormal sound is due to a "floppy" valve, which is pushed back into the atrium (prolapse) by the force of the left ventncular contraction. Pansystolic murmur, loudest at the apestis most likely due to mitral valve insufficiency. ‘This differs from the systolic murmur of aortic stenosis because ofits pansystolic duration, its location near the apex, and absence of accompanying clicks. Again, rheumatic disease and infective endocarditis are the most common underlying etiologies. 2/24/2014 6:26:22 AM ‘Marie this question & => Question Id: 206807 Question 27 of 30 A51 year old female has dyspnoea, a new murmur and fever and is diagnosed with infective endocarditis. Which of the following is associated with the best prognosis? a) Aortic valve infection b) Culhure negative endocarditis c) Low complement levels 4d) Staphylococcus aureus infection @) Streptococcus viridans infection Answer | Explanation Other User's Explanation Report An Error Question Explanation: Features suggestive of a worse prognosis are: + Acute endocarditis (Staphylococcus aureus) Heat failure Intravenous drug abuse (often left and right sided disease) Prosthetic valve infection Tfection of the aortic rather than mitral valve Associated rhythun disturbance Subacute bacterial endocarditis (Streptococcus viridans) has a better prognosis. 2/24/2014 6:26:22 AM ‘Marie this question & => Question Id: 206807 Question 27 of 30 ASI year old female has dyspnoea, a new murmur aad fever and ic clagaosed with infective endocarditis. Which of the fellowing is associated with the best prognosis? a) Aortic valve infection +b) Cuure negative endocarditis c) Low complement levels 4d) Staphylococcus aureus infection JV © ©) Streptococcus viridans infection Answer | Explanation Other User's Explanation Report An Error Question Explanation: Features suggestive of a worse prognosis are: + Acute endocarditis (Staphylococcus aureus) Heat failure Intravenous drug abuse (often left and right sided disease) Prosthetic valve infection Tfection of the aortic rather than mitral valve Associated rhythun disturbance Subacute bacterial endocarditis (Streptococcus viridans) has a better prognosis. Mark this question <> Question Td : 216845 Question 28 of 30 A 59-year-old diabetic has substernal chest pain for 90 minutes radiating to the lef arm and not relieved by nitroglycerine ST- segment elevation exists in leads V1-V/5, Being vitally stable, lungs are clear and no edema exists. Which medicine is most effective in reducing mortality in thie setting? a) ACE inhibitor ) Aspirin c) Beta-blocker 4) Calcium channel blockers €) Morphine 8) Furosemide Question Explanatio “Antiplatelet theraoy has been shown to be of maximum benefitin the setting of an acute myocardial infarction (MD. The preferred zntiplatelet agent is aspirin. The first dose should be given as soon as possible and unless there is a contraindication should be continned indefintely. In a recent review of antiplatelet therapy in acute Ml, antiplatelet therapy was associated with a 30% reduction in vascular events. Tae American College of Cardiology and the American Heart Association both give a Clase recommendation (prospective randonized tials indicating clear evidence of benefit) for use of aspirin in acute myocardial infarction (AMI) ACE inhibitors and beta-blocker: are indicated in patiente with acute Mil and have been shown to reduce mortality. Taers are no kead-to-head comparisons between aspirin and these medications. Patients likely to rective most benefit from ACE inhibitor therapy are those with evidence of left ventricular dysfunction andor beast falure, dott of which are not seen in this patient. Antiptatelet agents, however, are considered to be more beneficial than ACE inhibitors or beta-blockers in this patients setting. ACE inhibitors and beta-blockers generally are given in addition to aspirin Calcium channel blockers have not been shown to reduce mortality in patients with acute MI Jntravenous morphine is given to alleviate pain and anxiety Tncreased pain and anxiety causes sympathetic overactivity that may have a role in plaque rupture and thrombus propagation. ‘There are no studies, however, to show any benedit in mortality wit the use of morphine in acute MI. Morphine also has a role in management of pulmonary edema because it causes venous dilatation Furosemide is used it there is acute heart flure and it will not decrease mortality. Mark this question <> Question Td : 216845 Question 28 of 30 A 59-year-old diabetic has substernal chest pain for 90 minutes radiating to the lef arm and not relieved by nitroglycerine ST- segment elevation exists in leads V1-V/5, Being vitally stable, lungs are clear and no edema exists. Which medicine is most effective in reducing mortality in thie setting? a) ACE inhibitor Y © b) Aspirin c) Beta-blocker 4) Calcium channel blockers €) Morphine 8) Furosemide Question Explanatio “Antiplatelet theraoy has been shown to be of maximum benefitin the setting of an acute myocardial infarction (MD. The preferred zntiplatelet agent is aspirin. The first dose should be given as soon as possible and unless there is a contraindication should be continned indefintely. In a recent review of antiplatelet therapy in acute Ml, antiplatelet therapy was associated with a 30% reduction in vascular events. Tae American College of Cardiology and the American Heart Association both give a Clase recommendation (prospective randonized tials indicating clear evidence of benefit) for use of aspirin in acute myocardial infarction (AMI) ACE inhibitors and beta-blocker: are indicated in patiente with acute Mil and have been shown to reduce mortality. Taers are no kead-to-head comparisons between aspirin and these medications. Patients likely to rective most benefit from ACE inhibitor therapy are those with evidence of left ventricular dysfunction andor beast falure, dott of which are not seen in this patient. Antiptatelet agents, however, are considered to be more beneficial than ACE inhibitors or beta-blockers in this patients setting. ACE inhibitors and beta-blockers generally are given in addition to aspirin Calcium channel blockers have not been shown to reduce mortality in patients with acute MI Jntravenous morphine is given to alleviate pain and anxiety Tncreased pain and anxiety causes sympathetic overactivity that may have a role in plaque rupture and thrombus propagation. ‘There are no studies, however, to show any benedit in mortality wit the use of morphine in acute MI. Morphine also has a role in management of pulmonary edema because it causes venous dilatation Furosemide is used it there is acute heart flure and it will not decrease mortality. ‘Mark this question => Question Id: 217064 Question 29 of 30 A.45-year-old man who is feeling dizzy has the folowing ECG Aredhel merle Intetce Macca Edscaton His blood pressure is 70/40mm Hy. Which of the noe is the most appropriate next step in management? a) Intravenous digoxin +) Start intravenous ditiazem drip 6) Start intravenous heparin deip 4) Stact warfarin ©) Synchronized cardioversion 8) Tasynchronized cardioversion Answer | Explanation Other User's Explanation Report An Error Question Explanation: In patients who have atrial fbrilation and hemodynamic instabilty, the treatment of choice is immediate synchronized cardioversion Hemodynamic compromise may be manifested clinically by hypotension and shock and by worsening angina pectoris, shortness of breath, or heart failure, Synchronized cardioversion is also indicated in alrial Quiter, atial tachycardia, and supraventricular tachycardia. Digoxin and ditiazem both can be used for rate control in atrial fibrilltion, Ta this patient, however, who is manifesting hemodynarnic instability, cardioversion is the most appropriate next step in management Anticoagulation with heparin acutely and warfarin (Coumadin) in the long tesm alse is recommended in patients who have attial fibrillation. Cardioversion, however, is a priority at this Point. ‘Unsynchronized cardioversionis used only in patients who have ventricular fibrillation or pulseless ventricular tachycardia ‘Mark this question => Question Id: 217064 Question 29 of 30 A.45-year-old man who is feeling dizzy has the folowing ECG Aredhel merle Irnace Mecca Esteatn His blood pressure is 7/4¢mm Hg, Which of the ee is the most appropriate next step in management? a) Intravenous digoxin +) Start intravenous ditiazem drip c) Start intravenous heparin drip 4) Stact warfarin Y¥ © ©) Synchronized cardioversion £) Unsynchronized cardioversion Answer | Explanation Other User's Explanation Report An Error Question Explanation: In patients who have atrial fbrilation and hemodynamic instabilty, the treatment of choice is immediate synchronized cardioversion Hemodynamic compromise may be manifested clinically by hypotension and shock and by worsening angina pectoris, shortness of breath, or heart failure, Synchronized cardioversion is also indicated in alrial Quiter, atial tachycardia, and supraventricular tachycardia. Digoxin and ditiazem both can be used for rate control in atrial fibrilltion, Ta this patient, however, who is manifesting hemodynarnic instability, cardioversion is the most appropriate next step in management Anticoagulation with heparin acutely and warfarin (Coumadin) in the long tesm alse is recommended in patients who have attial fibrillation. Cardioversion, however, is a priority at this Point. ‘Unsynchronized cardioversionis used only in patients who have ventricular fibrillation or pulseless ventricular tachycardia ‘Mark this question a Question Id: 217174 Question 30 of 30 A 59-year-old hypertensive with lef-sided chest pain has crackles at the bases of his lungs and an $4. ECG shows 2-mrn ST elevation, Thrombolytics, aspirin and Metoprolol are started. Cardiac enzymes and troponin are merkedly elevated, EF=30% and a large anterior wall motion abnormality exists. Which of the following medications should be added? a) Amiodarone b) Ditiazem ©) Lidocaine 4) Lisinopril ©) Procainamide Question Explanatior ‘The patient is status post a large myocardial infarction (Mf) and at risk for left ventricular dilation and aneurysm formation, Addition of an ACE inhibitor or angiotensin receptor blocker (ARB) will be beneficial in preventing remodeling Thus lisinopril will be useful in preventing-or at least minimizing-the formation of an aneurysm, ACE inhibitors have also been proven to increase survival in patients with an ejection fraction of 40% “Amiodarone is usefil in the setting of an arrhythmia such as atrial fibrillation, supraventricular tachycarcia, or ventricular tachycardia However, in the absence of such a disorder it is natindicated “A calcium channel blocker such as diltiazem is not usually indicated in patients with heart failure or in patients who have recently suffered an ML Tn fact, due to their negative inotropic effect, short-acting calcium channel blockers have been shown to increase mortality in patients with a low ejection fection. Lidocaine is indicated in the management of ischemic-rslated arrhythmias. This paticnt is not presenting with an acute arrhythmia such as V-fib or tach, so lidocaine is not recommended at this time. Ifhe suddenly developed one of these arrhythmias, then this should be the frst choice. Procainanide and other antiarrhythinics are not autornatically indicated in patients after an MI. Indeed, these patients are at risk for ‘malignant arrhythmias. However, a beta-blocker would be preferable Recent studies have supported the early use of a defibrillator if there is evidence of malignant arrhythmias, even after an ischemic episode is reversed, ‘Mark this question a Question Id: 217174 Question 30 of 30 A 59-year-old hypertensive with lef-sided chest pain has crackles at the bases of his lungs and an $4. ECG shows 2-mrn ST elevation, Thrombolytics, aspirin and Metoprolol are started. Cardiac enzymes and troponin are merkedly elevated, EF=30% and a large anterior wall motion abnormality exists. Which of the following medications should be added? a) Amiodarone b) Ditiazem ©) Lidocaine Y © d)Lisinopri ©) Procainamide Question Explanatior ‘The patient is status post a large myocardial infarction (Mf) and at risk for left ventricular dilation and aneurysm formation, Addition of an ACE inhibitor or angiotensin receptor blocker (ARB) will be beneficial in preventing remodeling Thus lisinopril will be useful in preventing-or at least minimizing-the formation of an aneurysm, ACE inhibitors have also been proven to increase survival in patients with an ejection fraction of 40% “Amiodarone is usefil in the setting of an arrhythmia such as atrial fibrillation, supraventricular tachycarcia, or ventricular tachycardia However, in the absence of such a disorder it is natindicated “A calcium channel blocker such as diltiazem is not usually indicated in patients with heart failure or in patients who have recently suffered an ML Tn fact, due to their negative inotropic effect, short-acting calcium channel blockers have been shown to increase mortality in patients with a low ejection fection. Lidocaine is indicated in the management of ischemic-rslated arrhythmias. This paticnt is not presenting with an acute arrhythmia such as V-fib or tach, so lidocaine is not recommended at this time. Ifhe suddenly developed one of these arrhythmias, then this should be the frst choice. Procainanide and other antiarrhythinics are not autornatically indicated in patients after an MI. Indeed, these patients are at risk for ‘malignant arrhythmias. However, a beta-blocker would be preferable Recent studies have supported the early use of a defibrillator if there is evidence of malignant arrhythmias, even after an ischemic episode is reversed, ‘Mark this question => Question Id: 22970 Question 1 of 30 A-man aged 55 presents with pain in both lus lower limbs. A weak popliteal artery pulse is noted on examination, What is the appropriate next step? a) Referral to vascular surgeon 1b) Angiogram ©) Prescribe anticoagulant 4) X-ray of legs ©) Prescribe NSAIDs Question Explanation: ‘The pulse of the popliteal artery can be palpated behind the knee. A weak pulse may indicate a blockage or thrombosis, Popliteal artery occlusion and the disease occurs by the decreased or complete blockage of the blood supply though the popliteal artery and into the lower leg and foot Tissue ischemia results. Clandication usually is the first manifestation, followed by rest pain and. tissue loss (gangrene). Once a portion of a lower extremity hecames gangrenous, infection becomes a constant mortal threat _Angography is the criterion standard evaluation for identifying popliteal occlnsion, It also allows visualization of possible targets for Pescara ce ‘Mark this question => Question Id: 22970 Question 1 of 30 A-man aged 55 presents with pain in both lus lower limbs. A weak popliteal artery pulse is noted on examination, What is the appropriate next step? a) Referral to vascular surgeon Y © b) Angiogram ©) Prescribe anticoagulant 4) X-ray of legs ©) Prescribe NSAIDs Question Explanation: ‘The pulse of the popliteal artery can be palpated behind the knee. A weak pulse may indicate a blockage or thrombosis, Popliteal artery occlusion and the disease occurs by the decreased or complete blockage of the blood supply though the popliteal artery and into the lower leg and foot Tissue ischemia results. Clandication usually is the first manifestation, followed by rest pain and. tissue loss (gangrene). Once a portion of a lower extremity hecames gangrenous, infection becomes a constant mortal threat _Angography is the criterion standard evaluation for identifying popliteal occlnsion, It also allows visualization of possible targets for Pescara ce ‘Mark this question & => Question ? of 30 ‘You are asked! to interpret this EKG: What is your answer? ' aA a) Left Bundle branch block & Left axis deviation b) Lef Bundle branch & Right axis deviation c) Right Bundle branch block & Right asic deviation 4) Right Bundle branch block & Lett axis deviation Question Explanation: ‘This EKG clearly shows a right bundle branch block, look at lead V1. The axis is calculated as follows: Look at the net amplitude in leads | and aVF, and then find which quadrant this falls in Both I end aVE positive = normal axs Both I and aVF negative = axis inthe Northwest Tenitory Lead I negative and aVVF positive = right axis deviation Lead I postive and aVE negative = left axis deviation Question Td : 46504 Canses of a Northwest axis (no man’s lend): empkyseme, hypeckaleria, lead transposition, artiicial cardiac pacing, veatricular tachycardia. Causes of right axis deviation: normal finding in children and tall thin adults, right ventricular hypestrophy, COPD, anterolateral MI, left posterior hemiblock, pulmonary embolas, Wolff Parkinson- White syndrome, ASD, VSD. Canses of left axis deviation: left anterior hemiblock, Q waves cfiiferior myocardial infarction, ertificial cardiac pacing, hyperkalemia and tricuspid atresia ‘Mark this question & => Question ? of 30 ‘You are asked! to interpret this EKG: What is your answer? ' aA a) Left Bundle branch block & Left axis deviation b) Lef Bundle branch & Right axis deviation c) Right Bundle branch block & Right asic deviation V © 4 Right Bundle branch block & Let axis deviation Question Explanation: ‘This EKG clearly shows a right bundle branch block, look at lead V1. The axis is calculated as follows: Look at the net amplitude in leads | and aVF, and then find which quadrant this falls in Both I end aVE positive = normal axs Both I and aVF negative = axis inthe Northwest Tenitory Lead I negative and aVVF positive = right axis deviation Lead I postive and aVE negative = left axis deviation Question Td : 46504 Canses of a Northwest axis (no man’s lend): empkyseme, hypeckaleria, lead transposition, artiicial cardiac pacing, veatricular tachycardia. Causes of right axis deviation: normal finding in children and tall thin adults, right ventricular hypestrophy, COPD, anterolateral MI, left posterior hemiblock, pulmonary embolas, Wolff Parkinson- White syndrome, ASD, VSD. Canses of left axis deviation: left anterior hemiblock, Q waves cfiiferior myocardial infarction, ertificial cardiac pacing, hyperkalemia and tricuspid atresia 2/24/2014 6:30:34 AM ‘Mark this question = => Question Td : 48763 Question 3 of 30 A.75 year cld man presents with pain in the right calf that recws on a regular basis. He is hypertensive and smokes 1 pack of cigarettes per day. He has a history of previous heart atiacks but s otherwise in fine health. Tae fincing that would support a diagnostic impression of peripheral vascular disease is a) Pain during rest exercise and the presence of swelling and soreness behind the knee and in the calf +) Pain that begins immediately upon walking and is uarelieved by stress ©) Doppler waveform analysis showing accentuated waveforms at a point of decreased blood flow 4) Treadwall arterial pressure immediately following exercise ) An ankle-brachial index of 1.15 Question Explanation: Peripheral vascular disease (PVD) is a clinical manifestation of artherosclserotic disease and is caused by occlusion ofthe arteries to the legs. Patients with significant arterial occlusive disease wil have 2 prominent decease in the ankle-brachial index from baseline folowing exercise, and usually a 20-mm Hg or greater decrease in systolic blood pressure. Pain during rest and exercise and the presence of swelling and soreness behind the knee and in the calfis found in those with Baker's cysts. Peripheral nerve pain commonly begns immediately upon walking and s uarelieved by rest. Doppler waveform anclysis is useful inthe diagnosis of PVD and will reveal attenuated waveform at a point of decreased blood flow. Employment of the ankle-brachial index is encouraged in daily practice as a simple means to diagnose the presence of PVD. Generally, ankle-brackial indices in the range of 0.91-1.30 are thought to be normal 2/24/2014 6:30:34 AM ‘Mark this question = => Question Td : 48763 Question 3 of 30 A.75 year cld man presents with pain in the right calf that recws on a regular basis. He is hypertensive and smokes 1 pack of cigarettes per day. He has a history of previous heart atiacks but s otherwise in fine health. Tae fincing that would support a diagnostic impression of peripheral vascular disease is a) Pain during rest exercise and the presence of swelling and soreness behind the knee and in the calf +) Pain that begins immediately upon walking and is uarelieved by stress ©) Doppler waveform analysis showing accentuated waveforms at a point of decreased blood flow Y © @) Treadmill arterial pressure immediately following exercise ) An ankle-brachial index of 1.15 Question Explanation: Peripheral vascular disease (PVD) is a clinical manifestation of artherosclserotic disease and is caused by occlusion ofthe arteries to the legs. Patients with significant arterial occlusive disease wil have 2 prominent decease in the ankle-brachial index from baseline folowing exercise, and usually a 20-mm Hg or greater decrease in systolic blood pressure. Pain during rest and exercise and the presence of swelling and soreness behind the knee and in the calfis found in those with Baker's cysts. Peripheral nerve pain commonly begns immediately upon walking and s uarelieved by rest. Doppler waveform anclysis is useful inthe diagnosis of PVD and will reveal attenuated waveform at a point of decreased blood flow. Employment of the ankle-brachial index is encouraged in daily practice as a simple means to diagnose the presence of PVD. Generally, ankle-brackial indices in the range of 0.91-1.30 are thought to be normal 2/24/2014 6:30:52 AM ‘Mark this question & => Question Td : 49681 Question 4 of 30 ‘You are measuring the blood pressure in an adult patient with suspected hypertension. For an accurate measurement which one of the following is recommended? a) Use a mercury manometer with the column at atrial level ‘b) Choose a cuff with bladder width about 40% of arm circumference c) Increase cuff pressure rapidly until korotkoff sound disappear then release for measurement 4) The rate of drop of mercury should be 2 mm per second e) Record the pressure at which the radial artery pulse first appears (systolic pressure) Question Explanation: “When measuring the blood pressure the cuff is inflated until the blood flow through the brachial artery stops. With sufficient compression, the cuff outs off blood flow through the artery, and.no sound is heard in the stethoscope. The pressure in the cuff is increased rapidly to 30 milimeters of mercury above the point that no blood flow is taking place through the cuff when no sound can tbe heard in the stethoscope or when a pulse can no longer be felt in the wrist. ‘Then valve is loosened in the bulb and to lessen the air pressure. Pressure is then decreased so that the rate of drop is 2 millineters per second When the pressure falls to the point that blood begins to flow through the artery again, the number that the column of mercury has risen to at the, first sound heard in the stethoscope is the systolic blood pressure (SBP), the first mumber in the blood pressure reading Look: at the column of mercury to see the number at that pressure point, When the cuff decompresses to the point that blood flows frvely in the artery, the sound is no longer heard in the stethoscope. The number next to the top of the column of mercury when the sound ceases is the diastolic blood pressure (DBP), the second number in the blood pressure reading, Again look at the column of mercury to see the number at that pressure point, Record the SBP and the DBP numbers inunediately (don’t depend on memory), and note the acm (right or left) used for taking the measurement is noted, 2/24/2014 6:30:52 AM ‘Mark this question & => Question Td : 49681 Question 4 of 30 ‘You are measuring the blood pressure in an adult patient with suspected hypertension. For an accurate measurement which one of the following is recommended’ a) Use a mercury manometer with the column at atrial level ‘b) Choose a cuff with bladder width about 40% of arm circumference c) Increase cuff pressure rapidly until korotkoff sound disappear then release for measurement V © A) The rate of drop of mercury should be 2mm per second e) Record the pressure at which the radial artery pulse first appears (systolic pressure) Question Explanation: “When measuring the blood pressure the cuff is inflated until the blood flow through the brachial artery stops. With sufficient compression, the cuff outs off blood flow through the artery, and.no sound is heard in the stethoscope. The pressure in the cuff is increased rapidly to 30 milimeters of mercury above the point that no blood flow is taking place through the cuff when no sound can tbe heard in the stethoscope or when a pulse can no longer be felt in the wrist. ‘Then valve is loosened in the bulb and to lessen the air pressure. Pressure is then decreased so that the rate of drop is 2 millineters per second When the pressure falls to the point that blood begins to flow through the artery again, the number that the column of mercury has risen to at the, first sound heard in the stethoscope is the systolic blood pressure (SBP), the first mumber in the blood pressure reading Look: at the column of mercury to see the number at that pressure point, When the cuff decompresses to the point that blood flows frvely in the artery, the sound is no longer heard in the stethoscope. The number next to the top of the column of mercury when the sound ceases is the diastolic blood pressure (DBP), the second number in the blood pressure reading, Again look at the column of mercury to see the number at that pressure point, Record the SBP and the DBP numbers inunediately (don’t depend on memory), and note the acm (right or left) used for taking the measurement is noted, 272472014 6:31.07 AM ‘Mark this question = => Question Td : 54611 Question 5 of 30 The antihypertensive agent that may help preserve bone mineral density is a) Atenolol (Lenormi) ) Doxazosin (Carchura) c) Enalapril (Vasotec) ) Hydrochlorothiazide ©) Nifedipine Frocarda, Adalat) (Question Explanation: Tn healthy older adults, low-dose hydrochlorothiazide preserves bone mineral density of the hip and spine. The modest effects observed over 3 years, if accumulated over 10-20 years, may explain the one-third reduction in risk for hp fracture associated with thiazide use in many epidemiologic stuckes. There are no such benefits reported for the other medications listed. 272472014 6:31.07 AM ‘Mark this question = => Question Td : 54611 Question 5 of 30 The antihypertensive agent that may help preserve bone mineral density is a) Atenolol (Lenormi) ) Doxazosin (Carchura) c) Enalapril (Vasotec) Y © &) Hydrochlorethiaride ©) Nifedipine Frocarda, Adalat) (Question Explanation: Tn healthy older adults, low-dose hydrochlorothiazide preserves bone mineral density of the hip and spine. The modest effects observed over 3 years, if accumulated over 10-20 years, may explain the one-third reduction in risk for hp fracture associated with thiazide use in many epidemiologic stuckes. There are no such benefits reported for the other medications listed. ‘Mate this question & => (Question Td : 55352 Question 6 of 30 A.62 year old develops typical symptoms of angina. He also complains of occesional lightheadedness during exercise. On physical examination, blood pressure is 140/80 mmHg, pulse is 80/minute and reguler. These is a left ventricular heave and a grade 416 systolic ejection murmur heard at the second right interspace and radiating to the neck. CXR shows cardiac enlargement and the electrocardiogram shows left ventricular hypertrophy. The correct course of management is which of the following? a) Begin digtalis. a diuretic end nitroglycerin and reassess in 1 month b) Begin isosorbide dinitrate c) Refer for cardiac catheterization and consideration for cardiac surgery 4) Begin digtalis and nitroglycerin and reassess in 1 month ©) Begin nitroglycerin and propranolol and reassess in 1 month Question Explanation: Hypertrophic cardiomyopathy (HCM) is a complex type ofheart disease marked by thickening of the heart muscle, left ventricular stiffness, mitral valve changes and cellular changes. Symptoms can occur at any age and may include: chest pain or pressure that usually occurs with exercise or physical actwvity, but also may cccur with rest or afier meals. Echocardiogram is the most common testused to diagnose HOM, as the characteristic thickening of the heart walls is usually visible on the echo. Other tests may include blood tests, electrocardograrn, chest x-ray, echocardiogram, exercise stress test and cardiac catheterization ‘Treatment can involve a septal myectomy. During this surgical procedure, the surgeon removes a small amount of the thickened septal wallto widen the outflow tract (the path the blood takes) from the lef ventricle to the aorta. Myectomy is, considered when medications are not effective in treating HCM. ‘Mate this question & => (Question Td : 55352 Question 6 of 30 A.62 year old develops typical symptoms of angina. He also complains of occasional lightheadedness during exercise. On physical examination, blood pressure is 140/80 mmHg, pulse is 80/minute and reguler. These is a left ventricular heave and a grade 416 systolic ejection murmur heard at the second right interspace and radiating to the neck. CXR shows cardiac enlargement and the electrocardiogram shows left ventricular hypertrophy. The correct course of management is which of the following? a) Begin digtalis, a diuretic end nitroglycerin and reassess in 1 ronth b) Beginisosorbide dinitrate ¥ © c) Refer for cardiac catheterization and consideration for cardiac surgery 4) Begin digtalis and nitroglycerin and reassess in 1 month €) Begin nitroglycerin and propranclol and reassess in 1 month Question Explanation: Hypertrophic cardiomyopathy (HCM) is a complex type ofheart disease marked by thickening of the heart muscle, left ventricular stiffness, mitral valve changes and cellular changes. Symptoms can occur at any age and may include: chest pain or pressure that usually occurs with exercise or physical actwvity, but also may cccur with rest or afier meals. Echocardiogram is the most common testused to diagnose HOM, as the characteristic thickening of the heart walls is usually visible on the echo. Other tests may include blood tests, electrocardograrn, chest x-ray, echocardiogram, exercise stress test and cardiac catheterization ‘Treatment can involve a septal myectomy. During this surgical procedure, the surgeon removes a small amount of the thickened septal wallto widen the outflow tract (the path the blood takes) from the lef ventricle to the aorta. Myectomy is, considered when medications are not effective in treating HCM. 2/24/2014 AM ‘Mark this question & => Question 7 of 30 A 61-year-old men with a history of controlled hypertension presents with acute onset weakness of his left arm that resolved over 12 hours, He had suffered two similar cpisodes over last three months. Blood pressure is 134/80 mmiig and he is in airial fibrillation with, a ventricular rate of 85 per minute. CT brain scan is normal, Whatis the most appropriate management? a) Warfarin b) Labetadol ) Aspirin 9) Digoxin €) Dipyridarmole Question Explanation: This patient has had three transient ischemic attacks cue to atrial fibrillation. The most appropriate therapeutic strategy for this patient would be warfarin, Studies teveal that warfarin would be therapeuticaly, superior to aspirin in such a patient's case 2/24/2014 AM ‘Mark this question & => Question 7 of 30 A 61-year-old man with a history of controlled hypertension presents with acute onset weakness of his le8 arm that resolved over 12 hours, He had sufered two sinilar episodes over last three months, Blood pressure is 134/00 mumllg and he is ia atrial Sbrillation with a ventricular rate of 85 per minute, CT brain scan is normal, Whatis the most appropriate management? Y © a) Warfarin 'b) Labetlol ©) Aspirin 4) Digoxia ©) Dipyridamole Question Explanation: This patient has had three transient ischemic attacks cue to atrial fibrillation. The most appropriate therapeutic strategy for this patient would be warfarin, Studies teveal that warfarin would be therapeuticaly, superior to aspirin in such a patient's case 2/24/2014 6:3152 AM ‘Mark this question << => Question Id : 59451 Question 8 of 30 A.55 year dld black male with long stancing hypertension and a 30 pack year smoking history has a history of dyspnea on exertion for 2 days, Physical ezaminatioa is unremarkable except for rare crackles at the bases. The serclogic test that would be most helpful for detecting let venticular dysfunction is a) Beta Natrinretic peptide (BNP) b) Troponin-T ¢) C-reactive proten (CRP) 4) D-dimer ©) Cardiac interleukin-2 Question Explanation: Beta-Netriuretic peptide (BNE) is a 32-amino acid polypeptide secreted from the cardiac ventricles in response to ventricular volume expansion and pressure overload, The major source of BNP is the cardiac ventricles, and because ofits minimal presence in storage gramules, is release is directly proportional to ventricular dysfunction. Tris a simple and rapid test that reliable predicts the presence or absence of left ventricular dysfnctton on an echocarcograrn. 2/24/2014 6:3152 AM ‘Mark this question << => Question Id : 59451 Question 8 of 30 A.55 year dld black male with long stancing hypertension and a 30 pack year smoking history has a history of dyspnea on exertion for 2 days, Physical ezaminatioa is unremarkable except for rare crackles at the bases. The serclogic test that would be most helpful for detecting let venticular dysfunction is Y © a) Beta Natriaretic peptide BNP) b) Troponin-T ¢) C-reactive proten (CRP) 4) D-dimer e) Cardiac interleulin-2 Question Explanation: Beta-Netriuretic peptide (BNE) is a 32-amino acid polypeptide secreted from the cardiac ventricles in response to ventricular volume expansion and pressure overload, The major source of BNP is the cardiac ventricles, and because ofits minimal presence in storage gramules, is release is directly proportional to ventricular dysfunction. Tris a simple and rapid test that reliable predicts the presence or absence of left ventricular dysfnctton on an echocarcograrn. 2/24/2014 6:32:09 AM ‘Matte this question & => Question Td : 59853 Question 9 of 30 2.45 year old man who is being treated with hycrochlorothiaside (HCTZ) for control of mild edema presents complaining of malaise, fatigue, muscular weakness, and muscle cramps. The most likely cause of the patient's symptoms is a) Hyperglycemia b) Hyperuricemia c) Drug hypersensitivity 4) Hyperkalemia ©) Hypokalemia Answer | Explanation Other User's Explanation Report An Error Question Explanation: Hyper GLUC is a mnemonic used to remember the side effects of ICTZ, GLUC stands for hyperglycemia, hyperlipidemia, hyperuricemia, and hypercalcemia. ‘Thiazide diuretics increase the excretion of water by inkibiting the reabsorption of sodiuin and chloride ions at the distal renal tubule ‘The natriuretic effects are accompanied by a secondary loss of potassium and bicarbonate which can cause a mild hypokalemic, hypochloremic, metabolic alkalosis. 2/24/2014 6:32:09 AM ‘Matte this question & => Question Td : 59853 Question 9 of 30 A.45 year old man who is being treated with hycrochlorothiaside (HCTZ) for control of mild edema presents complaining of malaise, fatigue, muscular weakness, and muscle cramps. The most likely cause of the patient's symptoms is a) Hyperglycemia 'b) Hypenuricemia c) Drug hypersensitivity 4) Hyperkalemia Y © ©) Hypokalemia Answer | Explanation Other User's Explanation Report An Error Question Explanation: Hyper GLUC is a mnemonic used to remember the side effects of ICTZ, GLUC stands for hyperglycemia, hyperlipidemia, hyperuricemia, and hypercalcemia. ‘Thiazide diuretics increase the excretion of water by inkibiting the reabsorption of sodiuin and chloride ions at the distal renal tubule ‘The natriuretic effects are accompanied by a secondary loss of potassium and bicarbonate which can cause a mild hypokalemic, hypochloremic, metabolic alkalosis. 2/24/2014 6:32:23 AM ‘Mark this question ez Question Td Question 10 of 30 2500 ‘The statement that favors 2 diagnosis of cardiogenic shock in a patient with acute myocardial infarction is which of the Following? a) Acute pulmonary edema refractory to standard treatment ) Ventricular arrhythmia and hypotension ©) Hypotension and low central venous pressure 4) Hypotension and sinus bradycardia €) Hypotension and high central venous pressure Question Explanation Cardiogenic shock is based upon an inadequate circulation of blood due to primary failure of the ventricles of the heart to function effectively. Since this is a category of shock there is insufficient perfusion of tissue (ie. the heart) to meet the required demand for oxygen and ‘mutrients. This leads to cell death from oxygen starvation, hypoxia Because of his it may lead to cardiac arrest (or circulatory arrest) ‘which is an acute cessation of cardiac pump function, Cardiogenic shock is defined by sustained hypotension with tissue hypoperfusion despite adequate left ventricular filing pressure Signs of tissue hypoperfusion nclade olguria («30 mL/h), cool extremities, and altered mentation, hypotension due to decrease in cardiac output, rapid, weak, thready pulse due to decreased circulation combined with tachycerdia and distended jugular veins due to increased jugular venous pressure. 2/24/2014 6:32:23 AM ‘Mark this question ez Question Td Question 10 of 30 2500 ‘The statement that favors a diagnosis of cardiogenic shock in a patient with acute myocardial infarction is which ofthe following? a) Acute pulmonary edema seffactory to standard treatment ) Ventricular arrhythmia and hypotension ©) Hypotension and low central venous pressure 4) Hypotension and sinus bradycardia Y © e) Hypotension and high certral venous pressure Question Explanation Cardiogenic shock is based upon an inadequate circulation of blood due to primary failure of the ventricles of the heart to function effectively. Since this is a category of shock there is insufficient perfusion of tissue (ie. the heart) to meet the required demand for oxygen and ‘mutrients. This leads to cell death from oxygen starvation, hypoxia Because of his it may lead to cardiac arrest (or circulatory arrest) ‘which is an acute cessation of cardiac pump function, Cardiogenic shock is defined by sustained hypotension with tissue hypoperfusion despite adequate left ventricular filing pressure Signs of tissue hypoperfusion nclade olguria («30 mL/h), cool extremities, and altered mentation, hypotension due to decrease in cardiac output, rapid, weak, thready pulse due to decreased circulation combined with tachycerdia and distended jugular veins due to increased jugular venous pressure. ‘Mark this question & => Question 11 of 30 A76 year old othervise healthy woman states that she has passed out three in the last month while walling briskly during her daily walk. with the local senior citizens mall walkers club. The most likely etiology ofher syndrome according to the history is a) Vasovagal syncope ) Transient ischemic attack c) Aortic stenosis 4) Orthostatis hypotension ©) Atrial myxoma Question Explanation: Syncope with exercise is a manifestation of organic heart disease in which cardiac outputis fixed and does not rise (or even fall) with exertion Syncope, commonly on exertion, is reported in up to 42% of patients with severe aortic stenosis. Vasovagal syncope is associated with unpleasant stimuli or physiologic conditions, including sights, souns, and smells, sudden pain, sustained upright posture, heart, hunger, and acute blood loss. Transient ischemic attacks are not related to exertion, Orthostatic hypotension is associated with changing from a siting or lying position to an uptight postion, Atrial myxoma is associated with syncope related to changes in position, such as bending, changng from sitting to ying, or turning over in bed. ‘Mark this question & => Question 11 of 30 A76 year old otherwise healthy woman states that she has passed out three in the last month while wallcing brislely during her daily walle with the local senior citizens mall walkers club. The most llcely etiology ofher syndrome according to the history is a) Vasovagal syncope ) Transient ischemic attacke Y © c) Aottic stenosis ) Orthostatis hypotension e) Atrial myzoma Question Explanation: Syncope with exercise is a manifestation of organic heart disease in which cardiac outputis fixed and does not rise (or even fall) with exertion Syncope, commonly on exertion, is reported in up to 42% of patients with severe aortic stenosis. Vasovagal syncope is associated with unpleasant stimuli or physiologic conditions, including sights, souns, and smells, sudden pain, sustained upright posture, heart, hunger, and acute blood loss. Transient ischemic attacks are not related to exertion, Orthostatic hypotension is associated with changing from a siting or lying position to an uptight postion, Atrial myxoma is associated with syncope related to changes in position, such as bending, changng from sitting to ying, or turning over in bed. 2/24/2014 6:32:51 AM ‘Matte this question & => Question 12 of 30 Shortly after having a subclavian venous catheter inserted, a patient is noted to be acttely short of breath to the point where he is breathing heavily and rapidly, although he is not yet in respiratory distress. A chest X-ray reveals a pneumothorax. Management should consist of a) Removal of the central line +b) Insertion of a new central ine on the opposite side ©) Insertion of an angiocath in the third intercostal space on the side of the central line 4) Insertion of a chest tube on the side ofthe central line ©) Insertion of a chest tube cn the side opporite the central line. Answer | Explanation Other User's Explanation Report An Error Question Explanation: ‘A pacumothoraxis a well-documented complication of subclavian central line placement. Usually the line is in proper position, but the pleura were violated by the guide needle. Thus, there is no need to remove the line. You would never want to put a line in the other side, as that could result in bilateral pneumothoracis. Treatment consists of chest tube insertion to re-inflate the lang. There is no reason to place a chest tube on the opposite side. An angiocath is used emergently in the treatment of a tension pneumothorax, which rarely occurs following central line placement and usually presents with a shit on X-ray and possibly with distended neck vens. 2/24/2014 6:32:51 AM ‘Matte this question & => Question 12 of 30 Shortly after having a subclavian venous catheter inserted, a patient is noted to be acutely short of breath to the point where he is breathing heavily and rapidly, although he is not yet in respitatory distress, A chest K- ray reveals a pneumothorax. Management should consist of a) Removal of the central line b) Insertion of a new central line on the opposite side. c) Insertion of an angiocath in the third intercostal space cn the side of the ceatral line. Y © dj Insertion of a chest tube on the side of the central line. e) Insertion of a chest tube on the side opposite the central line. Answer | Explanation Other User's Explanation Report An Error Question Explanation: ‘A pacumothoraxis a well-documented complication of subclavian central line placement. Usually the line is in proper position, but the pleura were violated by the guide needle. Thus, there is no need to remove the line. You would never want to put a line in the other side, as that could result in bilateral pneumothoracis. Treatment consists of chest tube insertion to re-inflate the lang. There is no reason to place a chest tube on the opposite side. An angiocath is used emergently in the treatment of a tension pneumothorax, which rarely occurs following central line placement and usually presents with a shit on X-ray and possibly with distended neck vens. 2/24/2014 6:33:05 AM ‘Mark this question & => Question 13 of 30 4.38 year old woman with acute endocarditis remains febrile for 10 days. The most lily micro-organism involved in this patient is which of the following? a) Streptococcus viridans ) Enterococcus spp. «) Staphylococcus aureus 4) Staphylococcus epidemnidis ) None of the above. Ancwer (PERERA) oshereare Exton Question Explanation: ‘Typically, fever associated with bacterial endocarditis will ast approximately four days when the etiologic agent s Streptococcus viridens (A), Enterococens (B), or Stapayloceceus epidermidis (.e., coagulase-negatwve staphylococcus) (D). However, f the causative agent is Staphylococeus aureus or Pseudomonas aeruginosa, then fever will remain for up to two weeks Report An Error 2/24/2014 6:33:05 AM ‘Mark this question & => Question 13 of 30 4.38 year old woman with acute endocarditis remains febrile for 10 days. The most lily micro-organism involved in this patient is which of the following? a) Streptococcus viridans ) Enterococcus spp. JY © ©) Staphylococcus aureus 4) Staphylococcus epidemnidis ) None of the above. Ancwer (PERERA) oshereare Exton Question Explanation: ‘Typically, fever associated with bacterial endocarditis will ast approximately four days when the etiologic agent s Streptococcus viridens (A), Enterococens (B), or Stapayloceceus epidermidis (.e., coagulase-negatwve staphylococcus) (D). However, f the causative agent is Staphylococeus aureus or Pseudomonas aeruginosa, then fever will remain for up to two weeks Report An Error ‘Mark this question e& => Question Id Question 14 of 30 ‘Which EKG feature will be abnormal in hyperkalemia (serum K+ = 6.0 meqiL.)? a) P-wave b) PR interval c) ORS complex 4) ST segment ¢) T-wave Answer | Beanation Other User's Explanation Report An Error Question Explanation: Frrat-degres AV blocke is defined as a PR interval (B) greater than 0.20 seconds. Right atrial enlargement causes an increase voltage ofthe P-wave (A) on the ECG. Myocardial ischemia classically causes ST-segment (D) depression, Pericardial effusion will increase the distance between the source of electical activity (myocardium) and the voltage sensor (ECG leads), decreasing the amount of voltage detected and therefore causing a low-voltage QRS complex (C). Eatly, mild hyper-Kalemia causes peaked T-waves (E), which can progress to PR segment prolongation and QRS widening, ‘Mark this question e& => Question Id Question 14 of 30 ‘Which EKG feature will be abnormal in hyperkalemia (serum K+ = 6.0 meqiL.)? a) P-wave b) PR interval c) ORS complex 4) ST segment Y © 6) T-wave Answer | Beanation Other User's Explanation Report An Error Question Explanation: Frrat-degres AV blocke is defined as a PR interval (B) greater than 0.20 seconds. Right atrial enlargement causes an increase voltage ofthe P-wave (A) on the ECG. Myocardial ischemia classically causes ST-segment (D) depression, Pericardial effusion will increase the distance between the source of electical activity (myocardium) and the voltage sensor (ECG leads), decreasing the amount of voltage detected and therefore causing a low-voltage QRS complex (C). Eatly, mild hyper-Kalemia causes peaked T-waves (E), which can progress to PR segment prolongation and QRS widening, ‘Mark this question <=> Question Id : 83738 of 30 Question AT5 year old woman dies suddenly 2 days after complaining of severe substernal chest pain that lasted for several hours. She did aot seek medical attention at the time. Postmortem histologic examination of the cardiac tissue demonstrates coagulative necrosis infiltrated by neutrophils, as shown in the photomicrograph above. No infarcts were found in the brain, kidneys, or spleen The most likely cause of this patient's sudden death is which of the following? Sie a] a) Arrhythmia b) Fibrinous pericarditis c) Maral thrombosis ) Ventricular aneurysm ¢) Ventricular mpture Answer (Explanation | Other User's Explanation Report An Error Question Explanation: ‘This patient presents with cudden death following a suspected myocardial infarction (Ml). During the fist 24 to 48 howre after an MI, the heart is particuleely susceptible to conduction disturbances and myocardial iritability which commcaly result in arrhythmias Ventricular Girillation is the most common cause ofdeath 2 days after an acute myocardial ischemic event. As seen in the figure, the necrotic muscle is infllrated by neutrophils, and ischemic myofibers appea contvacied and hyperecsinophilic on H&E. These features are characteristic of 2- to 3-day-old infarction. Remember that any infarcted organ wil undergo coagulation necrosis. Fibrinous pericarditis (choice B) is a frequent manifestation of a transmural infarct. The necrctic muscle incites an acute indlammatory reaction that involves the visceral pericardium and results mn fibrin deposition. It presents with fever, chest pain and a pericardial ficton nun, Itis aot thought of as life-threatening unless severe pericardial effusion develops and does not result in sudden cardiac death. Mural thrombosis (choice C) commonly develops on the endothelial surface of infarcted myocardium. This may lead to thromboembolism when fragments of thrombi detach and pass into the systemic circulation, Bram infarcts due to this mechanism may be fatal Ventricular aneurysm (choice D)is a late complication, developing weeks after infarction when the necrotic myocardium is replaced by scat. Fibrous tissue may bulge out, forming an aneurysmal dilatation that may predispose to rural thrombosis, Ventricular rupture (choice E) most commonly occurs between days 5 and 8 post-infarction, during the pase of granulation tissue formation, Granulation tissue is highly fiable and may yield to the high intraventricular pressure, causing fatal rupture of the ventricular wall ‘Mark this question <=> Question Id : 83738 of 30 Question AT5 year old woman dies suddenly 2 days after complaining of severe substernal chest pain that lasted for several hours. She did aot seek medical attention at the time. Postmortem histologic examination of the cardiac tissue demonstrates coagulative necrosis infiltrated by neutrophils, as shown in the photomicrograph above. No infarcts were found in the brain, kidneys, or spleen The most likely cause of this patient's sudden death is which of the following? Sie a] Y © a) Arrhythmia b) Fibrinous pericarditis c) Maral thrombosis 4) Ventricular aneurysm ©) Ventricular rupture Answer (Explanation | Other User's Explanation Report An Error Question Explanation: ‘This patient presents with cudden death following a suspected myocardial infarction (Ml). During the fist 24 to 48 howre after an MI, the heart is particuleely susceptible to conduction disturbances and myocardial iritability which commcaly result in arrhythmias Ventricular Girillation is the most common cause ofdeath 2 days after an acute myocardial ischemic event. As seen in the figure, the necrotic muscle is infllrated by neutrophils, and ischemic myofibers appea contvacied and hyperecsinophilic on H&E. These features are characteristic of 2- to 3-day-old infarction. Remember that any infarcted organ wil undergo coagulation necrosis. Fibrinous pericarditis (choice B) is a frequent manifestation of a transmural infarct. The necrctic muscle incites an acute indlammatory reaction that involves the visceral pericardium and results mn fibrin deposition. It presents with fever, chest pain and a pericardial ficton nun, Itis aot thought of as life-threatening unless severe pericardial effusion develops and does not result in sudden cardiac death. Mural thrombosis (choice C) commonly develops on the endothelial surface of infarcted myocardium. This may lead to thromboembolism when fragments of thrombi detach and pass into the systemic circulation, Bram infarcts due to this mechanism may be fatal Ventricular aneurysm (choice D)is a late complication, developing weeks after infarction when the necrotic myocardium is replaced by scat. Fibrous tissue may bulge out, forming an aneurysmal dilatation that may predispose to rural thrombosis, Ventricular rupture (choice E) most commonly occurs between days 5 and 8 post-infarction, during the pase of granulation tissue formation, Granulation tissue is highly fiable and may yield to the high intraventricular pressure, causing fatal rupture of the ventricular wall ‘Marke this question & => Question Td : 88782 Question 16 of 30 Antopsy of a 72 year old female who died a sudden death reveals a solitary pecunculated mass within the left atrial chamber Microscopically, the cells are stellate mesenchymal cells admixed with inflammatory and endothelial cells. The most likely diagnosis is a) Papillary fibroelastorna b) Nonbacteridl thrombotic endocarcitis c) Infective endocarditis ) Cardiac myzoma €) Acute mural thromous Anewor [UEQNGNRAN) Othercors Explanation Report An Eros Question Explanation: Cardiac myxoma is the most frequent primary cardiac neoplasm, more common in females. Itis benign and consists of stellate mesenchymal cells within a myxoid background admixed with inflammatory and endothelial cells. Since the left atrium is the most frequent location, this tumor can produce mitral stenosis by a ball valve effect Sudden death occurs in 15% of those afflicted. Death is typically caused by coronary ot systemic embolization or by obstruction of blood flow at the mitral or tricuspid valve An acute ‘mural thrombus would aothave a core of fibrous tissue Acute roural thrombesis usually develops as a result of stasis in the ventricular cavities. in association with ventricular enlargement, myocardial infarction, or ventricular aneurysm, for example. Thrombosis often develops in the atsia when there is atrial fibrillation. Both forms of endocarditis are associated with formation of vegetations attached to the surface of the atrioventricular valves, Vegetations of infective endocarditis are bulky and composed of Abbrin, bacteria, and inflammatory cells. Since norbacterial thrombotic endocarditis is caused by hypercoagulable states, the vegetation: consist of aggregates of fibrin but few inflammatory cells and no bacteria. The lesion of papillary Sbroelastomais not neoplastic, despite the sound of the name. Tt probably results from organized thrombi forming on the endocardial sufaces of the mitral valve or left ventricular cavity. Tt appears as a papillary lesion, most commonly in the left ventricle, with finger like projections attached to the mitral valve without associated valvular or cardiac lesions. Histologically, each pepillary structare appears as a core of, fibrin material lined by endothelium, Papillary Sbroelastoma ate usuelly clinically silent and are discovered at autopsy as an incidental finding Note that all ofthe above conditions may lead to systemic embolization. Fragments of vegetations, thrombi, myxoma, and papillary fibroelastoma may detach and be released into the bloodstream, causing infarcts. ‘Marke this question & => Question Td : 88782 Question 16 of 30 Autopsy af'a 72 year old female who died a sudden death reveals a solitary peduncalated macs within the lef arial chamber Microscopicaly, the cells are stellate mecenchymal cells admized with inflemmatory and endothelial cell. The most llcely diagnosic ie a) Papillary Bbroelastorna b) Nonbacteria thrombctic endocarditis c) Infective endocarditis ¥ © 4) Cardiac myxoma ©) Acute mural thrombus Anewor [UEQNGNRAN) Othercors Explanation Report An Eros Question Explanation: Cardiac myxoma is the most frequent primary cardiac neoplasm, more common in females. Itis benign and consists of stellate mesenchymal cells within a myxoid background admixed with inflammatory and endothelial cells. Since the left atrium is the most frequent location, this tumor can produce mitral stenosis by a ball valve effect Sudden death occurs in 15% of those afflicted. Death is typically caused by coronary ot systemic embolization or by obstruction of blood flow at the mitral or tricuspid valve An acute ‘mural thrombus would aothave a core of fibrous tissue Acute roural thrombesis usually develops as a result of stasis in the ventricular cavities. in association with ventricular enlargement, myocardial infarction, or ventricular aneurysm, for example. Thrombosis often develops in the atsia when there is atrial fibrillation. Both forms of endocarditis are associated with formation of vegetations attached to the surface of the atrioventricular valves, Vegetations of infective endocarditis are bulky and composed of Abbrin, bacteria, and inflammatory cells. Since norbacterial thrombotic endocarditis is caused by hypercoagulable states, the vegetation: consist of aggregates of fibrin but few inflammatory cells and no bacteria. The lesion of papillary Sbroelastomais not neoplastic, despite the sound of the name. Tt probably results from organized thrombi forming on the endocardial sufaces of the mitral valve or left ventricular cavity. Tt appears as a papillary lesion, most commonly in the left ventricle, with finger like projections attached to the mitral valve without associated valvular or cardiac lesions. Histologically, each pepillary structare appears as a core of, fibrin material lined by endothelium, Papillary Sbroelastoma ate usuelly clinically silent and are discovered at autopsy as an incidental finding Note that all ofthe above conditions may lead to systemic embolization. Fragments of vegetations, thrombi, myxoma, and papillary fibroelastoma may detach and be released into the bloodstream, causing infarcts. 2242014 AM “Mark this question & => Question 17 of 30 A 26 year old TV drug abuser begins to feel febrile. Blood cultures arc drawn end an echocardiogram demonstrates characteristic tricuspid valve vegetations indicative of bacterial endocarditis. Entsrosoccal bacteremia is demonstratzd, Which statement regarding this patient is FALSE? a) Murmurs are likely absent in this patient b) Chest X-ray demonstrates diffuse infiltrates ©) Chest X-ray demonstrates mnitinle abscesses 9) Patient is likely to have underlying cardiac disease. €) None of the above Anewor ERRNRANY) otters Explanation Report AnEnor Question Explanation: Although most patients (90%) will exhibit cardice murmurs with bactenal endocardii, itis seen rarely in patients with right-sided infections (.e., via intravenous drug abuse) (A). Chest X-rays of intravenous deug abusers typically demonstrate diffe infiltrates (B) ‘with or without multiple abscesses. However, patients with right sided bacterial endocarditis do not usualy have urdesiying cardiac disease, athough patients with left sided bacterial endocarditis do. 2242014 AM “Mark this question & => Question 17 of 30 6.26 year old TV drug abuser begins to fedl febrile. Blood cultures are draven and an echocardiogram demonstrates characteristic tricuspid valve vegetations indicative of bacterial endocarditis. Enterococcal bacteremia is demonstrated, Which statement regarding this patient is FALSE? a) Murmurs are likely absent in this patient. ) Chest X-ray demonstrates clffuse infiltrates ©) Chest X-ray demonstrates railtiple abscesses YW © o) Patient is likely to have underlying cardiac disease. ¢) None of the above Anewor ERRNRANY) otters Explanation Report AnEnor Question Explanation: Although most patients (90%) will exhibit cardice murmurs with bactenal endocardii, itis seen rarely in patients with right-sided infections (.e., via intravenous drug abuse) (A). Chest X-rays of intravenous deug abusers typically demonstrate diffe infiltrates (B) ‘with or without multiple abscesses. However, patients with right sided bacterial endocarditis do not usualy have urdesiying cardiac disease, athough patients with left sided bacterial endocarditis do. ‘Male this question & => (Question Id : 94332 Question 18 of 30 Amide aged female is admitted unconscious to the Emergency Department following a head injury in a road trafic accident. There is bmuising over the upper abdomen Blood pressure is 80mmElg systolic, pulse 120/iinte. The most imsportent intial step in the management of this patient is a) X-ray (three views) of abdomen. b) Immediate laparotomy c) Perform emergency burtholes 4) Draw blood sample and start IV Buids e) Diagnostic pertoneal lavage Question Explanation: “Almost all citculatory shock states require large volume TV uid replacement, as does severe intravascular volume depletion (eg, from diarrhea or heat stroke). Intravascular volume deficiency is acutely compensated by vasoconstriction, followed over hours by ‘migration of uid from the extravascular compartment to the intravascular, maintaining circulating volume at the expense of total body water. ‘Male this question & => (Question Id : 94332 Question 18 of 30 A-midele aged fernale is admitted unconscious to the Emergency Department following a head injury in a road trafic accident, There is bmising over the upper abdomen. Blood preseure is 80mmHg systolic, pulse 120/minste. The most imp ortent initial step in the management of tis patient is a) X-ray (three views) of abdomen. b) Immediate laparotomy: c) Perform emergency burtholes Y © d)Draw blood sample and start IV fluids e) Diagnostic peritoneal lavage Question Explanation: “Almost all citculatory shock states require large volume TV uid replacement, as does severe intravascular volume depletion (eg, from diarrhea or heat stroke). Intravascular volume deficiency is acutely compensated by vasoconstriction, followed over hours by ‘migration of uid from the extravascular compartment to the intravascular, maintaining circulating volume at the expense of total body water. 2/24/2014 6:34:51 AM ‘Mark this question & => Question Td : 97394 Question 19 of 30 A.22 year old basketball player has a syncopel cpisode during a game. Examination demonstrates marked left ventricular hypertrophy, brisk carotid upsiroke with pulsus bisferiens, and a harsh sysiolic murmur along the left sternal border which increases during the Valsalva mansuver. ECG revealed systolic motion of the mitral valve. The most likely diagnosis is a) Myocardiis b) Consttictive pericarditis ©) Hypertrophic obstructive cardiomyopathy 9) Mitral valve prolapse @) Endocardtis Question Explanation: This young patient has hypertrophic obstructive carciomyopathy. Asyrmetric septal wall hypertrophy with stiff ventricles are present, as well as a functional aortic obstruction which causes the harsh murmur and which changes with valsalva. Arrhythmias are common and can be fatal. Myocarditis would present with elevation of CPK levels and chest pains. It is most likely caused by coxsackievirus B and can be asymptomatic, only leading to congestive heart failure in the future. Constrictive pericarditis would produce diminished heart sounds on auscultation and fiction rubs. Mitral valve prolapse is very common and can be found in five percent of the population. On ausculation, a midsystolic click is often heard. Endocarditis would present with fever and heart murmur. Tt does not usually present as syncope. Report An Error 2/24/2014 6:34:51 AM ‘Mark this question & => Question Td : 97394 Question 19 of 30 4.22 year old basketball player has a syncopal episode during a game. Examination demonstrates marked left ventricular hypertrophy, brisk carotid upstroke with pulsus bisferiens, and a harsh systolic murmur along the left sternal border which increases during the Valsalva mancuver. ECG revealed systolic motion of the mitral valve. The most likely diagnosis is a) Myocardeis ) Conshictive pericarditis Y © 2) Hypertrophic obstractive cardiomyopathy d) Mitral valve prolapse 2) Endocarditis Question Explanation: This young patient has hypertrophic obstructive carciomyopathy. Asyrmetric septal wall hypertrophy with stiff ventricles are present, as well as a functional aortic obstruction which causes the harsh murmur and which changes with valsalva. Arrhythmias are common and can be fatal. Myocarditis would present with elevation of CPK levels and chest pains. It is most likely caused by coxsackievirus B and can be asymptomatic, only leading to congestive heart failure in the future. Constrictive pericarditis would produce diminished heart sounds on auscultation and fiction rubs. Mitral valve prolapse is very common and can be found in five percent of the population. On ausculation, a midsystolic click is often heard. Endocarditis would present with fever and heart murmur. Tt does not usually present as syncope. Report An Error 2/24/2014 AM ‘Matte this question q => Question Td: 100203 Question 20 of 30 Hypertrophic cardiomyopathy is associated with which of the following pulse? a) Pulsus parvus et tardus ) Pulsus paradoans c) Puleus alternane 4) Pulsus bisferiens ¢) None of the above Ancwor EIUNAR) otertisars Explanation Repo An Era Question Explanation Pulsus paradcsus is a decrease in the ampitude of the arterial pulse by greater than 10 mm Hg curing inspiration, and is associated with severe myocardial depression (¢.g, cardiac tamponade), Pulsus bisfetiens is a pulse with two systolic peaks and occars with hypertrophic cardiomyopathy and other conditions that produce a large forward-stroke volume ike aortic insufficiency. Aortic stenosis causes the pulse waveform of parmus et tardus ("slow and late’) 2/24/2014 AM ‘Matte this question q => Question Td: 100203 Question 20 of 30 Hypertrophic cardiomyopathy is associated with which of the following pulse? a) Pulsus parvus et tardus ) Pulsus paradoans c) Puleus alternane ¢ © d)Pulsus bisferiens ¢) None of the above Ancwor EIUNAR) otertisars Explanation Repo An Era Question Explanation Pulsus paradcsus is a decrease in the ampitude of the arterial pulse by greater than 10 mm Hg curing inspiration, and is associated with severe myocardial depression (¢.g, cardiac tamponade), Pulsus bisfetiens is a pulse with two systolic peaks and occars with hypertrophic cardiomyopathy and other conditions that produce a large forward-stroke volume ike aortic insufficiency. Aortic stenosis causes the pulse waveform of parmus et tardus ("slow and late’) 2/24/2014 6:35:19 AM ‘Mark this question e& => 104052, Question 21 of 30 A.65 year old man with a 17 pack year smoking history presents to the emergency room complaining of shortness of breath and dyspnea on exertion. He has tachypnea, tachycardia, and mild cyanosis The BCG reveals an irregularly irregular rhythm with a rate of 112. There are discemible ?-waves in lead I, which show three different morphologies. The QRS complex is 0.08 sec. What type of rythm is suggested by the EKG? a) Atrial futter b) Atal fibrillation «) Mutifocal atrial tachycardia &) Ventriculer fibrilation 2) Ventricular tachycardia Question Explanation: ‘Multifocal atrial tachycardia (MAT) is an itregulasly irregular thytirn with heart rate =100 and demonswating three different P wave morphologies. Iris commonly found in patients with chronic obstructive pulmonary disease and congestive heart failure. Atrial fuer is usvally regular and has one P wave morphology (sawtooth), Atrial fibrillation does not usually show well defined P waves, ‘Ventricular tachycardia and fibrillation are wide complex tachycardias (QRS> 12 sec). 2/24/2014 6:35:19 AM ‘Mark this question e& => 104052, Question 21 of 30 A.65 year old man with a 17 pack year smoking history presents to the emergency room complaining of shortness of breath and dyspnea on exertion. He has tachypnea, tachycardia, and mild cyanosis The BCG reveals an irregularly irregular rhythm with a rate of 112. There are discemible ?-waves in lead I, which show three different morphologies. The QRS complex is 0.08 sec. What type of rythm is suggested by the EKG? a) Atrial futter b) Atal fibrillation Y © ©) Muttifocal atrial tachycardia @) Ventriculer fibrillation 2) Ventricular tachycardia Question Explanation: ‘Multifocal atrial tachycardia (MAT) is an itregulasly irregular thytirn with heart rate =100 and demonswating three different P wave morphologies. Iris commonly found in patients with chronic obstructive pulmonary disease and congestive heart failure. Atrial fuer is usvally regular and has one P wave morphology (sawtooth), Atrial fibrillation does not usually show well defined P waves, ‘Ventricular tachycardia and fibrillation are wide complex tachycardias (QRS> 12 sec). ‘Matte this question ez Question Id: 106372 Question 22 of 30 A.44 year old sinoker visits his physician for aroutine examination, His serum cholesterol level was 350 mgjal and blood pressure ‘was 160/96 mu, He leads a sedentary lifestyle. He is at an increased risk of having a coronary event and that he must decrease some of his risk factors. Tre statement regarding hypercholesterolertia is a) High HDL levels (greater than 60 mg/dl) are a risk factor for heart disease. b) Tuberous xanthomas are painful lipid deposits in patients with hiahly elevated cholesterol levels c) The National Cholesterol Education Progra recorumends dietary modification iftwo consecutive LDL chelesteral levele are 160 mag/dl or more. 4) target LDL cholesterol levels are not achieved by three months, drug treatment with EMG CoA reductase inhibitors can be initiated. ) Shudies have not shown that regression of coronary artery disease occurred with chelesteral lowering "stain" drugs Anowor ERPAREEER) oer cre Explanation Ropert Aner Question Explanation Dietary measures are also recommended iftwo LDL determinations are 130 to 159 mg/dl when the patient has two or more risk factors for coronary heart disease. Low HDL levels (less than 35 mgldl) are a tisk factor for heart disease. Tuberous xanthomas are painless yellowish papules that are lipid deposés and occur in patients wih very high cholesterol levels. Adjuvant drug therapy with cholesterol lowering drugs should be initiated in six months if dietary measures have failed to lower the LDL levels. Studies have shown that starting a patient on HMG CoA reductase inhibitors, such as lovastatin and pravastatin, do produce regression of carotid ais Seana Stas canine ‘Matte this question ez Question Id: 106372 Question 22 of 30 A.44 year old sinoker visits his physician for aroutine examination, His serum cholesterol level was 350 mgjal and blood pressure ‘was 160/96 mu, He leads a sedentary lifestyle. He is at an increased risk of having a coronary event and that he must decrease some of his risk factors. Tre statement regarding hypercholesterolertia is a) High HDL levels (greater than 60 mg/dl) are a risk factor for heart disease. b) Tuberous xanthomas are painful lipid deposits in patients with hiahly elevated cholesterol levels Y © 6) The Natienal Cholesterol Education Program reconmends dietary modification iftwo consecutive LDL chelesteral levele are 160 mag/dl or more. 4) target LDL cholesterol levels are not achieved by three months, drug treatment with EMG CoA reductase inhibitors can be initiated. ) Shudies have not shown that regression of coronary artery disease occurred with chelesteral lowering "stain" drugs Anowor ERPAREEER) oer cre Explanation Ropert Aner Question Explanation Dietary measures are also recommended iftwo LDL determinations are 130 to 159 mg/dl when the patient has two or more risk factors for coronary heart disease. Low HDL levels (less than 35 mgldl) are a tisk factor for heart disease. Tuberous xanthomas are painless yellowish papules that are lipid deposés and occur in patients wih very high cholesterol levels. Adjuvant drug therapy with cholesterol lowering drugs should be initiated in six months if dietary measures have failed to lower the LDL levels. Studies have shown that starting a patient on HMG CoA reductase inhibitors, such as lovastatin and pravastatin, do produce regression of carotid ais Seana Stas canine 2/24/2014 63 AM ‘Maric this question e& => Question Td : 109571 Question 23 of 30 ‘Tae foods that are associated with acute worsening of hypestension in susceplible individuals include which of the following? a) Salted pretzels, ) Apples ©) Lemonade ) Steak ©) Exgs Anewor (NENA) other User's Explanation Report An Eve Question Explanation: Ingestion of salt can acutely raise the blood pressure. Apples and lemonade do not affect the blood pressure, In the long term, steake and eggs can worsen atherosclerosis and thus hypertension, but this would not occur acutely. 2/24/2014 63 AM ‘Maric this question e& => Question Td : 109571 Question 23 of 30 ‘The foods that are associated with acute worsening of hypertension in susceptible mdividuals include which of the following? Y © a) Salted pretzels ) Apples ©) Lemonade ) Steak ©) Exgs Anewor (NENA) other User's Explanation Report An Eve (Question Explanation: Ingestion of salt can acutely raise the blood pressure. Apples and lemonade do not affect the blood pressure, In the long term, steake and eggs can worsen atherosclerosis and thus hypertension, but this would not occur acutely. 2/24/2014 6:36:11 AM ‘Mark this question & => Question 24 of 30 4.6 year old male presents to the emergency room with substemal chest pain radiating to the left shoulder and left arm. He has shortness of breath. ECG reveals 3 mm ST-T wave elevations in the anterior leads. His CPE level is elevated to 300. The treatment option NOT acceptable for this patient is a) Calsium channel blocker. b) ACE inhibitor. ¢) Nitrogiycerin 9) Aspirin. ¢) Beta-blocker Question Explanation: Calcium chanel blockers such as nifedipine, verapamil, and diltiazem are aot the first line of treatment in a myocardial infarction ‘New studies have shown that: ACE inhibitors such as captopril and enalapril, gven within 24 hours of an MI, can preserve ventricular function and prevent remodeling of the emyocardium. Nitroglycerin is given initally to decrease ventricular preload and also to cause vasodilatation of the coronary arteries. Aspinn decreases platelet aggregetion and should be initiated as soon as possible, Beta blockers, such as metoprolol and atenolol, can decrease the heart rate and decrease the sympathetic catacholamnes which cause tachycardia and arrhythmias. Report An Error 2/24/2014 6:36:11 AM ‘Mark this question & => Question 24 of 30 A 66 year old male presents to the emergeacy room with substemal chest paia radiating te the left shoulder and left arma, He has shortaess of breath, ECG reveals 3 mm ST-T wave elevations in the anterior leads. His CPK level is elevated to 300, The treatment option NOT acceptable for this patient is Y © a) Calcium channel blocker. ) ACE inhibitor. ¢) Nitrogiycerin ) Aspicin ©) Beta-blocker. Question Explanation: Calcium chanel blockers such as nifedipine, verapamil, and diltiazem are aot the first line of treatment in a myocardial infarction ‘New studies have shown that: ACE inhibitors such as captopril and enalapril, gven within 24 hours of an MI, can preserve ventricular function and prevent remodeling of the emyocardium. Nitroglycerin is given initally to decrease ventricular preload and also to cause vasodilatation of the coronary arteries. Aspinn decreases platelet aggregetion and should be initiated as soon as possible, Beta blockers, such as metoprolol and atenolol, can decrease the heart rate and decrease the sympathetic catacholamnes which cause tachycardia and arrhythmias. Report An Error ‘Mark this question e& => ‘Question Id : 115522 Question 25 of 30 “Which one of the following clinical signs/symotoms is NOT inclided in persons with mitral stenosis? a) Hemoptysis may result fom rupture of pulmonary bronchial venous connections 'b) Systemic emboli are often caused by thrombi in the atrial appendages c) Abdominal ain, secondary to hepatic congestion, results from left ventricular failure ‘Question Id : 115522 Question 25 of 30 ‘Which one of the following clinical sions/symptems is NOT included in persons with mitral stenosis? a) Hemoptysis may result from rupture of pulmonary bronchial venous connections. 'b) Systemic emboli are often caused by thrombi in the atrial appendages. Y © 0) Abdominal pain, secondary to hepatic congestion, resuits from left ventricular failure d) Cough may result from atrial compression on the mainstem bronchus. ¢) Dyspaea may result fom pulmonary hypertension. Question Explanation Hemoptysis resuits from rapture of pulmonary bronchial venous connections that developed due to pulmonary hypertension, Thrombi form in the erlarged atrium, particularly the atrial appendages. Disiodgement of a part or all ofthe thrombns produces systemic emboli, especially to the brain, kidneys, spleen, and extremities Abdominal pain, secondary to hepatic congestion, results from sight ventricular faiure. Cough may develop as the left atrium enlarges and impinges on the mainstem bronchus. Dyspnea most commonly results from pulmonary venous hypertension. Tris often precipitated by exertion, severe anemia, cardiac arshythmias, and fever, even in those persons with mild mitral stenosis. 2/24/2014 6:36:39 AM ‘Marke this question & => Question Id : 119338 Question 26 of 30 ‘The medication thet has been shown te prolong survival in congestive heart failure is 2) Acpiin ) Captopril ©) Verapanil 4) Lasix «) Digoxin Question Explanation: The Survival and Ventricular Enlargement (SAVE) study showed that ACE inhibitors can prolong survival of patients with congestive heart failure. Digoxin has not been shown to prolong survival, but withdrawal of digoxin, once initiated, has been shown to be detrimental. Aspirin, verapamil, and lasix have not been showa to prolong survival in congestive heart failure 2/24/2014 6:36:39 AM ‘Marke this question & => Question Id : 119338 Question 26 of 30 ‘The medication thet has been shown te prolong survival in congestive heart failure is 2) Acpiin ¥ © b) Captopril ©) Verapanil 4) Lasix «) Digoxin Question Explanation: The Survival and Ventricular Enlargement (SAVE) study showed that ACE inhibitors can prolong survival of patients with congestive heart failure. Digoxin has not been shown to prolong survival, but withdrawal of digoxin, once initiated, has been shown to be detrimental. Aspirin, verapamil, and lasix have not been showa to prolong survival in congestive heart failure 2/24/2014 6:36:57 AM ‘Mark this question & => ‘Question Id : 166649 Question 27 of 30 A.26 year old smoker has a chronic cough He also gets ficquent headaches and aches in his legs whea he exercises. CAR shows irregularities and scalloping on the under surface of his ribs. What undiagnosed congenial defect may be responsible for these findings? a) Coarctation of the aorta b) Eisenmenger syndrome ©) Tetralogy of Fallot 4d) Transposition of great vessels €) Ventricular septal defect Question Explanation: ‘Coarctation of the aorta occurs in two patterns. In the infantile type, the stenosis is proximal to the insertion of the ductus arteriosus (preductal), this pattern is associated with Tumer syndrome. In the adut form, the stenosis is distal to the ductus arteriosus (postductal) and is associated with notching of the ribs, seen as imegulatities and scalloping on the under surface ofthe ribs Coarctation of the aorta is secondary to continuous arterial pressure from the aorta, causing collateral circulation through the intercostal arteries. Classic presentation is the presence of hypertension in the upper extremities and weak pulses in the lower extremities. Headache, cold extremities, and lower extremty claudication with exercise are typical ithe patient is symptomatic (many adults with mild distal coarctation may remain asymptomatic for years). Upper extremity hypertension with weak pulses in the lower extremities, and a midsystolic (or continous) murmur over the chest or back, may be the only obvious signs in some patients. Note that the chronic cough is probably related to the man’s smoking and is aot caused by the coarctation. Hisenmenger syndrome is a shift from a left to right shunt to a right to left shunt secondary to pulmonary hypertension, Tetralogy of Fallot and transposition of great arteries cause cyanosis and are usually diagnosed in infancy. Ventricular septal defect might remain undiagnosed unil adulthood, but would not cause notching of the ribs. 2/24/2014 6:36:57 AM ‘Mark this question & => ‘Question Id : 166649 Question 27 of 30 A.26 year old smoker has a chronic cough He also gets frequent headackes and aches in his legs when he exercises. CMR shows inegularities and scalloping on the under surface of bis ribs. Wha! undiagnosed congenital defect may be responsible for these findings? Y¥ © a) Coarciation of the aorta ') Bisenmenger syndrome ©) Tetralogy of Fallot 4) Trenspostion of groat vessels ©) Ventricular septal defect Question Explanation: ‘Coarctation of the aorta occurs in two patterns. In the infantile type, the stenosis is proximal to the insertion of the ductus arteriosus (preductal), this pattern is associated with Tumer syndrome. In the adut form, the stenosis is distal to the ductus arteriosus (postductal) and is associated with notching of the ribs, seen as imegulatities and scalloping on the under surface ofthe ribs Coarctation of the aorta is secondary to continuous arterial pressure from the aorta, causing collateral circulation through the intercostal arteries. Classic presentation is the presence of hypertension in the upper extremities and weak pulses in the lower extremities. Headache, cold extremities, and lower extremty claudication with exercise are typical ithe patient is symptomatic (many adults with mild distal coarctation may remain asymptomatic for years). Upper extremity hypertension with weak pulses in the lower extremities, and a midsystolic (or continous) murmur over the chest or back, may be the only obvious signs in some patients. Note that the chronic cough is probably related to the man’s smoking and is aot caused by the coarctation. Hisenmenger syndrome is a shift from a left to right shunt to a right to left shunt secondary to pulmonary hypertension, Tetralogy of Fallot and transposition of great arteries cause cyanosis and are usually diagnosed in infancy. Ventricular septal defect might remain undiagnosed unil adulthood, but would not cause notching of the ribs. 2/24/2014 6:37:12 AM ‘Mark this question & => Question Id : 175317 Question 28 of 30 A.54 year old female has a pansystolic rmurmur along the lower left sternal border radiating toward the midclavicular lin. It is medinm pitched, has a blowing quality, and increases slight¥y on inspiration, An S3 is audible along the lower left sternal border. Her JVP is elevated, and a promment v wave is visible. What s the licely etiology ofthe $37 a) Aortic stenosis +) Mitral regurgitation ©) Pulmonic stenosis 4) Tricuspid regurgitation 2) Volume overloaded right ventricle Question Explanatio The orign end radiation of the parsystolic murmur suggest tricuspid valve incompetence. This is further supported by its ritch and quality, and by the fact that it increases on inspiration when cardiac volnme increases, The regurgitant blood low fiom the ventricle during systole increases jugular venous blood pressure and atrial vwave amplitude The origin of the $3 sound, which occurs during erly rapid filling, is the fling of a volume overloaded right ventricle. The right ventricle overload is caused by the combination of systemic venous retum and the retam of the regurgitated blood volume into the right ventricle. Right ventricular failure and dilatation with enlargement of the tricuspid valve orifice, is the most common cause of tricuspid regurgitation and is often secondary to pulmonary hypertension or left ventricular failure, Aortic stenosis causes a harsh, shrill, midsystolic, crescendo-decrescendo murmur and would not necessarily elevate right heart pressures (thus, jugular venous pressure is not elevated). Its often associated with an ‘S4 (late rapid filing) rather than an S3. Mitral regurgitation causes a soft, blowing, pansystolic murmur and is associated with an $3 Tr would elevate left atrial pressures not right atvial (and so jugular venous) pressures. Pulmonic stenosis would also cause a crescendo-decrescendo murmur, aot an S3. Tricuspid regurgitation is the source of the murmur, but not the source of the 3 sound. 2/24/2014 6:37:12 AM ‘Mark this question & => Question Id : 175317 Question 28 of 30 4.54 year old female has a parsystolic murmur along the lower left sternal border radiating toward the midclavicular line. It is mediam pitched, has a blowing quality, and increases slightly on inspiration. An S3 is audible along the lower left sternal border. Her IVP is elevated, and a promment v wave is visible, Whats the Ucely etiology of the $3? a) Aortic stenosis +b) Mitral regurgitation ©) Pulmoric stenosis 4) Tricuspid regurgitation Y © 2) Volume overloaded right ventricle Question Explanatio The orign end radiation of the parsystolic murmur suggest tricuspid valve incompetence. This is further supported by its ritch and quality, and by the fact that it increases on inspiration when cardiac volnme increases, The regurgitant blood low fiom the ventricle during systole increases jugular venous blood pressure and atrial vwave amplitude The origin of the $3 sound, which occurs during erly rapid filling, is the fling of a volume overloaded right ventricle. The right ventricle overload is caused by the combination of systemic venous retum and the retam of the regurgitated blood volume into the right ventricle. Right ventricular failure and dilatation with enlargement of the tricuspid valve orifice, is the most common cause of tricuspid regurgitation and is often secondary to pulmonary hypertension or left ventricular failure, Aortic stenosis causes a harsh, shrill, midsystolic, crescendo-decrescendo murmur and would not necessarily elevate right heart pressures (thus, jugular venous pressure is not elevated). Its often associated with an ‘S4 (late rapid filing) rather than an S3. Mitral regurgitation causes a soft, blowing, pansystolic murmur and is associated with an $3 Tr would elevate left atrial pressures not right atvial (and so jugular venous) pressures. Pulmonic stenosis would also cause a crescendo-decrescendo murmur, aot an S3. Tricuspid regurgitation is the source of the murmur, but not the source of the 3 sound. Mark this question = => Question Td = 199037 Question 29 of 30 A 67-year-old hypertensive, ex-smokcer has had diminished ability to walk and is bed ridden for 2 days. His right leg has become painful and he can’t put any weight on it and his left Ieg also feels like alog He has blood in siool. He has had atrial fibrillation. His pulse is 70'min and irregular. Right leg appears ill and feels cold with sensory loss and no pulses palpable. Doppler exatn is inaudible in right leg. Next best step is a) Amputation of the right leg 1b) Asteriography of the right leg ©) CT scan of the right lower extremity DIV thrombolytic therapy €) Percutaneous mechanical thrombectomy Answer (Expionation |) Other User's Explanation Report An Error Question Explanation: Amnputation of the aifected leg would likely be the best approach to this patient. This patient is presenting with acute limb ischemia. He has paralysis with complete lack of sersation, which indicates an irreversible state of ischemia and a nonviable extremity caused by permanent nerve danage and muscular necrosis. He lacks arterial and venous pulsations. To avoid systemic sepsis, primary amputation would likely be the best approach. Below is a summary of the severity of acute limb ischemia, this classification is used to determine proper management: Viable: no sersory or muscle weakness with audible pulses marginally threatened: minimal sensory loss (just the toes), ao muscle weakness, usually inaudible arterial pulses and audible venous pulses: salvageable iftreated fast. Inmediately threatened. greater sensory loss (further up than the toes), resting pain, mild to moderate muscular weakness, no audible atterial pulses but audible venous pulses; requires immediate revasculatization Tneversible damage’ as deserbed. Arteriography is an smaging modality used to locate the thrombus or embolus that is causing the limb ischemia for the purpose of planing an intervention CT soan of the aifected extremity is incorrect. There ic no need to put this patient through an imaging modality that is not licely te change the management course. Intravenous thrombolytic therapy would be a good modaliy in this patient there were a limb to salvage. I is obvious from the physical examination, however, that the leg is dead and therefore thrombolytic therapy would only increase the patient's risk for hemorrhage. Additionally, this petient has one of the major contraindications to thrombolytic therapy ie. gastrointestinal bleed. Percutaneous mechanical thrombectomy (PMT) devices are newer modalties used in patients who can tolerate thrombolytic therapy. ‘They are used in patients for whom urgent intervention would be required. This patient's leg is not salvageable, however, as explained. Mark this question = => Question Td = 199037 Question 29 of 30 A 67-year-old hypertensive, ex-smokcer has had diminished ability to walk and is bed ridden for 2 days. His right leg has become painful and he can’t put any weight on it and his left Ieg also feels like alog He has blood in siool. He has had atrial fibrillation. His pulse is 70'min and irregular. Right leg appears ill and feels cold with sensory loss and no pulses palpable. Doppler exatn is inaudible in right leg. Next best step is Y © a) Amputation of the right leg 1b) Asteriography of the right leg ©) CT scan of the right lower extremity DIV thrombolytic therapy €) Percutaneous mechanical thrombectomy Answer (Expionation |) Other User's Explanation Report An Error Question Explanation: Amnputation of the aifected leg would likely be the best approach to this patient. This patient is presenting with acute limb ischemia. He has paralysis with complete lack of sersation, which indicates an irreversible state of ischemia and a nonviable extremity caused by permanent nerve danage and muscular necrosis. He lacks arterial and venous pulsations. To avoid systemic sepsis, primary amputation would likely be the best approach. Below is a summary of the severity of acute limb ischemia, this classification is used to determine proper management: Viable: no sersory or muscle weakness with audible pulses marginally threatened: minimal sensory loss (just the toes), ao muscle weakness, usually inaudible arterial pulses and audible venous pulses: salvageable iftreated fast. Inmediately threatened. greater sensory loss (further up than the toes), resting pain, mild to moderate muscular weakness, no audible atterial pulses but audible venous pulses; requires immediate revasculatization Tneversible damage’ as deserbed. Arteriography is an smaging modality used to locate the thrombus or embolus that is causing the limb ischemia for the purpose of planing an intervention CT soan of the aifected extremity is incorrect. There ic no need to put this patient through an imaging modality that is not licely te change the management course. Intravenous thrombolytic therapy would be a good modaliy in this patient there were a limb to salvage. I is obvious from the physical examination, however, that the leg is dead and therefore thrombolytic therapy would only increase the patient's risk for hemorrhage. Additionally, this petient has one of the major contraindications to thrombolytic therapy ie. gastrointestinal bleed. Percutaneous mechanical thrombectomy (PMT) devices are newer modalties used in patients who can tolerate thrombolytic therapy. ‘They are used in patients for whom urgent intervention would be required. This patient's leg is not salvageable, however, as explained. 2242014 6:37:41 AM ‘Mark this question e Question Td : 206757, Question 30 of 30 Characteristic electrocardiogram (ECG) features of hypokalaemia include which one of the following? a) Flattened p waves b) ST depression c) U waves 4) Ventricular tachycardia 2) Wide ORS Answer | Explanation | Other User's Explanation Report An Error Question Explanation: U waves are characteristic features of hypokalaemia, Further ECG features of hypokalaemia include ST elevation and ventricular fibrillation, Ventricular tachycardia is thought to be exceedingly rare in hypokalaemia, The remaining answers are ECG findings typical of hyperkalaemia, together with absent P waves, tall tented T waves, asysiole and ventricular fibrillation. 2242014 6:37:41 AM ‘Mark this question e Question Td : 206757, Question 30 of 30 Characteristic electrocardiogram (ECG) features of hypokalaemia include which one of the following? a) Flattened p waves b) ST depression Y © c) U waves 4) Ventricular tachycardia 2) Wide ORS Answer | Explanation | Other User's Explanation Report An Error Question Explanation: U waves are characteristic features of hypokalaemia, Further ECG features of hypokalaemia include ST elevation and ventricular fibrillation, Ventricular tachycardia is thought to be exceedingly rare in hypokalaemia, The remaining answers are ECG findings typical of hyperkalaemia, together with absent P waves, tall tented T waves, asysiole and ventricular fibrillation. ‘Marie this question => Question Ta: Question 1 of 30 A72 year old man presents to the clinic complaining of steady, dull back pain over the past 3 weeks. His past medical history is significant for diverticulosis, prior smoking, and hypertension. He says that he has run out of his blood pressure medication. There is no history of trauma to his back. On physical exemination his blood pressure is 170/93 mm Hg with a pulse of 88/min, His abdomen is obese but there is a suggestion of a non-tender, pulsetile mass in the epigastric region. The rest of the physical examination is normal. What diagnosis should be considered at this time? a) Abdominal gorte aneurysm (AAA) b) Acute aortic dissection ©) Canda equina syndrome d) Lumbosacral disk herniation. e) Pancreas Anewor [NEIMAN Other sors Explanation Repost An Ero: Question Explanation: It is mperetive to recognize the potential presence of an abdominal aortic aneurysm (AAA). The combination of the history of hypertension and smokeng, the new back pain, and a pulsatile mass on examination is highly suggestive for abdominal aneurysm, The back pain occurs as the expanding mass compresses structures in the retroperitoneum. Its particularly important to make the agnosis because large aneurysms (greater than 5 cm in clameter) are associated with a very high risk of rupture and subsequent ‘mortality. ‘Marie this question => Question Ta: Question 1 of 30 A72 year old men prevents to the cinic complaining of steady, cull back pain over the past 3 weelee. Hic past medical history is significant for divertioulecis, prior smoking, and hypertension, He cays thathe has rm out of his blocd preccure medication. There is no history of trauma to his backe. On physical sxcenination his blood pressure is 170/93 mim Hg with a pulse of £8/imin, His abdomen is obese but there is a suggestion of a non-tender, pulsatile mass in the spigastric region. The rest of the physical examination is normal, What diagnosis should be considered at this time? Y¥ © a) Abdominal orhe aneurysm (AAA) 'b) Acate aortic dissection ©) Canda equina syndrome ) Lumbosacral disk herniation ©) Pancreatitis Anewor [NEIMAN Other sors Explanation Repost An Ero: Question Explanation: It is mperetive to recognize the potential presence of an abdominal aortic aneurysm (AAA). The combination of the history of hypertension and smokeng, the new back pain, and a pulsatile mass on examination is highly suggestive for abdominal aneurysm, The back pain occurs as the expanding mass compresses structures in the retroperitoneum. Its particularly important to make the agnosis because large aneurysms (greater than 5 cm in clameter) are associated with a very high risk of rupture and subsequent ‘mortality. 2/24/2014 6:45:36 AM ‘Mark this question & => Question Td : 31025 Question 2 of 30 A72 year old man has bad increasingly severe, throbbing headaches for several months, centered on the right. There is a palpable tender cord-like area over his right temple. His heart rate is regular with no murmurs, gallops, or rubs. Pulses are equal and fill a all extremities, BP is 110/85 manHg, Histologic examination reveals a muscular artery with lurnenal nacrowing and medial inflammation with lymphocytes, macrophages, and occasional gant cells. He improves with 2 course of high-dose corticosteroid therapy. Which laboratory test finding is most licely to be present with this disease? a) Erythrocyte sedimentation rate of 110 mmfhr b) Anti-double stranded DNA ttre of 1:1024 ©) HDL cholesterol of 0. fmmolL od) pANCA titre of 1.160 e) Rheumatoid factor ttre of 80 [Ui Question Explanation: ‘These are classic findings for temporal arteritis, the most typical involvement with giant cell arteritis. Corticosteroid therapy typically produces areduciion of symptoms, Not treating this condition puts the patient at risk for involvement of other branches of the extemal carotid artery, the worst of which would be the ophthelmic branch. 2/24/2014 6:45:36 AM ‘Mark this question & => Question Td : 31025 Question 2 of 30 4.72 year old man has had increasingly severe, throbbing headaches for several months, centered on the right. There is a palpable tender cord-like area over his right temple. His heart rate is regular with no murmurs, gallops, or rubs. Pulses arc equal and fall in all extremities, BF is 110/85 mmHg. Histologic examination reveals a muscular artery with lumenal narrowing and medial inflammation ‘with lymphocytes, macrophages, and occasional giant cells. He improves with a course of high-dose corticosteroid therapy. Which laboratory test finding is most lcely to be present with this disease? Y © a) Erythrocyte sedimentation rate of 110 mm‘hr +b) Anti-double stranded DNA tire of 11024 c) HDL cholesterol of 0.6mmol/L d) pANCA titre of 1:160 ¢) Rheumatoid factor tite of 80 1UimL Question Explanation: ‘These are classic findings for temporal arteritis, the most typical involvement with giant cell arteritis. Corticosteroid therapy typically produces areduciion of symptoms, Not treating this condition puts the patient at risk for involvement of other branches of the extemal carotid artery, the worst of which would be the ophthelmic branch. 2/24/2014 6:45:51 AM ‘Mare this question & => Question Td : 54289 Question3 of 30 -A.50 year old man with chest pain just suffered a motor velicle accident. Oa examination you find that at the end of expiration his biood pressure is 130/90 nanHg and at the end of inspiration itis 110/92 mmHg, The most likely condition in this patients a) Cardiac tamponade b) Pheumothoraz c) Multiple rib fractures 4) Acute myocardial infarction ©) Acute mupture Answer | Expianation | Other User's Explanation Report An Error Question Explanation: Beck's triad is a collection of three medical signs associated with acute cardiac tamponade, an smergency concition wherein fluid accumulates around the heart and impairs its ability to pump blood. The result is the triad of low arierial blood pressure (hypotension), increased central venous pressure (CVP), and distant (or muffled) heast sounds. ‘This patient also exhibits pulsus paradoms. This is an exaggeration (>12 mm Hg or 9%) of the normal inspiratory decrease in systemic blood pressure. 2/24/2014 6:45:51 AM ‘Mare this question & => Question Td : 54289 Question3 of 30 -A.50 year old man with chest pain just suffered a motor velicle accident. Oa examination you find that at the end of expiration his biood pressure is 130/90 nanHg and at the end of inspiration itis 110/92 mmHg, The most likely condition in this patients Y © a) Cardiac tamponade b) Pneumothorax c) Multiple nb fractures 4) Acute myocardial infarction e) Acute rupture Answer | Expianation | Other User's Explanation Report An Error Question Explanation: Beck's triad is a collection of three medical signs associated with acute cardiac tamponade, an smergency concition wherein fluid accumulates around the heart and impairs its ability to pump blood. The result is the triad of low arierial blood pressure (hypotension), increased central venous pressure (CVP), and distant (or muffled) heast sounds. ‘This patient also exhibits pulsus paradoms. This is an exaggeration (>12 mm Hg or 9%) of the normal inspiratory decrease in systemic blood pressure. 1014 6:46:07 AM ‘Mark this question & => Question 4 of 30 Question Id : 54981 ‘A female presents with headache, lethargy and weight loss. Which of the following make the diagnosis of giant cell arteritis unliely? a) Elevated ESR 'b) Papilledeme without vieval lose ©) Bilateral headache 4d) Non-tender temporal arteries e) The patient is 52 years old Answer | Expionaton | Other User's Explanation Report An Error Question Explanation: Patients are usually elderly with a typical age of 70 but not exclusively so. ‘The temporal arteries are usually tender bat they may be non-tender. Similarly there is usually a unilateral headache but often presents as bilateral headache Exythrocyte sedimentation rate (ESR) is typically elevated but anommal ESR is well recognized, However, papilledema without visual loss would suggest raised intracranial pressure ‘One would expect visual loss with anterior ischemic optic neuropathy in giant cell artestis (GCA). 1014 6:46:07 AM ‘Mark this question & => Question 4 of 30 Question Id : 54981 ‘A female presents with headache, lethargy and weight loss. Which of the following make the diagnosis of giant cell arteritis unliely? a) Elevated ESR SY © b) Papilledema without visual loss ©) Bilateral headache 4d) Non-tender temporal arteries e) The patient is 52 years old Answer | Expionaton | Other User's Explanation Report An Error Question Explanation: Patients are usually elderly with a typical age of 70 but not exclusively so. ‘The temporal arteries are usually tender bat they may be non-tender. Similarly there is usually a unilateral headache but often presents as bilateral headache Exythrocyte sedimentation rate (ESR) is typically elevated but anommal ESR is well recognized, However, papilledema without visual loss would suggest raised intracranial pressure ‘One would expect visual loss with anterior ischemic optic neuropathy in giant cell artestis (GCA). 2/24/2014 6:46:18 AN “Mack this question & => Question 5 of 30 ‘A.44 year old man presents with fever, chills and weakness. On physical exam he is found to have a heart murmur. You also find multiple splinter hemorchages on his ingemals. Whatis the most appropriate diagnostic test to perform? a)EKG b) Urine culture ¢) Blood culture ) Chest X-ray Question Explanation: Tafective endocarditis is infection of the endocardium, usually with bacteria (commonly, streptococci and staphylococci) or fungi Tt produces fever, heart murmurs, petechiae, anemia, embolic phenomena, and endocardial vegetations, Vegetations may result in valvular incompetence or obstruction, myocardial abscess, or mycotic aneurysm, Retinal emboli can cause round or oval hemorshagic retinal lesions with small white centers Roth's spots). Cutaneous manifestations include petechiae (on the upper trunk, conjuactivae, mucous membranes, and distal extremities), painful erythernatous subcutaneous nodules on the tips of digits (Osler’s nodes), nontender hemorrhagic macules on the palms or soles (Janeway lesions), and splinter hemorrhages under the nails. Diagnosis requires demonstration of meroorganistrs in blood and usually echocardiography. ‘Treatment consists of prolonged antimicrobial treatment and sometimes surgery. 2/24/2014 6:46:18 AN “Mack this question & => Question 5 of 30 A.44 year old man presents with fever, chills and weakness. Oa physical exam he is found to have a heart murmar. You also find ‘mutiple splinter hemorrhages on his fingemads. Whats the most appropriate diagnostic test to perform? a EKG 6) Urine culture Y © c)Blood culture 4) Chest X-ray Question Explanation: Tafective endocarditis is infection of the endocardium, usually with bacteria (commonly, streptococci and staphylococci) or fungi Tt produces fever, heart murmurs, petechiae, anemia, embolic phenomena, and endocardial vegetations, Vegetations may result in valvular incompetence or obstruction, myocardial abscess, or mycotic aneurysm, Retinal emboli can cause round or oval hemorshagic retinal lesions with small white centers Roth's spots). Cutaneous manifestations include petechiae (on the upper trunk, conjuactivae, mucous membranes, and distal extremities), painful erythernatous subcutaneous nodules on the tips of digits (Osler’s nodes), nontender hemorrhagic macules on the palms or soles (Janeway lesions), and splinter hemorrhages under the nails. Diagnosis requires demonstration of meroorganistrs in blood and usually echocardiography. ‘Treatment consists of prolonged antimicrobial treatment and sometimes surgery. 2/24/2014 6:46:33 AM ‘Mark this question & => Question 6 of 30 An 81 year old man, who is taking warferin because of a history of atsial Gbuillation, presents to the emergency room with a syncopal episode. His stool has been black. His hemoglobin is $5 g/L and INR 7.2, The best immediste approach to the management of raised INRis a) Vitemnin K, 1 mg orally ) Vitamin HL, 10 mg intravenously ©) Hold warfarin for 3 days J) A units of fresh frozen plasma ¢) Desmopressin DDAVE) Question Explanation: Ifthe INK is between 5.0-$.0 and bleedingis absent, you should cease warfarin therepy: consider reasons for elevated INR and patient-specific factors, If bleeding risk is high, give vitamin K (1.0-2.0 mg orally or 0.5-1.0 mg intravenously) ‘Measure INR within 24 hours; resume warfarin at a reduced dose once INK isin therapeutic range. However, there is any clinically significant bleeding where warfarin induced coagulopathy is considered a contributng factor, you should cease warfarin therapy, give 5.0-10.0 mg vitamin K intravenously, as well as fresh frozen plasma (150-300 mL}, assess patient continously until INR < 5.0, and bleeding stops. 2/24/2014 6:46:33 AM ‘Mark this question & => Question 6 of 30 Aa B1 year old man, who is taking warferin because of abistory of atrial Gbiilation, presents to the emergency room with a syncopal episode, His stool has been black. His hemoglebin is 65 g/L and INR7.2, The best immediate approach to the management of raised INRis a) Vitemin K, 1 mg orally ) Vitemnin H, 10 mg intravenously ©) Hold warfarin for 3 days Y © 4) 4 units of fresh frozen plasma 2) Desmopressin DDAVP) Question Explanation: Ifthe INK is between 5.0-$.0 and bleedingis absent, you should cease warfarin therepy: consider reasons for elevated INR and patient-specific factors, If bleeding risk is high, give vitamin K (1.0-2.0 mg orally or 0.5-1.0 mg intravenously) ‘Measure INR within 24 hours; resume warfarin at a reduced dose once INK isin therapeutic range. However, there is any clinically significant bleeding where warfarin induced coagulopathy is considered a contributng factor, you should cease warfarin therapy, give 5.0-10.0 mg vitamin K intravenously, as well as fresh frozen plasma (150-300 mL}, assess patient continously until INR < 5.0, and bleeding stops. 2/24/2014 66:45 AM ‘Marc this question & => Question Id : 61648 Question 7 of 30 A 60 year old woman who is a regular patient calls the office because she has developed severe muscle weakness, muscle cramps and polyuria, She began treatment 6 weeks ago with 50 mg of chlorthalidons daily for mild to modsraie essential hypertension, The most likely explanation for her symptoms is the development of? a) Hypokalemia ) Diabetes melitus ©) Hypomagnesemia ) Hyponatremia ©) Metabolic acidosis Anewor (NEIMAN) other User's Explanation Report An Enon (Question Explanation: Hypokelemia ss a very common side effect of nonpotasstum sparing curetics (¢.g., chlorthalidone). This is often more pronounced in the clder age group. Patients usually complain of rauscle weakness, fatigue, and cramps. Constipation and ileus characterize the smooth muscle involvement, whereas hyp oreflexia, flaccid paralysis, and tetany are signs of severe hypokalemia 2/24/2014 66:45 AM ‘Marc this question & => Question Id : 61648 Question 7 of 30 A 60 year old woman who is a regular patient calls the office because she has developed severe muscle weakness, muscle cramps and polyuria, She began treatment 6 weeks ago with 50 mg of chlorthalidons daily for mild to modsraie essential hypertension, The most likely explanation for her symptoms is the development of? Y © a) Hypokalemia ') Diabetes melitus c) Hypomagnesemia d) Hyponatremia €) Metabolic acidosis Anewor (NEIMAN) other User's Explanation Report An Enon (Question Explanation: Hypokelemia ss a very common side effect of nonpotasstum sparing curetics (¢.g., chlorthalidone). This is often more pronounced in the clder age group. Patients usually complain of rauscle weakness, fatigue, and cramps. Constipation and ileus characterize the smooth muscle involvement, whereas hyp oreflexia, flaccid paralysis, and tetany are signs of severe hypokalemia 2/24/2014 6:47:02 AM ‘Mark this question = => Question Td : 62740 Question 8 of 30 A.55 year old man is admitied to the hospital froma the emergency department because of an acute myocardial infarction. He has no identified risk factors and no history of heart disease. He had no major complications and is discharged after 8 days. Ar discharge, the diagnostic test that should be scheduled for the 2 week follow up vist in order to predict most accurately his tisk for another infarction is which of the following? a) dlectrocardiogram b) Exercise stress test c) 24 hour ambulatory EK G monitoring 4) Radiomclde ventriculography ©) Titrascund of the heart Answer (Botahaiion | Other User's Explanation Report An Error Question Explanation: The most important test to determine risk for recurrent infarction would be an exercise stress test, A submaximal exercise test can even be done before the patient is discharged from the hospital. An electrocardiogram and a 24 hour ambulatory EKG would demonstrate any predispositions to arrhythmia, but would not yield any information regarding risk ofischemia Radionuclide ventriculography and ultrasonography of the heart can give useful information regarding ventricular contractility and any wall motion abnomnalities, but would not yield any information regarding ischemia risk, 2/24/2014 6:47:02 AM ‘Mark this question = => Question Td : 62740 Question 8 of 30 A.59 year old man is admitted to the hospital from the emergency department because of an acute myocardial infarction. He has no identified risk factors and no history of heart disease. He had no major complications and is discharged after 8 days. Ar discharge, the diagnostic test that should be scheduled for the 2 week follow up vist in order to predict most accurately his tisk for another infarction is which of the following? a) dlectrocardiogram Y © b) Exercise stress test ©) 24 how ambulatory EKG monitoring 4) Radiomclde ventriculography ¢) Ultrasound of the heart Answer (Botahaiion | Other User's Explanation Report An Error Question Explanation: The most important test to determine risk for recurrent infarction would be an exercise stress test, A submaximal exercise test can even be done before the patient is discharged from the hospital. An electrocardiogram and a 24 hour ambulatory EKG would demonstrate any predispositions to arrhythmia, but would not yield any information regarding risk ofischemia Radionuclide ventriculography and ultrasonography of the heart can give useful information regarding ventricular contractility and any wall motion abnomnalities, but would not yield any information regarding ischemia risk, 2/24/2014 67-17 AM ‘Mark this question & => Question Td : 66022 Question 9 of 30 A 56-year-old presents in acute severe circulatory failure, Neck veins are elevated and rales are noted on chest auscultation, This is consistent with a) Cardiogeric shock, b) Hypovolemic shock ) Septic shock. 4) Anaphylactic shock. ©) Shock secondary to severe blood loss. Answer | Exrianaton | Other User's Explanation Report An Error Question Explanation: Elevated neck veins are consistent with cardiogenic shock. Hypovolemic shock and septic shock are associated with collapsed neck veins, Shock secondary to severe blood loss is hypovolemic shock. Anaphylactic shock can have normal neck veins of collapsed, neck vens. Rales are another clue to the pulmonary congestion that is associated with pulmonary edema. 2/24/2014 67-17 AM ‘Mark this question & => Question Td : 66022 Question 9 of 30 A 56-year-old presents in acute severe circulatory failure, Neck veins are elevated and rales are noted on chest auscultation, This is consistent with Y © a) Cardiogeric shock. b) Hypovolemic shock ) Septic shock. 4) Anaphylactic shock. ©) Shock secondary to severe blood loss. Answer | Exrianaton | Other User's Explanation Report An Error Question Explanation: Elevated neck veins are consistent with cardiogenic shock. Hypovolemic shock and septic shock are associated with collapsed neck veins, Shock secondary to severe blood loss is hypovolemic shock. Anaphylactic shock can have normal neck veins of collapsed, neck vens. Rales are another clue to the pulmonary congestion that is associated with pulmonary edema. ‘Mark this question = => Question Td Question 10 of 30 ‘The EKG feature that wil be abnormal in a pesicardial effusion is a) P-wave ) PR interval ©) QRS complex 4) ST segment ©) T-wave (Question Explanation: First-degres AV block is defined as a PR interval (B) greater than 0.20 seconds. Right atrial enlargement causes an increase voltage ofthe P-wave (A) on the ECG. Myocardial ischemia classically causes ST-segment (D) depression, Pericardial effusion will increase the cistance between the source of electrical activity (myocardium) and the voltage sensor (ECG leads), decreasing the amount of voltage detected and therefore causing alow-voltage QRS complex (C). Early, mild hyperkalemia causes peaked T-waves (E), which can progress to PR segmert prolongation and QRS widening, ‘Mark this question = => Question Td Question 10 of 30 The EKG feature that will be abnormal in a pericardial effusion is a) Pewave ') PR interval Y © 0) QRS complex d) ST segment e) T-wave (Question Explanation: First-degres AV block is defined as a PR interval (B) greater than 0.20 seconds. Right atrial enlargement causes an increase voltage ofthe P-wave (A) on the ECG. Myocardial ischemia classically causes ST-segment (D) depression, Pericardial effusion will increase the cistance between the source of electrical activity (myocardium) and the voltage sensor (ECG leads), decreasing the amount of voltage detected and therefore causing alow-voltage QRS complex (C). Early, mild hyperkalemia causes peaked T-waves (E), which can progress to PR segmert prolongation and QRS widening, Mark this question & => Question Ta = 83603 Question 11 of 30 4.47 year old woman presents with complaint of shortness of breath with exercise, Hler past medical history is significant for sheumatoid arthritis for which she takes nonsteroidal anti-inflammatory agents. Physical examination is remarkable for slightly muffled heart sound and fine bilateral sales in her fungs. A chest X-ray reveals cardiomegaly and exidence of mild pulmonary edema; an echocardiogram demonstrates a reduced ejection fraction, Examination ofa hematoxylin & cosin stained section of biopsy matetial fiom heart muscle. The material is most likely composed of which of the fllowing protrins? a) AA amyloid protein b) AE amyloid protein ©) AF amyloid protein 9) AL amyloid protein €) Beta amyloid precursor protein Anowor (ERRERER) o%hor Usor's Exton Question Explanation: ‘The symptoms described in this case are due to amploidosis associcted with theumatoid arthritis. The amyloid protein has deposited inher heart and is causing symptoms of congestive heart failure. The term amyloid describes a variety of fibrilary proteins deposited in different tissues during pathologic circumstances. It stains pink with routine H&E stains, but shows apple-green bireflingence when Congo reé-stained material is viewed with polarized licht. Type AA amyloid protein is an acute phase protein (MW 18,000) that is produced by the liver during inflammatory reactions such as chronic infections and rheumatoid arthritis AE amyloid protein (choice B) is a peptide hormone precursor that is usually associated with meduilary carcinoma of the thyroid and pancreatic islet cell adenomas. The amyloid protein is not systemically distributed but instead is found locally within the neoplasm "AF amyloid protein (choice C) is a pre-albumin protein. Ttis associated with certain familial amyloidosis symdcomes, in which itis distributed within peripheral nerves and kidney. Tis also associated with senile cardiac amyloidosis in the elderly, which affects individuals without any significant, past medical histcry. Deposition is usually within a single organ. AL amyloid (choice D) is a protein composed of immunoglobulin light chains. it ir ascociated with myeloma aad B cell malignant ipmsphomeas. The neoplastis ceils preduse this protein, which deposits in the heart, GI tract, kidney, spleen, and tongue Beta amayloid precursor protein (choice E) also known as Ad peptide, is a 4000D peptide associated with Alsheimer disease and Down syndrome. Ibis derived fom a serum protein encoded on chromosome 21. It appears in neurofibrilary tangles, in plaques, and in the walls of cerebral vessels. n Report An Error Mark this question & => Question Ta = 83603 Question 11 of 30 4.47 year old woman presents with complaint of shortness of breath with exercise. Her past medical history is significant for rheumatoid arthritis for which she takes nonsteroidal anti-inflammatory agents Physical examination is remarkable for slightly mulled heart sound and fine bilateral rales in her lunge. A chest X-ray reveals cardiomegaly and evidence of mild pulmonary edema; an echocardiogram demonstrates a reduced ejection fraction, Examination ofa hematoxylin & eosin stained section of biopsy material from heart muscle. The material is most likcly composed of which of the following protcins? Y © a) AA amyloid proten 'b) AE amyloid protein c) AF amyloid protein d) AL amyloid protein ¢) Beta amyloid precursor protein Anowor (ERRERER) o%hor Usor's Exton Question Explanation: ‘The symptoms described in this case are due to amploidosis associcted with theumatoid arthritis. The amyloid protein has deposited inher heart and is causing symptoms of congestive heart failure. The term amyloid describes a variety of fibrilary proteins deposited in different tissues during pathologic circumstances. It stains pink with routine H&E stains, but shows apple-green bireflingence when Congo reé-stained material is viewed with polarized licht. Type AA amyloid protein is an acute phase protein (MW 18,000) that is produced by the liver during inflammatory reactions such as chronic infections and rheumatoid arthritis AE amyloid protein (choice B) is a peptide hormone precursor that is usually associated with meduilary carcinoma of the thyroid and pancreatic islet cell adenomas. The amyloid protein is not systemically distributed but instead is found locally within the neoplasm "AF amyloid protein (choice C) is a pre-albumin protein. Ttis associated with certain familial amyloidosis symdcomes, in which itis distributed within peripheral nerves and kidney. Tis also associated with senile cardiac amyloidosis in the elderly, which affects individuals without any significant, past medical histcry. Deposition is usually within a single organ. AL amyloid (choice D) is a protein composed of immunoglobulin light chains. it ir ascociated with myeloma aad B cell malignant ipmsphomeas. The neoplastis ceils preduse this protein, which deposits in the heart, GI tract, kidney, spleen, and tongue Beta amayloid precursor protein (choice E) also known as Ad peptide, is a 4000D peptide associated with Alsheimer disease and Down syndrome. Ibis derived fom a serum protein encoded on chromosome 21. It appears in neurofibrilary tangles, in plaques, and in the walls of cerebral vessels. n Report An Error ‘Mack this question e& => Question Ta : 84860 Question 12 of 30 4.50 year old man with along history of cardiovascular disease is found dead in his home, Autopsy reveals hemopericardium secondary to ventricular wall rupture. The man probably had a myocardial infarction approximately how many days before his death? a) 2 days 6) 7 days ©) 12 days 4) 20 days 2) 60 days Question Explanation: Unsuspected (or denied) myocardial infarction is not uncommon, and death may occur because of untreated complications. A umber of serious complications can occur between 3 and 10 days following infarction, due to marked weakening of the necrotic myocardium. These include mpmre of the ventricular wall leading to hemopericardium and cardiac tamponade (es this patient hed), rupture of the interventricular septum, and rupture of the papillary muscle “Arrhythmias are the most common compication 2 days post infarction (choice A) Fibrinous pericarditis secondary to an autoinmune phenomenon (Dressler’s syndrome) can be seen several weeks afer infarctions (choices C and D) By 60 days after infarction (choice E) the contracted scar is usually complete and residval complicatons include left ventricular failure and arrhythmias ‘Mack this question e& => Question Ta : 84860 Question 12 of 30 4.50 year old man with along history of cardiovascular disease is found dead in his home, Autopsy reveals hemopericardium secondary to ventricular wall rupture. The man probably had a myocardial infarction approximately how many days before his death? a) 2 days Y © b)7 days ©) 12 days d) 20 days €) 60 days Question Explanation: Unsuspected (or denied) myocardial infarction is not uncommon, and death may occur because of untreated complications. A umber of serious complications can occur between 3 and 10 days following infarction, due to marked weakening of the necrotic myocardium. These include mpmre of the ventricular wall leading to hemopericardium and cardiac tamponade (es this patient hed), rupture of the interventricular septum, and rupture of the papillary muscle “Arrhythmias are the most common compication 2 days post infarction (choice A) Fibrinous pericarditis secondary to an autoinmune phenomenon (Dressler’s syndrome) can be seen several weeks afer infarctions (choices C and D) By 60 days after infarction (choice E) the contracted scar is usually complete and residval complicatons include left ventricular failure and arrhythmias 2/24/2014 648-15 AM ‘Matte this question <=> Question 13 of 30 Question Id : 89344 ‘A Swan Ganz catheter is inserted in a 64 year old patie for evaluation of a suspected shock. It demonstrates elevated cardiac cutput, decreased pulmonary capillary wedge pressure, and decreased night alvial pressure. This presentation is most consistent with which of the following? 8) Cardiogenic shecke. b) Volume depietion ) Septic shock. ) Right ventricle infarction. ) Pericardial tamponade Question Explanation: ‘These Swan Ganz determinations are consistent with septic shock. All of the other choices are associated with decreased cardiac output. 2/24/2014 648-15 AM ‘Matte this question <=> Question 13 of 30 Question Id : 89344 ‘A Swan Ganz catheter is inserted in a 64 year old patie for evaluation of a suspected shock. It demonstrates elevated cardiac cutput, decreased pulmonary capillary wedge pressure, and decreased night alvial pressure. This presentation is most consistent with which of the following? 8) Cardiogenic shecke. b) Volume depietion Y © 0) Septic shock. ) Right ventricle infarction. ) Pericardial tamponade Question Explanation: ‘These Swan Ganz determinations are consistent with septic shock. All of the other choices are associated with decreased cardiac output. 2/24/2014 6:48:32 AN Matte this question & => Question Ta : 90326 Question 14 of 30 A 68 year old male wales up in the middle of the aight complaining of severe shortness of breath He is brought to the emergency department where he was diagnosed as having congestive heart failure. Which of the folowing test is NOT useful to diagnose systolic dysfunction of the ventricle? a) chest X-rav. bb) Exercise stress testing. ©) Echocardiogram, d) MUGA scans @) Technetium 99 scans Answer | Gxplanation Other User's Explanation Report An Error Question Explanation: Technetium 99 scanning is a muclear medicine test whichis used to define areas of acute myocardial necrosis and infarcts. While it provides important information about structure, it is not a practical way to evaluate funciion dynamically A chest X-ray would reveal the consequences of systolic dysfunction: an enlarged cardiac silhouette, a cardiothoracic ratio greater than 0.5, Kerley’s B lines representing interstiial edema, and a perihiler haze. Exercise stress testing would reveal significant left ventricular dysfunction ifthe patient develops ischemia before six minutes and/or develops hypotension durng exercise. An echocardiogram would reveal a left ventricular ejection fraction less than 50% and segmental wall motion abnomnelities. MUGA scan is a nuclear medicine test which offers rapid and efficient imaging of the left ventricle during rest and exercise and thus determines the ejection traction. It is especially valuable for evaluating lef ventricular systolic performance in the setting of chronic aortic and mitral valvular regurgitation. 2/24/2014 6:48:32 AN Matte this question & => Question Ta : 90326 Question 14 of 30 A 68 year old male wakes up in the middle of the aight complaining of severe shortness of breath He is brought to the emergency department where he was diagnosed as having congestive heart failure. Which of the following test is NOT useful to diagnose systolic dysiunction of the ventricle? a) chest X-ray. 'b) Exercise stress testing, c) Echocardiogram. ) MUGA scans Y © ¢) Technetum 99 scans Answer | Gxplanation Other User's Explanation Report An Error Question Explanation: Technetium 99 scanning is a muclear medicine test whichis used to define areas of acute myocardial necrosis and infarcts. While it provides important information about structure, it is not a practical way to evaluate funciion dynamically A chest X-ray would reveal the consequences of systolic dysfunction: an enlarged cardiac silhouette, a cardiothoracic ratio greater than 0.5, Kerley’s B lines representing interstiial edema, and a perihiler haze. Exercise stress testing would reveal significant left ventricular dysfunction ifthe patient develops ischemia before six minutes and/or develops hypotension durng exercise. An echocardiogram would reveal a left ventricular ejection fraction less than 50% and segmental wall motion abnomnelities. MUGA scan is a nuclear medicine test which offers rapid and efficient imaging of the left ventricle during rest and exercise and thus determines the ejection traction. It is especially valuable for evaluating lef ventricular systolic performance in the setting of chronic aortic and mitral valvular regurgitation. 2/24/2014 6:48:19 AM “Mark this question ez Question 15 of 30 A65 year old black male was brought to the emergency room complaining of severe substemal chest pain associated with nausea, diaphoresis, and shoriness of breath. ECG reveals ST segment elevations in leads Tl, IT, and AVF. The heaut rate was 52 beats per minuts, Cardiac event associated with the above findings is which of the following? a) Inferior myocardial infarction ) Pericardial effusion ©) Antetior myocardial infarction 4) Aortic stenosis €) Dilaed cardiomyopathy Question Explanation: Inferior wall myocardial infarctions usually ase associeted with bradyacrhythmias, nausea, and relatively mild damage to the myocardium compared t anterior wall myocardial infarctions. Q waves are seen in leads TI, TT, and AVF after the damage has been completed. Vagal nerve stirmuletion during the infarction causes nausea and vorniting. Usually the right coronary artery is involved. Patients with a large pericardial effusion have diffuse ST elevations in all lacs if pericarditis is present. However, nausea and vomiting are not typically present. Also, peticardial pain differs from ischemic pain, in thet ischemic pain is not exacerbated by chest movement of lying down. Patients who have an antetior wall myocardial infarction have ST elevations and Q waves in the anterior lateral leads, such as V1, V2, and V3. The thrombus would involve the left anterior coronary artery of the left main artery Aortic stenosis is, associated with a harsh, systolic murmur in the early stages and there is usually no obvious change on the electrocardiogrem, but the echocardiogram may show hypertrophic cardiomyopathy Dilated cardiomyopathy can lead to arrhythmias and decompersaton of congestive heart failure, leading to possible myocardial infarction. 2/24/2014 6:48:19 AM “Mark this question ez Question 15 of 30 A.65 year old blacke male was brought to the emergency room complaining of severe substemal chest pain associated with nausea, diaphoresis, and shoriness of breath. ECG reveals ST segment clevations in leads I, II, and AVE. The heart rate was 52 beats per minute. Carcliac event associated with the above Giadings is which of the follewing? Y © a) Inferior myocardial infarction ) Pericardial effusion ©) Antetior myocardial infarction 4) Aortic stenosis ©) Dilated cardiomyopathy Question Explanation: Inferior wall myocardial infarctions usually ase associeted with bradyacrhythmias, nausea, and relatively mild damage to the myocardium compared t anterior wall myocardial infarctions. Q waves are seen in leads TI, TT, and AVF after the damage has been completed. Vagal nerve stirmuletion during the infarction causes nausea and vorniting. Usually the right coronary artery is involved. Patients with a large pericardial effusion have diffuse ST elevations in all lacs if pericarditis is present. However, nausea and vomiting are not typically present. Also, peticardial pain differs from ischemic pain, in thet ischemic pain is not exacerbated by chest movement of lying down. Patients who have an antetior wall myocardial infarction have ST elevations and Q waves in the anterior lateral leads, such as V1, V2, and V3. The thrombus would involve the left anterior coronary artery of the left main artery Aortic stenosis is, associated with a harsh, systolic murmur in the early stages and there is usually no obvious change on the electrocardiogrem, but the echocardiogram may show hypertrophic cardiomyopathy Dilated cardiomyopathy can lead to arrhythmias and decompersaton of congestive heart failure, leading to possible myocardial infarction. 2/24/2014 6:49:03 AM ‘Mark this question & => Question 16 of 30 A.45 year old female has an elevated blood pressure that was first detected 1 week ago during a routne screening at heath fair. On examination today her blood pressure is 145/95 mmiEig. 1 year later she is asymptomatic and her blood pressure is 140/85 mmHg. She is continuing to take the prescribed antihypertensive medication. The most appropriate management at this time is a) Discontinue the antihypertensive therapy ) Continue the anthypertensive therapy c) Order a chest x-ray film and electrocardiography 4) Order complete blood count, serum electrolyte concentrations and lver chemistry profile ©) Reduce the dosage of the antihypertensive medicatien by 50% Question Explanation: She has forunately remained asymptomatic with a blood pressure that has declined to the normal range. She most likely suffers from essential hypertension and, since this is a lifelong disorder, she will most licely require long-term antihypertensive therapy and therefore discontinuing it and reducing the dose are inappropriate at this time 2/24/2014 6:49:03 AM ‘Mark this question & => Question 16 of 30 A.45 year old female has an elevated blood pressure that was first detected 1 week ago during a routne screening at heath fair. On examination today her blood pressure is 145/95 mmiEig. 1 year later she is asymptomatic and her blood pressure is 140/85 mmHg. She is continuing to take the prescribed antihypertensive medication. The most appropriate management at this time is a) Discontinue the antihypertensive therapy Y © b) Continue the anthypertensive therapy c) Order a chest x-ray film and electrocarciography 4) Order complete blood count, serum electrolyte concentratons and iver chemistry profile ©) Reduee the dosage of the anthypertensive medication by 50% Question Explanation: She has forunately remained asymptomatic with a blood pressure that has declined to the normal range. She most likely suffers from essential hypertension and, since this is a lifelong disorder, she will most licely require long-term antihypertensive therapy and therefore discontinuing it and reducing the dose are inappropriate at this time 2/24/2014 6: 9 AM. ‘Marc this question ez Question Id: 94142 Question 17 of 30 A58 year oldman strikes the steering wheel of his car during alow speed automobile accident. He is alect and his vitel signs are nontnal when examined in the emergency department, He complains of mild tenderness on sternal compression. Chest X-ray filma shows a widened mediastinarn ‘What is the most appropriate fist step? a) Bronchoscopy b) CT scan of the chest with contrast ©) Thoracentesis 9) Thoracic ultrasonography Answer [Exvienaton | Other User's Explanation Report An Error Question Explanation: “A classic history such as this should always trigger a rule out diagnosis for aortic arch disseciion. Traumatic dissection is very common in trauma situations and the classical description is a widened mediastinum on plain chest lms. However, this finding is highly non-specific and therefore, CT imaging with contrast of the thorax is now the preferred method for ruling out this diagnosis, Bronchoscopy is used to assess the wachea and upper airways via direct visualization, Ithas no role in the evaluation of possible aortic injury. Thoracocentesis is used to evaluate fluid present in the pleural space, a so called pleural effusion. A needle is inserted Jeteral or inferior to the lang but within the pleural space to withdraw fluid for analysis. Thoracic ultrasonography could refer 10 imaging of the thorax or of the heart, Regardless, trans-thoracic ultrasound has limted ability to evaluate the aortic arch. A trans- esophageal ultrasound however is exquisitely sensitive for detecting aortic arch injury and allows complete inspection of the aortc rootand valve, as well as the coronary cusps and sinuses 2/24/2014 6: 9 AM. ‘Marc this question ez Question Id: 94142 Question 17 of 30 4.58 year oldman strikes the steering wheel of his car daring alow speed automobile accident. He is alert and his vital signs are normal when examined in the emergency department. He complains of mild tenderness oa sternal compression. Chest X-ray film shows a widened mediastinum. What is the most appropriate first step? a) Bronchoscopy V © b)CT scan of the chest with contrast c) Thoracentesis 9) Thoracic ultrasonography Answer [Exvienaton | Other User's Explanation Report An Error Question Explanation: “A classic history such as this should always trigger a rule out diagnosis for aortic arch disseciion. Traumatic dissection is very common in trauma situations and the classical description is a widened mediastinum on plain chest lms. However, this finding is highly non-specific and therefore, CT imaging with contrast of the thorax is now the preferred method for ruling out this diagnosis, Bronchoscopy is used to assess the wachea and upper airways via direct visualization, Ithas no role in the evaluation of possible aortic injury. Thoracocentesis is used to evaluate fluid present in the pleural space, a so called pleural effusion. A needle is inserted Jeteral or inferior to the lang but within the pleural space to withdraw fluid for analysis. Thoracic ultrasonography could refer 10 imaging of the thorax or of the heart, Regardless, trans-thoracic ultrasound has limted ability to evaluate the aortic arch. A trans- esophageal ultrasound however is exquisitely sensitive for detecting aortic arch injury and allows complete inspection of the aortc rootand valve, as well as the coronary cusps and sinuses 2/24/2014 6:49:39 AM ‘Mark this question & => Question 18 of 30 ‘A tisk factor for perioperative arrhythmia is which of the following? a) Congestive heart failure ) Supraventricular tachycardia ©) Age > 60 4) Premature atrial contractions €) Bast history of hyperthyroidisia Question Explanatioy Significant predictors of mtra operative and penoperative ventricular arrhythmias include preoperative ventricular (hot supraventricular) ectopy, a history of congestive heart failure and a history of cigarette smoking Age and a history of hyperthyrcidism are not signdicant predictors of peni-operative ventricular arrhythmias. 2/24/2014 6:49:39 AM ‘Mark this question & => Question 18 of 30 Avrisk factor for perioperative arrhythmia is which of the following? Y © a) Congestive heart failure ) Supraventricular tachycardia ©) Age > 60 4) Promatace atial contractions «) Past history of hyperthyroidism Question Explanatioy Significant predictors of mtra operative and penoperative ventricular arrhythmias include preoperative ventricular (hot supraventricular) ectopy, a history of congestive heart failure and a history of cigarette smoking Age and a history of hyperthyrcidism are not signdicant predictors of peni-operative ventricular arrhythmias. 2/24/2014 53 AM (Maré this question =z Question 1d: 96821 Question 19 of 30 Cyanotic congenital heart disease, especially in adults, is associated with which ofthe following? a) Erythrocytosis. b) Hemochromatosis ©) Leulkopenia ) Jaundice ¢) Exythroderma Question Explanation: Erythrocytosis is one of the body's attempts to compensate for the chronic hypoxemia of cyanotic congenital heart disease, Hemochromatosis, leukopenia, and jaundice are not typically associated with this form of heart disease. Exythroderma, a skin condition sometimes seen in certain leukemias, also is not related to cyanotic congenital heart disease. 2/24/2014 53 AM (Maré this question =z Question 1d: 96821 Question 19 of 30 Cyanotic congenial heart disease, especially in adults, is associated with which of the following? Y © a) Enythrocytosis. b) Hemochromatosis, c) Leukopenia d) Jaundice. ¢) Exythroderma Question Explanation: Erythrocytosis is one of the body's attempts to compensate for the chronic hypoxemia of cyanotic congenital heart disease, Hemochromatosis, leukopenia, and jaundice are not typically associated with this form of heart disease. Exythroderma, a skin condition sometimes seen in certain leukemias, also is not related to cyanotic congenital heart disease. (Maré this question =z Question 1d: 102961 Question 20 of 30 Noncardiogenic pulmonary ederra does NOT result from which of the following? a) Narcotic overdose ) High altinde exposure ©) Mitral stenosis, 4) Brain mmors. ©) Uremia Question Explanation: Tlicit parenteral heroin use is the most common cause of narcotic-induced pulmonary edema. Because pulmonary edema may also occur following an overdose with other oral and parenteral preparations, the idea of injected impuriies producing the edema has been largely discounted Most probably, alteration in the permeability of alveolar and capillary membrenes is a major factor in procucing the edema. Hypoxia, pulmorary venous constriction, and pulmonary arteriolar constriction have all been implicated as factors producing pulmonary edema at high akitudes. Abnormalities of sympathetic nervous system actwity have been proposed to produce neurogenic pulmonary edema Even n the absence of a volume load, pulmonary edema may occur in uremia. Ths “low pressure” pulmonary edema is associated with increased permeability of the alveolar capillary membranes. Carciogenic pulmonary edema occurs in mitral stenosis due to an increased pulmonary venous pressure. This may occur in the absence of left ventricular failure. (Maré this question =z Question 1d: 102961 Question 20 of 30 ‘Noncardiogenic pulmonary edema does NOT result from which of the following? a) Narcotic overdose 'b) High altinde exposure Y © ©) Mitral stenosis @) Brain tumors, e) Uremia Question Explanation: Tlicit parenteral heroin use is the most common cause of narcotic-induced pulmonary edema. Because pulmonary edema may also occur following an overdose with other oral and parenteral preparations, the idea of injected impuriies producing the edema has been largely discounted Most probably, alteration in the permeability of alveolar and capillary membrenes is a major factor in procucing the edema. Hypoxia, pulmorary venous constriction, and pulmonary arteriolar constriction have all been implicated as factors producing pulmonary edema at high akitudes. Abnormalities of sympathetic nervous system actwity have been proposed to produce neurogenic pulmonary edema Even n the absence of a volume load, pulmonary edema may occur in uremia. Ths “low pressure” pulmonary edema is associated with increased permeability of the alveolar capillary membranes. Carciogenic pulmonary edema occurs in mitral stenosis due to an increased pulmonary venous pressure. This may occur in the absence of left ventricular failure. 2242014 AM. ‘Mark this question & => Question 21 of 30 A 65 ycar old man with a 17 pack year smoking history presents to the emergency room complaining of shortness of breath and dyspnea on exertion, He has tachypnea, tachycardia, and mild cyanosis. The ECG reveals an irregulanly iregular rhythm with a rate of 112, There are discemible P-waves in lead TI, which show three different morphologies. The QRS complex is 0.08 sec. What is the indicated treatment of this rythrn’? a) Blood. b) Adenosine ©) Lidoceine 4) Synchronized DC cardioversion. &) Oxygen. newer (UEIPANSEERIY) tne sore Explanation Report An vor (Question Explanation: ‘Treatment of MAT is diected toward the underlying cause. In this case, the patient is hypoxic (by the blood gas and cyanosis), probably because he has COPD. The treatment of choice is oxygen. 2242014 AM. ‘Mark this question & => Question 21 of 30 A 65 ycar old man with a 17 pack year smoking history presents to the emergency room complaining of shortness of breath and dyspnea on exertion, He has tachypnea, tachycardia, and mild cyanosis. The ECG reveals an irregulanly iregular rhythm with a rate of 112, There are discemible P-waves in lead TI, which show three different morphologies. The QRS complex is 0.08 sec. What is the indicated treatment of this rythrn’? a) Blood. b) Adenosine ©) Lidoceine 4) Synchronized DC cardioversion. Y © €) Oxygen. newer (UEIPANSEERIY) tne sore Explanation Report An vor Question Explanation: ‘Treatment of MAT is diected toward the underlying cause. In this case, the patient is hypoxic (by the blood gas and cyanosis), probably because he has COPD. The treatment of choice is oxygen. 2/24/2014 AM ‘Mark this question = => Question Td Question 22 of 30 A.59 year old woman has been on digozin for chronic atvial fibrillation for more than ten years. She presents with nausea and near syncope and also develops hypotension. Her ECG revedls paroxystnal atrial tachycardia with 21 block. Which of the following is NOT a treatment for digoxin toxicity? a) Temporary pacemaker b) Lowering the potassium level ©) Ant digoxin antibody 4) Hlevating the potassium level e) Phenytoin (Question Explanation: Hypokalemia and hypomagnesemia can precipitate digoxin toxicity. Acute digoxin toxicity can present with nausea, cardiac asrhythmias, A-V blocks, "yellow vision,” and confision. A temporary pacemaker can stabilize a patient who is hemodynamically compromised irom paroxysmal artrial tachycardia with block. Ant digosan antibodies can be given to somebody with a massive digoxin overdose. Keeping the potassium level between 4.0 and 5 0 is crucial when dgoxin is taken. Phenytoin and lidocaine may be given to treat digoxin induced ventricular tachycardias. 2/24/2014 AM ‘Mark this question = => Question Td Question 22 of 30 4.59 ycar old woman has been on digoxin for chronic azial fibrillation for more than ten years. She presents with nausea and near syncope and also develops hypotension. Her ECG reveals paroxysmal atrial tachycardia with 21 block. Which of the following is NOT a weatment for digoxin toxicity? a) Temporary pacemaker Y¥ © b) Lowering the potassium level c) Ant digoxin antibody d) Elevating the potassium level. €) Phenytoin. (Question Explanation: Hypokalemia and hypomagnesemia can precipitate digoxin toxicity. Acute digoxin toxicity can present with nausea, cardiac asrhythmias, A-V blocks, "yellow vision,” and confision. A temporary pacemaker can stabilize a patient who is hemodynamically compromised irom paroxysmal artrial tachycardia with block. Ant digosan antibodies can be given to somebody with a massive digoxin overdose. Keeping the potassium level between 4.0 and 5 0 is crucial when dgoxin is taken. Phenytoin and lidocaine may be given to treat digoxin induced ventricular tachycardias. 2/24/2014 6:50:57 AM ‘Mark this question e& => Question Id : 118410 Question 23 of 30 Goldman's cardiac tisk index aims to predict the posubilty of cardiac problems associated with nen carciac surgery. All of the following are correlative with increased risk of cardiac problems as defined by Goldman’s criteria, EXCEPT a) Presence of $3 gallop b) Old age c) Emergency surgery 4) Non-sinus rhythm or premature atrial contractions on ECG ©) Neurological surgery Answer | Exrianaton | Other User's Explanation Report An Error Question Explanation: Goldinan’s cardiac risk index includes the following >70 years of age, acute MI within the last half year, S3 gallop, emergency surgery, intraperitoneal or intrathoracic operation, presence of non situs shythm or premature atrial contractions on ECG, and generalized poor medical status. Neurologic surgery is not predictive for cardiac problems in Goldman's cardiac risk index. 2/24/2014 6:50:57 AM ‘Mark this question e& => Question Id : 118410 Question 23 of 30 Goldman's cardiac risk index aims to predict the possbilty of cardiac problems associated with non cardiac surgery. All ofthe following are correlative with increased risk of cardiac problems as defined by Goldman's criteria, EXCEPT a) Presence of $3 gallop b) Old age c) Emergency surgery 4) Non-sims rhythm or premature atria contractions oa ECG Y © e) Neurological surgery Answer | Exrianaton | Other User's Explanation Report An Error Question Explanation: Goldinan’s cardiac risk index includes the following >70 years of age, acute MI within the last half year, S3 gallop, emergency surgery, intraperitoneal or intrathoracic operation, presence of non situs shythm or premature atrial contractions on ECG, and generalized poor medical status. Neurologic surgery is not predictive for cardiac problems in Goldman's cardiac risk index. ‘Maz this question <=> Question Td : 126590 Question 24 of 30 ‘A 62 years old man presents with chest pain, He has a long history of angina and cardiac falure and his angina has recently increased in frequency and severity. Over the last 24 hours he has experienced angina at sest. Which one of the following should NOT be included in the management? a) Admit to intensive care urit and place on bed rest b) Start a beta blocker. ©) Stert nitroglycerin therapy. o) Start verapamil €) Schedule for coronary arteriography when acute chest pain resolves. newer (UERREREEII) otertisereExplanason Report AnEror Question Explanation: Calcium channel blockers such as verapamil should be avoided in patients with heart failure. The other answer choices are all part of standard management of a patient with unstable angina. Beta blockers would be contraindicated in a patient with heart block, but there is no mention of this inthis patient. Coronary arteriography should be done once the patient is stabilized, ‘Maz this question <=> Question Td : 126590 Question 24 of 30 A 62 years old man presents with chest pain. He has a longhhistory of angina and cardiac failure and his angina has recently increased in frequency and severity. Over the last 24 hours he has experienced angina at rest. Which one of the following should NOT be included in the management? a) Admit to intensive care urit and place on bed rest. ) Start a beta blocker. ©) Start nitroglycerin therapy, Y © ob) Start verapamil. ©) Schedule for coronary arteriography when acute chest pain resolves. newer (UERREREEII) otertisereExplanason Report AnEror Question Explanation: Calcium channel blockers such as verapamil should be avoided in patients with heart failure. The other answer choices are all part of standard management of a patient with unstable angina. Beta blockers would be contraindicated in a patient with heart block, but there is no mention of this inthis patient. Coronary arteriography should be done once the patient is stabilized, Mark this question ez Question Td : 160223 Question 25 of 30 A 63-year-old woman is symptomless but is found to have a harsh midsystolic rmurmar heard best at the apex and radiating to the carctids. Her apical impulse is displaced to the 6th intercostal space, midclavicular line. Breath sounds are clear bilaterally. ECG is normal. Follow-up echocardiography reveals mild aortic stenosis. Whatis the best next step in management? a) Balloon valyaloplasty ) Enalapril c) Heart transplant 4) Hydrochlorothiazide €) Metoprolol 8) Valve replacement Question Explanation: ‘This patient has asymptomatic aortic stenosis. The best next step in management is to prescribe a diuretic, such as hydrochlorothiaride, in an attempt to reduce preload Reducing preloadiis the basis behind medical management of stenctic left-heart lesions, ‘The reason behind this s that a stenotic lesion, either at the mitral valve or atthe aortic valve, will lead to an increase in preload and a teack-presmure tothe hungs, causing pulmoxary congestion. Therefore, giving diuretics early in cortic stenosis prolongs the development of cymotoms (emacope, angina, and dyspnea) and thus the need for valve replacement surgery Escentially, anything that decreases preload with ctenotic lesiene decreases the murmur and thuc decreases eymptoms or decreases progrecsion te eymptomes. Giving diuretics also decreases the cardiac workload, prolonging the development of angina in those pationts with aortic stenosis Balloon valvuloplasty is not really indicated for acrtic stenosis. This procedure is best for mitral stenosis because of the nature of the stenosis, Mitral valve stencsis is moslly Gbrotic innature, meaning that itis easier to dilate by passing a guidewire fom the femoral tery to the left heast and 'stretching" the stenosis Aortic stenosis is composed of calcific hardening of the aontc leaflets and thus is less amenable to balloon dilatation, and there is a greater risk of "showering" calfic microembol to the brain, gut, and extremities when dating an aortic valve with balloon valvaloplasty. Enalapril is incorrect. ACE inhibitors do not form part of the medical management of stenotic lesions. ACE mhibitors can actually be dangerous in patients with aortic stenosis because they are not able to increase the CO in response to a decrease n afterload, ACE inhibitors are used as part of the medical management of regurgitent left-heart lesions in an zttempt to decrease afterload (decreasing afterload increases cardiac output in those patients with regurgitant left-heart murmaurs) Heart transplant is not the best next step because itis a bit over the top, considering this woman is asymptomatic. It is best to start cout with medical management prior to considering surgery with asymptomatic aortic stenosis. Valve replacement is indicated in severe acrtic stenosis and when the following features have been identified: an area <1em2 or a gradient >50 mm Hi. Metoprolcl, a beta-blocker, can be used in the treatment of aortic stenosis when there is atrial Fibrillation in order to control the lhuthm Retachlarere are weed with eaten in arder ta avaid a eevere decreace in heattrate which will lead ta aredured C(O Mark this question ez Question Td : 160223 Question 25 of 30 A 63-year-old woman is symptomless but is found to have a harsh reidaystolic murmur keard best at the apex and radiating to the carotide. Her apical impulse ic dieplaced to the 6th intercostal space, midclavicular line. Breath sounds are clea: bilaterally. ECG ie normal, Follow-up echocardiography reveals mild acrtic stenosis. Whatis the best next step in management? a) Balloon valvaloplasty ) Enalapril c) Heart transplant Y © d) Hydrechlorethiazide ©) Metoprolol £) Valve replacement Question Explanation: ‘This patient has asymptomatic aortic stenosis. The best next step in management is to prescribe a diuretic, such as hydrochlorothiaride, in an attempt to reduce preload Reducing preloadiis the basis behind medical management of stenctic left-heart lesions, ‘The reason behind this s that a stenotic lesion, either at the mitral valve or atthe aortic valve, will lead to an increase in preload and a teack-presmure tothe hungs, causing pulmoxary congestion. Therefore, giving diuretics early in cortic stenosis prolongs the development of cymotoms (emacope, angina, and dyspnea) and thus the need for valve replacement surgery Escentially, anything that decreases preload with ctenotic lesiene decreases the murmur and thuc decreases eymptoms or decreases progrecsion te eymptomes. Giving diuretics also decreases the cardiac workload, prolonging the development of angina in those pationts with aortic stenosis Balloon valvuloplasty is not really indicated for acrtic stenosis. This procedure is best for mitral stenosis because of the nature of the stenosis, Mitral valve stencsis is moslly Gbrotic innature, meaning that itis easier to dilate by passing a guidewire fom the femoral tery to the left heast and 'stretching" the stenosis Aortic stenosis is composed of calcific hardening of the aontc leaflets and thus is less amenable to balloon dilatation, and there is a greater risk of "showering" calfic microembol to the brain, gut, and extremities when dating an aortic valve with balloon valvaloplasty. Enalapril is incorrect. ACE inhibitors do not form part of the medical management of stenotic lesions. ACE mhibitors can actually be dangerous in patients with aortic stenosis because they are not able to increase the CO in response to a decrease n afterload, ACE inhibitors are used as part of the medical management of regurgitent left-heart lesions in an zttempt to decrease afterload (decreasing afterload increases cardiac output in those patients with regurgitant left-heart murmaurs) Heart transplant is not the best next step because itis a bit over the top, considering this woman is asymptomatic. It is best to start cout with medical management prior to considering surgery with asymptomatic aortic stenosis. Valve replacement is indicated in severe acrtic stenosis and when the following features have been identified: an area <1em2 or a gradient >50 mm Hi. Metoprolcl, a beta-blocker, can be used in the treatment of aortic stenosis when there is atrial Fibrillation in order to control the lhuthm Retachlarere are weed with eaten in arder ta avaid a eevere decreace in heattrate which will lead ta aredured C(O ‘Mark this question = => Question Td : 165371 Question 26 of 30 A male aged 27 years is admitted to the hospital for lethargy and confusion. He has a 5 year history of severe hypertension that has been refractory to medical therapy. Recently he has become increasingly lethargic and confused. On the day of admission, he was found in his home, lying on the flor and unresponsive. His roommate called emergency services, and he was found to have hypotension, dry mucous membranes. and low jugular venous pressure in the emergency department. Lab values are as follows: [Serum sodium [112 mEq/L [Serum potassium|5.1 mEq/L | ‘Serum osmolality|230 mOsmoleg H2 0 [Trine sodum [4 mEq/L | ‘Which substance is most likely mediating the hyponatremia? a) Aldosterone ») Antidiaretic hormone (ADH) c) Cortisol d) Somatostatin ¢) Thyroid stimulating hormone (TSE) Anower (ERRIREN) otter sors Explanation Repo An irr Question Explanation: Hyponatremia is the most common electrolyte abno:mality encountered in the hospital. The combination of hypotension, cry ameous membranes, low jugulas venous pressure, and low urine sodium suggests thatthe patient is markedly volume depleted, In states in which the body is volume depleted, the drive to replenish intravascular volume is greater than the need to maintan osmolaity, As cach, secretion of antidiuretic hormene (ADE) from the pituitary increases, and free water is setained at the level ofthe collecting ucts in the kidney. This process caa lead to profound hyponatremia, as inthis case. Aldosterone, refecred to as the “salt saving hhormore'" is also increased in hypovolemia This process occurs indrectly through the secretion of renin by the juxtaglomerular cells of the kidney, which leads to the conversion of angiotensin I to angiotensin I, the substance that ultimately canses aldosterone secretion, High levels of aldosterone would present as hypokalemia in the absence of hyponatremia, Although itis liely to be increased in this case, aldosterone is not the prime player mediating the hyponatremia. A. deficiency not an elevation of cortisol, as in ‘Addison disease, may cause hyponatremia secondary to ahypersecretion of ADH. This hypersecretion of ADH occurs directly through co-secretion with corticctrophin releasing factor and ndrectly through volume depletion Somatostatin is a hypothalamic hormone as well as a neurotransmitter. Somatostatin is also present in the gastrointestinal tract and helps regulate insulin and glucagon secretion. Despite these function, it plays litle role here in meciating the hyponatremia. Thyroid stimulating hormone (TSH) levels are elevated in the presence of hypothyroidism, a condition that can contribute to elevated levels of ADH and hyponatremia, indirectly, through decreased cardiac output Tn this case however elevated ADH levels from volume depletion appear to be the central mechanism mediating the |e: ‘Mark this question = => Question Td : 165371 Question 26 of 30 A male aged 27 years is admitted to the hospital for lethargy and confusion. He has a 5 year history of severe hypertension that has been refractory to medical therapy. Recently he has become increasingly lethargic and confused. On the day of admission, he was found in his home, lying on the flor and unresponsive. His roommate called emergency services, and he was found to have hypotension, dry mucous membranes. and low jugular venous pressure in the emergency department. Lab values are as follows: [Serum sodium [112 mEq/L [Serum potassium|5.1 mEq/L | ‘Serum osmolality|230 mOsmoleg H2 0 [Trine sodum [4 mEq/L | ‘Which substance is most likely mediating the hyponatremia? a) Aldosterone Y © b) Artidiuretic hormone (ADH) c) Cortisol d) Somatostatin ¢) Thyroid stimulating hormone (TSE) Anower (ERRIREN) otter sors Explanation Repo An irr Question Explanation: Hyponatremia is the most common electrolyte abno:mality encountered in the hospital. The combination of hypotension, cry ameous membranes, low jugulas venous pressure, and low urine sodium suggests thatthe patient is markedly volume depleted, In states in which the body is volume depleted, the drive to replenish intravascular volume is greater than the need to maintan osmolaity, As cach, secretion of antidiuretic hormene (ADE) from the pituitary increases, and free water is setained at the level ofthe collecting ucts in the kidney. This process caa lead to profound hyponatremia, as inthis case. Aldosterone, refecred to as the “salt saving hhormore'" is also increased in hypovolemia This process occurs indrectly through the secretion of renin by the juxtaglomerular cells of the kidney, which leads to the conversion of angiotensin I to angiotensin I, the substance that ultimately canses aldosterone secretion, High levels of aldosterone would present as hypokalemia in the absence of hyponatremia, Although itis liely to be increased in this case, aldosterone is not the prime player mediating the hyponatremia. A. deficiency not an elevation of cortisol, as in ‘Addison disease, may cause hyponatremia secondary to ahypersecretion of ADH. This hypersecretion of ADH occurs directly through co-secretion with corticctrophin releasing factor and ndrectly through volume depletion Somatostatin is a hypothalamic hormone as well as a neurotransmitter. Somatostatin is also present in the gastrointestinal tract and helps regulate insulin and glucagon secretion. Despite these function, it plays litle role here in meciating the hyponatremia. Thyroid stimulating hormone (TSH) levels are elevated in the presence of hypothyroidism, a condition that can contribute to elevated levels of ADH and hyponatremia, indirectly, through decreased cardiac output Tn this case however elevated ADH levels from volume depletion appear to be the central mechanism mediating the |e: 2/24/2014 6:52:03 AM ‘Marc this question & => ‘Question Id : 175285 Question 27 of 30 ‘A patient in an ICU develops sepsis. His blood pressure drops to 60/15 mm Hg and he starts to bleed from venipuncture sites. Disseminated intravascular coagulation is suspected. The diagnosis would be confirmed by a) Decreased PT, decreased PTT, decreased platelets b) Decreased PT elevated PTT, decreased platelets ©) Decreased PT elevated PTT, elevated platelets 4) Elevated PT elevated PTT, decreased platelets €) Elevated PT elevated PTT, elevated platelets Question Explanation: Disseminated intravascular coagulation (DIC) is a feared complication of many serious illnesses, including obstetric complications, sepsis, malignancies, and severe trauma. The most characteristic initiating event is formation (and often subsequent lysis) of microthrombi composed of platelets, fibrin, and coagulative factors. The consumption of coagulative factors leads to increased. prothrombin time (PT) and partial thromboplastin time (PTT). The consumption of platelets leads to a decreased platelet count, A more specific confirmatory test is an increase in fibrin spit products, such as "D-dimers." Characteastaly, DIC clinically produces a sist of hemorrhagic end coagulative complications, and is dificult to treat because comrecting the coagulative tendency tends to worsen the hemosrhage, and correcting the hemorrhagic tendency tends to worsen thrombotic complications 2/24/2014 6:52:03 AM ‘Marc this question & => ‘Question Id : 175285 Question 27 of 30 ‘A patient in an ICU develops sepsis. His blood pressure drops to 60/15 mm Hg and he starts to bleed from venipuncture sites. Disseminated intravascular coagulation is suspected. The diagnosis would be confirmed by a) Decreased PT, decreased PTT, decreased platelets b) Decreased PT elevated PTT, decreased platelets ©) Decreased PT elevated PTT, elevated platelets Y © A) Elevated PT elevated PTT, decreased platelets €) Elevated PT elevated PTT, elevated platelets Question Explanation: Disseminated intravascular coagulation (DIC) is a feared complication of many serious illnesses, including obstetric complications, sepsis, malignancies, and severe trauma. The most characteristic initiating event is formation (and often subsequent lysis) of microthrombi composed of platelets, fibrin, and coagulative factors. The consumption of coagulative factors leads to increased. prothrombin time (PT) and partial thromboplastin time (PTT). The consumption of platelets leads to a decreased platelet count, A more specific confirmatory test is an increase in fibrin spit products, such as "D-dimers." Characteastaly, DIC clinically produces a sist of hemorrhagic end coagulative complications, and is dificult to treat because comrecting the coagulative tendency tends to worsen the hemosrhage, and correcting the hemorrhagic tendency tends to worsen thrombotic complications ‘Mark this question & => Question Td : 178880 Question 28 of 30 A.52 year old man is brought wih substemal chest pain. ECG shows ST segment elevation and T wave inversion. He develops asthythunia several hours later. ECG shows random electrical actimty with out recognizable QRS complexes. Which statement describes this archythmia? a) Accelerated idioventricular chythm 'b) Accelereted junctional rhythm c) Premature ventricular contraction 4) Ventricular fibrillation ©) Ventricular tachycardia Answer | Explanation) Other User's Explanation Report An Error Question Explanation: ‘The rhythm described is that of ventricular brillation, which is a feared complication of myocardial infarction that must be corrected immediately (CPR, defibrilation, TV’ and intracardiac drugs incinding epinephrine, lidocaine, or procainamide) ifthe patient is to survive. In an accelerated idioventricular thythm, a normal letent pacemaker in the ventricles depolarizes at a regular, accelerated rete 9650 to 100/min, cach time producing unusually shaped (but similar to each other) QRS complexes. P waves related to the complexes art not seen, Tn an accelerated junctional rhytim, the P waves are typicaly inverted and may precede, follow, ot be hidden within regular QRS complexes thet occur at a rate of 60 to 150/mnin Ina premanure ventricular contraction, an ectopic, venticular pacemaker inserts an ectopic beat (typically with a wide and bizarre ORS complex) before the next sims beat occurs. Ia venticular tachycardia, wide and bizarre, but recognizable, QKS complexes occur at an accelerated rate. ‘Mark this question & => Question Td : 178880 Question 28 of 30 4.52 year old man is brought with substemal chest pain. ECG shows ST segment elevation and T wave inversion. He develops asrhythunia several hours later. ECG shows random electrical actinty with out recognizable QRS complexes, Which statement describes this artaythmia? a) Accelerated idioventsicular rhythm 'b) Accelerated junctional rhythm c) Premature ventricular contraction Y © 4) Ventricular fibrillation €) Ventricular tachycardia Answer | Explanation) Other User's Explanation Report An Error Question Explanation: ‘The rhythm described is that of ventricular brillation, which is a feared complication of myocardial infarction that must be corrected immediately (CPR, defibrilation, TV’ and intracardiac drugs incinding epinephrine, lidocaine, or procainamide) ifthe patient is to survive. In an accelerated idioventricular thythm, a normal letent pacemaker in the ventricles depolarizes at a regular, accelerated rete 9650 to 100/min, cach time producing unusually shaped (but similar to each other) QRS complexes. P waves related to the complexes art not seen, Tn an accelerated junctional rhytim, the P waves are typicaly inverted and may precede, follow, ot be hidden within regular QRS complexes thet occur at a rate of 60 to 150/mnin Ina premanure ventricular contraction, an ectopic, venticular pacemaker inserts an ectopic beat (typically with a wide and bizarre ORS complex) before the next sims beat occurs. Ia venticular tachycardia, wide and bizarre, but recognizable, QKS complexes occur at an accelerated rate. 2/24/2014 6:52:32 AM ‘Mat this question a => Question Id : 206747 Question 29 of 30 A74 year old man has chest pain and dyspnoea His troponin Tis 0.5 (NR “0.02 ng/ml). His pain subsices and he is generally well, tut dyspnoea restricts his movement, He is unable to manage the stairs but can mobilize solely around the ward. During monitoring, his telemetry shows short nuns of non-sustained ventricular tachycardia The next appropriate investigation for hisn is a) 24 hour tape b) Coronary angiography ©) Echocardiography ) Electrophysiological studies €) Outpatient cardiology referral Question Explanation ‘This question is a nod to the recent evidence suggesting that patient with reduced lett ventricular (LV) function and asymptomatic ventricular tachycardia (V'T) may beneft ffom implantation of an implantable cardiac defitelator (for example, the MADIT study) ‘Youneed to know the patient's cardiac fimction, and we feel that an echocardiogram (ECHO) would be the nest most appropnate study in this patient, followed by coronary angiography and inpatient cardiology referral. 2/24/2014 6:52:32 AM ‘Mat this question a => Question Id : 206747 Question 29 of 30 A74 year old man has chest pain and dyspnoea His troponin Tis 0.5 (NR “0.02 ng/ml). His pain subsices and he is generally well, ‘but dyspnoea restricts his movement. He is unable to manage the stairs but can mobilize solely around the werd. During monitoring, his telemetry shows short runs ofnoa-sustained ventricular tachycardia. The next appropriate investigation for him is a) 24 hour tape ') Coronary angiography JY © c) Echocardiography 4) Electrophysiological studies ©) Outpatient cardiology referral Question Explanation ‘This question is a nod to the recent evidence suggesting that patient with reduced lett ventricular (LV) function and asymptomatic ventricular tachycardia (V'T) may beneft ffom implantation of an implantable cardiac defitelator (for example, the MADIT study) ‘Youneed to know the patient's cardiac fimction, and we feel that an echocardiogram (ECHO) would be the nest most appropnate study in this patient, followed by coronary angiography and inpatient cardiology referral. Mark this question <= Question Id: 213669 Question 30 of 30 A 60-year-old arymptomatic mele with a history of rheumatic fever appears plethoric. Hle has an irregularly isreguler pulse. A tapping apex beat exists, Breath sounds are clear bilaterally. Auscultation of the chest with the patient in the lef lateral position shows a iastolic murmur heard best with the bell on expiration, An electrocardogram reveals atrial fibrillation, What is the best initial test? a) Dobuanine echocardiography b) MUGA sean c) Left heaut catheterization and angiography 4) Stress test e) Transthoracic echocardiography 8) Myocardial perfusion scanning Question Explanation: ‘This patient has signs of mitral stenosis plus history of rheumatic fever. Two-thirds of patients with mitral stenosis are female, The signs that point toward this diagnosis are 2 murmeur that is diastoic (the classic "rnid-diastolic rumble" heard best at the apex is mitral stenosis until proven otherwise) and is loudest on expiration (Which localizes the lesion to the left side of the heart), the tapping apex beat, and the evidence of atrial fbrilation on ECG, Whenever armurmur is suspected on physical examination, the best initial testis always transthoracic echo, which will visualize the valvular lesion, determine the extent of stenosis or regurgitation (through Doppler imaging), define the ejection fraction, and identify any left atrial thrombi that may have developed as a result of atrial fibrillation (Transesophageal echo actually has a better resclution in regard to picking up lei atvial thrombi, but itis not the best intial test) Stress testing is reserved for those patients who present with stable angina ot who have recently recovered fiomn an MI This patient has a murmur, the extent cf which needs to he vicuslized and quantified by transthoracic echo Dobutamine echocardiography ie reserved for those patients who present with a hictory of ctable angina but are unable to exercice on « treadmill because they physically cannot exercise or they have COPD, peripheral vascular disease, or morbid obesity. A MUGA scan (a mulligaied acquisition image) is obtained by intravenous injection of [99mmTc], which attaches to the paiient's own red cells in vivo and is therefore retained in the vascular space MUGA scanis designed to provide the most accurate determination of the ejection fraction in those patients with signs and symptoms of chronic heart falure. “Although left heart catheterization and angography is the most accurate at determining left heart pressures and visualizing the lesion, as well as determining ejection fraction and systolic dysfunction as a result of valvular heart disease, itis not the best intial test because itis invasive Myocardial perfision scanning is used to see myocardial function as affected by coronary artery disease, ett Report An Error Mark this question <= Question Id: 213669 Question 30 of 30 A 60-year-old asymptomatic mele with a history of rheumatic fever appears plethoric. He has an irregularly irreguler pulse. A tapping apex beat exists. Breath sounds arc clear bilaterally. Auscultation of the chest with the patient in the left lateral position shows a diastolic murmur heard best with the bell on expiration. An electrocardiogram reveds atrial fibrillation. What is the best initial test? a) Debutamine echocardiography 6) MUGA scan c) Left heart catheterization and angiography 4) Stress test Y © e) Transthoracic echocardiography £) Myocardial perfusion scanning Question Explanation: ‘This patient has signs of mitral stenosis plus history of rheumatic fever. Two-thirds of patients with mitral stenosis are female, The signs that point toward this diagnosis are 2 murmeur that is diastoic (the classic "rnid-diastolic rumble" heard best at the apex is mitral stenosis until proven otherwise) and is loudest on expiration (Which localizes the lesion to the left side of the heart), the tapping apex beat, and the evidence of atrial fbrilation on ECG, Whenever armurmur is suspected on physical examination, the best initial testis always transthoracic echo, which will visualize the valvular lesion, determine the extent of stenosis or regurgitation (through Doppler imaging), define the ejection fraction, and identify any left atrial thrombi that may have developed as a result of atrial fibrillation (Transesophageal echo actually has a better resclution in regard to picking up lei atvial thrombi, but itis not the best intial test) Stress testing is reserved for those patients who present with stable angina ot who have recently recovered fiomn an MI This patient has a murmur, the extent cf which needs to he vicuslized and quantified by transthoracic echo Dobutamine echocardiography ie reserved for those patients who present with a hictory of ctable angina but are unable to exercice on « treadmill because they physically cannot exercise or they have COPD, peripheral vascular disease, or morbid obesity. A MUGA scan (a mulligaied acquisition image) is obtained by intravenous injection of [99mmTc], which attaches to the paiient's own red cells in vivo and is therefore retained in the vascular space MUGA scanis designed to provide the most accurate determination of the ejection fraction in those patients with signs and symptoms of chronic heart falure. “Although left heart catheterization and angography is the most accurate at determining left heart pressures and visualizing the lesion, as well as determining ejection fraction and systolic dysfunction as a result of valvular heart disease, itis not the best intial test because itis invasive Myocardial perfision scanning is used to see myocardial function as affected by coronary artery disease, ett Report An Error 2/24/2014 65: AM Mack this question A 17-year-old female is admitted with an overdose of propranolol tablets approximately 2 hours ago. She ic drowsy, pul: => Question 1 of 30 Question Id : 24668 40 bpm, blood pressure— 80/40 mmHg, She is treated with activated charcoal, TV fluids and IV etropine but her bradycardia and hypotension fail to respond, Which of the following would be the most appropriate next stage in her management? a) TV adkenaline b)IV amiodarone ©) IV Phentolamine DTV gucagon €) Insertion of temporary pacemaker Anewor (UERMAEN) otherUcersExplanation Report An Exor (Question Explanation: Jn those in whom intial atropine is unsuccessful, [V Glucagon is a recommended treatment for beta-blocker overdose with some. evidence indicating improvernentin bradycardia end blood pressure. 2/24/2014 65: AM Mack this question A 17-year-old female is admitted with an overdose of propranolol tablets approximately 2 hours ago. She ic drowsy, pul: => Question 1 of 30 Question Id : 24668 40 bpm, blood pressure— 80/40 mmHg, She is treated with activated charcoal, TV fluids and IV etropine but her bradycardia and hypotension fail to respond. Which of the following would be the most appropriate next stage ia her management? a) TV adrenaline )IV amiodarone ©) IV Phentolamine Y © ATV glucagon ) Insertion of temporary pacemaker Anewor (UERMAEN) otherUcersExplanation Report An Exor (Question Explanation: Jn those in whom intial atropine is unsuccessful, [V Glucagon is a recommended treatment for beta-blocker overdose with some. evidence indicating improvernentin bradycardia end blood pressure. 2/24/2014 6:54:19 AM ‘Maz this question = => Question Td : 49864 Question 2 of 30 In patients with cocaine induced arrhythmias, which of the following is contraindicated? a) Lidocaine b) Sodium bicarbonate ¢) Metoprolol (Lopressor) 4) Lorazepam (Ativan) €) Verapamil (Calan, Isoptin) Question Explanation: ‘When treating arthythmias releted to cocaine toxicity, hypertonic sodium bicarbonate and benzodiazepines may be given when the istinction between sodium channel blockade induced QRS-complex widening and ischemia induced ventricular tachycardia is unclear. Lidocaine may subsequently be utlized ifnecessary. Verapamil has been shown to reverse cocaine-indaced coronary vasospasin, Beta adrenergic blocking drugs have been shown to exacerbate coronary vasospasm by sesulting in unopposed alpha- adrenergic activity. Beta blockers are therefore contraindicated in the treatment of cocaine-mduced cardiac problems 2/24/2014 6:54:19 AM ‘Maz this question = => Question Td : 49864 Question 2 of 30 In patients with cocaine induced arhythmias, which of the following is contraindicated? a) Lidocaine +b) Sodium bicarbonate Y © 0) Metoprolol Lopresson) d) Lorazepam (Ativan) €) Verapamd (Calan, Isoptin) Question Explanation: ‘When treating arthythmias releted to cocaine toxicity, hypertonic sodium bicarbonate and benzodiazepines may be given when the istinction between sodium channel blockade induced QRS-complex widening and ischemia induced ventricular tachycardia is unclear. Lidocaine may subsequently be utlized ifnecessary. Verapamil has been shown to reverse cocaine-indaced coronary vasospasin, Beta adrenergic blocking drugs have been shown to exacerbate coronary vasospasm by sesulting in unopposed alpha- adrenergic activity. Beta blockers are therefore contraindicated in the treatment of cocaine-mduced cardiac problems ‘Mark this question e => Question Td : 50569 Question 3 of 30 A.70 year old man with heart failure has had high blood pressure treated for long time. He was trought to ER. His vitals show a blood pressure of 80/50 and a heartrate of 100. The most appropriate nest step is to administer whick of the following? a) Procaine b) Lidocaine c) Cardioversion 4) Digoxin ©) Furosemide Answer | Explanation) Other User's Explanation Report An Error Question Explanation: Congestive heart failure (CETF) is a condition in which the heart's function as a pump to deliver cxygen rich blood to the body is inadequate to meet the body's needs. Symptoms include dyspnea, fatigue, and peripheral edema, Diagnosis is cinical and by chest x- ray and echocardiography. Treatment is the same as for heart faiure ACE inhibitors, beta-blockers, diuretics and digoxin. This patient is likely in an acute congestive heart failure exacerbation. ‘Mark this question e => Question Td : 50569 Question 3 of 30 A.70 year old man with heart failure has had high blood pressure treated for long time. He was trought to ER. His vitals show a blood pressure of 80/50 and a heartrate of 100. The most appropriate nest step is to administer whick of the following? a) Procaine b) Lidocaine c) Cardioversion ¥ © 4)Digoxin ©) Furosemide Answer | Explanation) Other User's Explanation Report An Error Question Explanation: Congestive heart failure (CETF) is a condition in which the heart's function as a pump to deliver cxygen rich blood to the body is inadequate to meet the body's needs. Symptoms include dyspnea, fatigue, and peripheral edema, Diagnosis is cinical and by chest x- ray and echocardiography. Treatment is the same as for heart faiure ACE inhibitors, beta-blockers, diuretics and digoxin. This patient is likely in an acute congestive heart failure exacerbation. ‘Mark this question Question Id : 55482 o> Question 4 of 30 ‘The most effective control oP heatt rate in atrial fibrillation, both at rest and with exercise or long-term therapy occurs with which one of the following? a) Digitalis b) Beta Adrenergic blockers c) Calcium channel blockers 4) Class 1A antiarrhythmios Question Explanation: For long term therapy, Beta-adrenergic antagonist drugs provide the most effective control of heart rate in atrial-fibrillation, both at rest and during exercise, Although calcium channel blockers also lower heart rate both at rest and with exercise, they are not as effective as Beta-blockers. Digitalis is primarily effective in controlling the heart rate at rest, and often dees not adequately control heart rate with exercise. The Class 1 antiarthythmics are most usefll in maintaining sinus chythm and, in fact, may paradoxically increase heart rate ‘Mark this question Question Id : 55482 o> Question 4 of 30 ‘The most effective control of heart rate in atrial fibrillation, both at rest and with exercise or long-term therapy occurs with which one of the following? a) Digitalis Y © b) Beta-Adrenergic blockers c) Calcium channel blockers 4) Class 1A antiarrhythmios Question Explanation: For long term therapy, Beta-adrenergic antagonist drugs provide the most effective control of heart rate in atrial-fibrillation, both at rest and during exercise, Although calcium channel blockers also lower heart rate both at rest and with exercise, they are not as effective as Beta-blockers. Digitalis is primarily effective in controlling the heart rate at rest, and often dees not adequately control heart rate with exercise. The Class 1 antiarthythmics are most usefll in maintaining sinus chythm and, in fact, may paradoxically increase heart rate 2/24/2014 65: AM. ‘Maric this question <= => Question Td: 59611 Question 5 of 30 A.54 year oldman is being considered for pharmacologic treatment of hyperlipidemia because of an LDL cholesterol level of 180 mg/AL. Before begining medication for his hyperlipidemia, he should be screened for which of the following? a) Hyperthyroidism b) Hypothyroidism ©) Addison’s disease 4) Cushing's disease €) Pemicious anemia Question Explanation: According to the Summary of the National Cholesterol Education Program (NCEP) Aduit Treatment Panel IT Report, auty person with elevated LDL cholesterol or any other form of hypertipidemia should undergo clinical or laboratory assessment to rule out secondary dyslipidemia before iniiation of ipid lowering therapy. Causes of secondary dyslipidemia include diabetes mellitus, hypothyroidism, obstructive liver disease, chronic renal failure, and some medications. 2/24/2014 65: AM. ‘Maric this question <= => Question Td: 59611 Question 5 of 30 4.54 ycar oldman is being considered for pharmacologic treatment of hyperlipidemia because of an LDL cholesterol level of 180 mg/dL. Before beginning medication for his hyperlipidemia, he should be screened for which of the folowing? a) Hyperthyroidism Y © b) Hypothyroidism ©) Addison’s disease d) Cushing's disease e) Pemicious anemia Question Explanation: According to the Summary of the National Cholesterol Education Program (NCEP) Aduit Treatment Panel IT Report, auty person with elevated LDL cholesterol or any other form of hypertipidemia should undergo clinical or laboratory assessment to rule out secondary dyslipidemia before iniiation of ipid lowering therapy. Causes of secondary dyslipidemia include diabetes mellitus, hypothyroidism, obstructive liver disease, chronic renal failure, and some medications. ‘Mat this question & => Question I Question 6 of 30 ‘A patient is noted to have a weak and delayed carotid pulse. This pulse is most consistent with, a) Aortic regurgitation b) Mitral stenosis: c) Aortic stenosis 4) Progressive systolic heart faihure ©) Hypertrophic cardiomyopathy Answer | Explanation Other User's Explanation —_ Report An Error Question Explanation: Pulsus parvus et tardus (weak and late) is the classic pulse of gortic stenosis. Obstruction to blood flow is the mechanistn behind this pulse patter. Aortic regurgitation and hypertrophic cardiomyopathy may both cause pulsus bisferiens (two systolic peaks.). Cardiac tamponade causes a pulses paradomns (fallin the systolic blood pressure 10-12 mm Hg dunng inspiration). Mitral stenosis is not associated with a particalar pulse pattern, Progressive systolic heart failue consists if Pulsus Alternans, ‘Mat this question & => Question I Question 6 of 30 ‘A patieat is ncted to have a weak and delayed carotid pulse, This pulse is most consistent with a) Aortic regurgitation b) Miral stenosis Y © c) Aortic stenosis 4) Progressive systolic heart faihure ©) Hypertrophic cardiomyopathy Answer | Explanation Other User's Explanation —_ Report An Error Question Explanation: Pulsus parvus et tardus (weak and late) is the classic pulse of gortic stenosis. Obstruction to blood flow is the mechanistn behind this pulse patter. Aortic regurgitation and hypertrophic cardiomyopathy may both cause pulsus bisferiens (two systolic peaks.). Cardiac tamponade causes a pulses paradomns (fallin the systolic blood pressure 10-12 mm Hg dunng inspiration). Mitral stenosis is not associated with a particalar pulse pattern, Progressive systolic heart failue consists if Pulsus Alternans, 2/24/2014 6:55:27 AM ‘Matte this question & => Question 7 of 30 A 70- year old patient being treated for cardiac heart failure presents with nausea and vomiting and a history of altered vision. Medications include ibuprofen, digitals, labetalol, ranitidine, and captopril The patient's laboratory vaiues demonstrate hypokalemia, Hectrocardiogram shows A-V block. These symptoms are most consistent with toxicity from what medication? a) Ibuprofen b) Digitalis c) Labetalol 4) Ranitidine ©) Captopril Question Explanation: These signs and symptoms are consistent with digtalis toxicity, which affects 5-15% of patients hospitalized and receiving digitalis. It is exacerbated by hypokalemia, The principal complications ofibuprofen are stomach irrtation and GI bleeding Labetaldl could produce heart block bur does not cause the pettem of other side effects described Rare case reports have described vision chenges with raniticine, but overall t hes few severe side effects. The principal side effects of captopail are hypotension, agranulocytosis, proteinuria, and hyperkalemia 2/24/2014 6:55:27 AM ‘Matte this question & => Question Td : 60431 Question 7 of 30 A 70- year old patient being treated for cardiac heart falure presents with nausea and vorniting and a history of altered vision. Medications include ibuprofen, digitals, labetalol, ranitidine, and captopril The patient's laboratory vaiues demonstrate hypokalemia, Hectrocardiogram shows A-V block. These symptoms are most consistent with toxicity from what medication? a) Ibuprofen Y © b) Digitalis c) Labetalol 4) Ranitidine ©) Captopeil Question Explanation: These signs and symptoms are consistent with digtalis toxicity, which affects 5-15% of patients hospitalized and receiving digitalis. It is exacerbated by hypokalemia, The principal complications ofibuprofen are stomach irrtation and GI bleeding Labetaldl could produce heart block bur does not cause the pettem of other side effects described Rare case reports have described vision chenges with raniticine, but overall t hes few severe side effects. The principal side effects of captopail are hypotension, agranulocytosis, proteinuria, and hyperkalemia 2/24/2014 6:55:43 AM “Mark this question & => Question 8 of 30 Antibiotic prophylasis for patients with rheumatic heast disease is eppropriate for all of the following conditions EXCEPT a) Dental cleaning ) Routine pelvic exam. ©) Dental extraction, 4) Colonescepy €) Sigmoidoscopy. Question Explanation Antibiotic prophylanis is not necessary for routine vaginal exams but is necessary for the other situations Report An Error 2/24/2014 6:55:43 AM “Mark this question & => Question 8 of 30 Antibiotic prophylasis for patients with rheumatic heast disease is eppropriate for all of the following conditions EXCEPT a) Dental cleaning Y © b) Routine pelvic exam. ©) Dental extraction, 4) Colonoscopy £) Sigmoidoscopy. Question Explanation Antibiotic prophylanis is not necessary for routine vaginal exams but is necessary for the other situations Report An Error ‘Mack this question <=> Question 9 of 30 An old woman who has hypertension and diabetes would benefit most from a) Metoprolol ) Amlodipine ©) Benazepzil 4) Hydrochlorothiazide Question Explanation: “Angiotensin converting enzyme (ACE) inhibitors lower blood pressure in part by dilating arterioles. They dilate arterioles by preventing the formation of angictensin I which causes arterioles to constrict. Specifically, these inhibitors block the action of angiotensin- converting enzyme, which converts angiotensin Ito angiotensin II These drugs are particularly useful for people with coronary artery disease ot heart failure, whites, young people, people with protein in their urine because of chronic kidney disease or diabetic kidney disease, and men who develop sexual dysfunction as a side elfect of another anthypertensive drug, ‘Mack this question <=> Question 9 of 30 An old woman who has hypertension and diabetes would benefit most from a) Metoprolol ) Amlodipine JY © c)Benazepril 4) Hydrochlorothiazide Answer | Bplanation Other User's Explanation Report An Error Question Explanation: “Angiotensin converting enzyme (ACE) inhibitors lower blood pressure in part by dilating arterioles. They dilate arterioles by preventing the formation of angictensin I which causes arterioles to constrict. Specifically, these inhibitors block the action of angiotensin- converting enzyme, which converts angiotensin Ito angiotensin II These drugs are particularly useful for people with coronary artery disease ot heart failure, whites, young people, people with protein in their urine because of chronic kidney disease or diabetic kidney disease, and men who develop sexual dysfunction as a side elfect of another anthypertensive drug, 2/24/2014 6:56:12 AM ‘Maz this question = => Question Td : 74458 Question 10 of 30 Used for the treatment of aizial fibrillation associated with acute hemodynamic compromise. a) Digitals ) Quinidine ©) Propranolol 9) Cardioversion €) Cardiac pacemaker Anewor (NENA) other User's Explanation Report An Enror Question Explanation: Cardioversion is dicated for patents with hemodynamic compromise. While drug therapy maybe added prior to cardioversion, the physician should aot postpone cardioversion to wait for the effect of medication ifthe patient is in acute hemodynamic compromise 2/24/2014 6:56:12 AM ‘Maz this question = => Question Td : 74458 Question 10 of 30 ‘Used for the treatment of atrial fitrilation associated with acute hemodynamic compromise a) Digitals ) Quinidine ©) Propranolol VY © Ad) Cardioversion ©) Cardiac pacemaker Anewor (NENA) other User's Explanation Report An Enror Question Explanation: Cardioversion is dicated for patents with hemodynamic compromise. While drug therapy maybe added prior to cardioversion, the physician should aot postpone cardioversion to wait for the effect of medication ifthe patient is in acute hemodynamic compromise ‘Mack this question ez Question Ta : 75763 Question 11 of 30 ‘The therapy of choice for acute myocardial infarction is a) Lidocaine 6) IV magnesiam infasions ©) Calcinen channel blockers ) Percutaneous Transluminal Coronary Angioplasty ©) Digitalis Answer [FERRI orner Users Explanation Report An Enor Question Explanation: Percutaneous Transluninal Coronary Angioplasty (PICA) is the therapy of choice in the management of acute myocardial infarction Ibis essential to establish reperfusion, and PTCA is the most expedient way to accomplish seperfusion. Lidocaine, magnesium and calcium channel blockers have not been shown to be of any benefit in the management of acute myocardial infarction. Digitalis treats failure. ‘Mack this question ez Question Ta : 75763 Question 11 of 30 ‘The therapy of choice for acute myocardial infarction is a) Lidocaine 6) IV magnesiam infasions ©) Calcinen channel blockers Y¥ © d) Percutaneous Transluminal Coronary Angioplasty ©) Digitalis Answer [FERRI orner Users Explanation Report An Enor Question Explanation: Percutaneous Transluninal Coronary Angioplasty (PICA) is the therapy of choice in the management of acute myocardial infarction Ibis essential to establish reperfusion, and PTCA is the most expedient way to accomplish seperfusion. Lidocaine, magnesium and calcium channel blockers have not been shown to be of any benefit in the management of acute myocardial infarction. Digitalis treats failure. 2/24/2014 6:56:38 AM ‘Mark this question e& => (Question Id : 76196 Question 12 of 30 ‘All of the following are goals of post-myocardial infarction exercise treadmill stress testing EXCEPT a) To determine the extent of the let main coronary artery diseace. b) To evaluate the hemodynamic response to exercise. c) To monitor the effect of medications. 4) To mule out exercise-induced ventricular arrhythmias @) To determine prognosis. Answer | Boplanation Other User's Explanation Report An Error Question Explanation: Known left main coronary artery disease is a contraindication for performing an exercise stress test, A patient with hypotension is at risk for sudden death In patients with a negative electrocardiogram response to a workload above 6 METS on strass testing performed three weeks after a heart attack; the one-year mortality rate is below two percent, Ifa patienthas been placed on beta- blockers afler a heart attack, exercise stress testing can determine IF the sympathetic nervous system has been adequately suppressed to prevent another ischemic episode. Exercise stress testingis indicated for most patients with uncomplicated myocardial infarction. IF a patient has ventricular arrhythmias during a stress test, the one-year mortality rate cen exceed 25%. Post-myocardial infarction exercise tolerance testing is a valuable and cost-effective tool for precicting she risk of adverse cardiac events 2/24/2014 6:56:38 AM ‘Mark this question e& => (Question Id : 76196 Question 12 of 30 ‘All of the following are goals of post-myocardial infarction exercise treadmill stress testing EXCEPT Y¥ © a)To determine the extent of the left main coronary artery disease. b) To evaluate the hemodynamic response to exercise. c) To monitor the effect of medications. 4) To mule ovt exercise-induced ventricular araythmias @) To determine prognosis. Answer | Boplanation Other User's Explanation Report An Error Question Explanation: Known left main coronary artery disease is a contraindication for performing an exercise stress test, A patient with hypotension is at risk for sudden death In patients with a negative electrocardiogram response to a workload above 6 METS on strass testing performed three weeks after a heart attack; the one-year mortality rate is below two percent, Ifa patienthas been placed on beta- blockers afler a heart attack, exercise stress testing can determine IF the sympathetic nervous system has been adequately suppressed to prevent another ischemic episode. Exercise stress testingis indicated for most patients with uncomplicated myocardial infarction. IF a patient has ventricular arrhythmias during a stress test, the one-year mortality rate cen exceed 25%. Post-myocardial infarction exercise tolerance testing is a valuable and cost-effective tool for precicting she risk of adverse cardiac events Mate this question = => Question Td : 81176 Question 13 of 30 An increased level of alkaline phosphatase is LEAST licely to originate from which one ofthe following areas? a) Bilary dacts } Bone ) Pheenta od) He o) Intestine Answer al Other Users Explanation Report An Error Question Explanation ‘The heartis the least lixely area to contain alkaline phosphatase, which is mostly found in the liver and bone (B), as well as the placenta (C), intestine (E), and kidney Mate this question = => Question Td : 81176 Question 13 of 30 An increased level of alkaline phosphatase is LEAST Likely to originate from which one of the following areas? a) Bilary dacts } Bone Recs YO die oerine Answer | Paptanation Other Users Explanation Report An Error Question Explanation ‘The heartis the least lixely area to contain alkaline phosphatase, which is mostly found in the liver and bone (B), as well as the placenta (C), intestine (E), and kidney ‘Marie this question & => (Question Td : 81901 Question 14 of 30 ‘The most common congenital cardiac defect is a) Tetralogy of Fallot 'b) Atsial septal defect c) Ventricular septal defect 4) Patent ductus arteriosus ©) Coarctation of the aorta Answer | Bplanation Other User's Explanation Report An Error Question Explanation: ‘A ventricular septal defectis the most common congerital cardiac abnormality. The others are less common, ‘Marie this question & => (Question Td : 81901 Question 14 of 30 ‘The most common congenital cardiac defect is a) Tetralogy of Fallot 'b) Atsial septal defect Y © c) Ventricular septal defect 4) Patent ductus arteriosus ©) Coarctation of the aorta Answer | Bplanation Other User's Explanation Report An Error Question Explanation: ‘A ventricular septal defectis the most common congerital cardiac abnormality. The others are less common, ‘Mark this question <= => Question Ta ; 89448 Question 15 of 30 ‘Which of the following is NOT associated with atrioventricular conduction disturbance? a) Marathoners 'b) Acute inferior wall myocardial infarction. c) Digitalis intoxication. 4) Spasms of the let circus: artery. ©) Chagas disease. Question Explanation: The atrioventricular (AV) node is supplied by both the parasymnpathetic and sympathetic systems. Tris also alfected by autonomic tone, AV nodal slowing, with associated bradycardia, may be seen in highly trained athletes. Nodal ischemia occurring in acute inferior wall myocardial infarction may produce complete heart block. Digtalis is therapeutically useful because i prolongs the refractory period of the AV node. With intoxication, sinus bradycardia and AV block may occur. In Chagas disease, destruction of the parasympathetic ganglia may cause AV conduction disturbances. The AV nodal artery usually arises from the right coronary artery. Spasm of the right coronary artery may produce AV conduction disturbances, ‘Mark this question <= => Question Ta ; 89448 Question 15 of 30 “Which of the following is NOT associated with attiovestricular conduction disturbance? a) Marathoners +b) Acute inferior wall myocardial infarction, ©) Digtalls intoxication. Y © @) Spasm of the left circumflex artery. €) Chagas disease Question Explanation: The atrioventricular (AV) node is supplied by both the parasymnpathetic and sympathetic systems. Tris also alfected by autonomic tone, AV nodal slowing, with associated bradycardia, may be seen in highly trained athletes. Nodal ischemia occurring in acute inferior wall myocardial infarction may produce complete heart block. Digtalis is therapeutically useful because i prolongs the refractory period of the AV node. With intoxication, sinus bradycardia and AV block may occur. In Chagas disease, destruction of the parasympathetic ganglia may cause AV conduction disturbances. The AV nodal artery usually arises from the right coronary artery. Spasm of the right coronary artery may produce AV conduction disturbances, 2/24/2014 6:57:33 AM ‘Mark this question & => Question Td : 89689 Question 16 of 30 ‘A patient presents with the signs and symptoms of congestive heart falurs. Which of the following therapies arc NOT correctly matched with their objective? a) Digitals-improve cardiac performance b) Vasodilators-improve cardiac performance. c) Diuretics-control excess salt and water. 4) Physical rest-reduce cardiac workload. ©) Inowopic agents-improve cardiac performance. Question Explanatioy ‘Vasodilators work by reducing cardiac workload. Physical rest and removal of exacerbating factors such as obesity and anemia also work by this method, Digitalis and inotropic agents, as well as other crugs that cotrect rhythm abnormaities, improve cardiac performance. Diuretics and sodium restriction, along with dialysis in severe cases, control excess salt and water. 2/24/2014 6:57:33 AM ‘Mark this question & => Question Td : 89689 Question 16 of 30 ‘A patient presents with the signs and symptoms of congestive heart falurs. Which of the following therapies arc NOT correctly matched with their objective? a) Digitalis-improve cardiac performance. Y © b) Vasodilators-improve cardiac performance. c) Dimretics-control excess salt and water. 4d) Physical rest-reduce cardiac worldoad. e) Inowropic agents-improve cardiac performance. Question Explanatioy ‘Vasodilators work by reducing cardiac workload. Physical rest and removal of exacerbating factors such as obesity and anemia also work by this method, Digitalis and inotropic agents, as well as other crugs that cotrect rhythm abnormaities, improve cardiac performance. Diuretics and sodium restriction, along with dialysis in severe cases, control excess salt and water. 2/24/2014 AM. Mark this question = => Question 17 of 30 A 28 year old sexually active woman taking oral contraceptives complains of pain in her right calf. She smokes one pack per day for 10 years. Ascending contrast venography demonstrates an obstruction of the deep veins of the calf: The most likely diagnosis is a) Strained calf rmscle ) Bilateral leg edema c) Thrombophlebitis 4) Metastatic tumor, ¢) None of the above. Question Explanation: Thrombophlebitis is a disease of the deep veins. There is thrombosis most often in the deep veins of the calf or thigh. Usually this disease is unilateral anc presents as calf or thigh pain ion Report An Error 2/24/2014 AM. Mark this question = => Question 17 of 30 A 28 year old sexually active woman taking oral contraceptives complains of pain in her right calf. She smokes one pack per day for 10 years. Ascending contrast venography demonstrates an obstruction of the deep veins of the calf: The most likely diagnosis is a) Strained calf rmscle ) Bilateral leg edema Y © 6) Thrombophlebiis, 4) Metastatic tumor, ¢) None of the above. Question Explanation: Thrombophlebitis is a disease of the deep veins. There is thrombosis most often in the deep veins of the calf or thigh. Usually this disease is unilateral anc presents as calf or thigh pain ion Report An Error 2/24/2014 6:58:03 AM ‘Marke this question & => Question Td : 95750 Question 18 of 30 For which of the following degrees of heart block are temporary pacemakers useful in the management of acute myocardial infarction? al wn om dad e)L 1, and I Question Explanation: Facemakers are useful for types Iand II heart block, but are not indicated for degree [heart block, and may be harmfil to these patients 2/24/2014 6:58:03 AM ‘Marke this question & => Question Td : 95750 Question 18 of 30 For which of the following degrees of heart block are temporary pacemakers useful in the management of acute myocardial infarction? al wr om ¥ © dl and I e)LL, nd I Question Explanation: Facemakers are useful for types Iand II heart block, but are not indicated for degree [heart block, and may be harmfil to these patients 2/24/2014 6:58:16 AM “Mauk this question & => (Question Td: 97343 Question 19 of 30 Arterial hypertension is most associated with cerebrovascular accidents in which one of the following population? a) Males b) Females c) Whites 4) ABican Americans ©) Clildren Question Explanation: ‘Affican Americans have high incidence of siroke in association with hypertension 2/24/2014 6:58:16 AM “Mauk this question & => (Question Td: 97343 Question 19 of 30 Asterial hypertension is most associated with cerebrovascular accidents iz which one of the following population? a) Males ) Females c) Whites Y © 4) Aftican Americans &) Children Question Explanation: ‘Affican Americans have high incidence of siroke in association with hypertension ‘Mark this question <= => Question Id : 109447 Question 20 of 30 ‘A.23 year old Chinese man presents with cyanosis, clubbing, increasing shortness of breath, and palpitations. The most common cause of cyanotic congenital heart diseace in this patient is which one of the following? a) Eisenmenger's syndrome b) Patent ductus arteriosus c) Tewalogy of Falot 4) Atvial septal defect ©) Pulmonary valve stenosis Answer (Bovianation | Other User's Explanation Report An Error Question Explanation: Tetralogy of Fallat consists of ventricular ontflow obstruction and a malaligned nonobstructive ventricular septal defect, with cbligatory overriding ofthe aorta and right ventricular hypertrophy. The cyanosis is caused by hypoxemia and right to left shunting, Eisenmenger's syndrome ie characterized by ventricular septal defect and marked right ventricular hypertrophy from severe pulmonary vascular disease. Ttcaa also cause arrhythmias and cyenosis, butis not that common, Pateat ductus is associated with a {eft to right shunt and is usually asymptomatic. Patients usually have a continous murmur at the left base. Ostium secundum atrial sepial defects are very common in the general population and most people are asymptomatic until they reach the age of 40. No ental prophylanis is required for atrial septal defects. Symptoms of pulmonary valve stenosis depend on the severity of the stenosis. Tuntreated in childhood, adults can decompensate and present with progressive right ventricular failure, This is not a common congenital condition. ‘Mark this question <= => Question Id : 109447 Question 20 of 30 A 23 year old Chinese man presents with cyanosis, chibbing, increasing shortness of breath, and palpitations, The most common cause of cyanotic congenital heart disease in this patient is which one of the following? a) Eisenmenger's syndrome +b) Patent ductus arteriocus Y © ¢)Tewalogy of Fallot 4) Atvial septal defect €) Pulnonary valve stenosis Answer (Bovianation | Other User's Explanation Report An Error Question Explanation: Tetralogy of Fallat consists of ventricular ontflow obstruction and a malaligned nonobstructive ventricular septal defect, with cbligatory overriding ofthe aorta and right ventricular hypertrophy. The cyanosis is caused by hypoxemia and right to left shunting, Eisenmenger's syndrome ie characterized by ventricular septal defect and marked right ventricular hypertrophy from severe pulmonary vascular disease. Ttcaa also cause arrhythmias and cyenosis, butis not that common, Pateat ductus is associated with a {eft to right shunt and is usually asymptomatic. Patients usually have a continous murmur at the left base. Ostium secundum atrial sepial defects are very common in the general population and most people are asymptomatic until they reach the age of 40. No ental prophylanis is required for atrial septal defects. Symptoms of pulmonary valve stenosis depend on the severity of the stenosis. Tuntreated in childhood, adults can decompensate and present with progressive right ventricular failure, This is not a common congenital condition. ‘Mark this question & => Question Id: 114560 Question 21 of 30 ‘Which of the following patients are NOT candidates for cardiac rehabilitation? a) Patients who have undergone coronary bypass surgery. ») Pationts with stable angina pectoris. c) Survivors of myocardial infarction 4) Patients who have undergone coronary angioplasty. ) Patients with known ventricular arrhythmias Question Explanation: Patients with known venizicular arrhythmias are at rick: of sudden death and hemodynamic compromise, Many insurance companies will not pay for rehab programs if veriticular arhythmias are decumented via Holter monitor. Carciac rehablitation programs have been shown to decrease cardiovascular mortality in patients who have undergone bypass surgery. Exercise taining on the treadmill at feast three times weekly improves objective measures of exercise tolerance without compromising cardiovascular mortality. Survivors of myocardial infarction who have congestive heart faiure and join exercise waining programs in cardiac rehabs have improvement of syinptoms of let ventricular dysfunction, A program of strength training improves skeletal muscle strength and endurance in coronary patients who are clinically stable, ‘Mark this question & => Question Id: 114560 Question 21 of 30 ‘Which of the following patients are NOT candidates for cardiac rehabilitation? a) Patients who have undergone coronary bypass surgery. ») Pationts with stable angina pectoris. c) Survivors of myocardial infarction 4) Patients who have undergone coronary angioplasty. JY © ®) Patients with known ventricular arrhythmias Question Explanation: Patients with known venizicular arrhythmias are at rick: of sudden death and hemodynamic compromise, Many insurance companies will not pay for rehab programs if veriticular arhythmias are decumented via Holter monitor. Carciac rehablitation programs have been shown to decrease cardiovascular mortality in patients who have undergone bypass surgery. Exercise taining on the treadmill at feast three times weekly improves objective measures of exercise tolerance without compromising cardiovascular mortality. Survivors of myocardial infarction who have congestive heart faiure and join exercise waining programs in cardiac rehabs have improvement of syinptoms of let ventricular dysfunction, A program of strength training improves skeletal muscle strength and endurance in coronary patients who are clinically stable, ‘Maz this question e& => Question 22 of 30 Question Id : 121097 An ERG shows no correspondence betwesn atrial and veniricular activity. The ECG diagnosis that applies is which one of the following? a) First degree AV block ) Second degree AV block, Mobitz Type I ©) Second degree AV block, Mobitz Type TZ ) Third degree AV block ©) Fourth degree AV block Question Explanation: ‘A third degree AV block would be the ECG that applies in this situation ‘Maz this question e& => Question 22 of 30 Question Id : 121097 An ERG shows no correspondence betwesn atrial and veniricular activity. The ECG diagnosis that applies is which one of the following? a) First degree AV block ) Second degree AV block, Mobitz Type I ©) Second degree AV block, Mobitz Type II VW © A Third degree AV bloc ©) Fourth degree AV block Question Explanation: ‘A third degree AV block would be the ECG that applies in this situation ‘Mark this question a => 129370 Question 23 of 30 A76 year old woman with a past history of stroke becomes confused and lightheaded, Her pulse is 73 beats per min and irregular and her blood pressure is 100/60mmlIg, Her ECG shows atrial fibrilation She takes a diuretic and an aspirin tablet daily. What is the appropriate next step in management? a) Admit for evaluation and treatment. ) Evaluate in the emergency department, c) Contact her physician, 4) Contact her Family. ) Initiate fiuid therapy in the emergency department, Question Explanation: “Although she may subsequently require admission, potentially life threatening problems requiring immnediate therapy (such as severe electrolyte abnormalities) should be assessed firs. It is desirable to contact the physician end family, but these will not affect the need for immediate evaluation. Prescription of appropriate fluid therapy is premature, since it requires quantitative assessment of the patient ‘s blood chernistiies ‘Mark this question <_c> Question 23 of 30 129370 A76 year old woman with a past history of stroke becomes confused and lightheaded, Her pulse is 73 beats per min and irregular and her blood pressure is 100/60mmlIg, Her ECG shows atrial fibrilation She takes a diuretic and an aspirin tablet daily. What is the appropriate next step in managemer? a) Admit for evaluation and treatment © b) Ewaluate in the emergency department, c) Contact her physician, 4) Contact her Family. ) Initiate fiuid therapy in the emergency department, Question Explanation: “Although she may subsequently require admission, potentially life threatening problems requiring immnediate therapy (such as severe electrolyte abnormalities) should be assessed firs. It is desirable to contact the physician end family, but these will not affect the need for immediate evaluation. Prescription of appropriate fluid therapy is premature, since it requires quantitative assessment of the patient “s blood chemistries ‘Marke this question Question Td: 140110 Question 24 of 30 A.55 year old woman presents aiter several episodes of syncope. A low pitched "plopping" sound during mid-systole is auscultated. ‘Two dimensional echocardiography shows an intermittent obstruction of the mitral valve. What would most be observed if the cause ofthe obstuction were biopsied? a) Benign glandular tissue b) Densely packed smocth muscle c) Densely packed striated muscle 4) Malignant glancular tissue ©) Scattered meseachymal cells in myxoid background Answer (Exanation | Other User's Explanation Report An Error Question Explanation: Atrial myxoma is the most common primary aduit cardiac tumor, which typicslly occurs as a single lesion in the left; atrinm that may intermitiently obstruct the mitral valve. Iris a benign mesenchymal tumor, histologically, these tumors are composed of scattered mesenchymal cells in a prominent myxoid background, Atrial myzomas in the left atrium can produce a ball valve effect that can block the mitral valve orifice end block diastolic filling of the left ventricle, thereby stimulating mitral valve stenosis. Benign glandular tissus suggests an adenoma which is not usually found in the heart. Densely packed smooth muscle suggests at Iciomyoma, also known as fibroid, most, commonly found in the uterus, Densely packed sitiated muscle suggests rhebdomyoma, whichis the most common primary cardiac tumor in children, not adults. Malignant glandular tissue suggests carcinoma, which can be metastatic to the hear: but does not usually cause a ball valve obstruction. ‘Marke this question Question Td: 140110 Question 24 of 30 4.55 year old woman presents after several episodes of syncope. A low pitched "plopping" sound durng mid-systole is auscultated. ‘Two dimensional echocardiography shows an intermittent obstruction of the mitral valve, What would most be observed ifthe cause of the obstruction were biepsiad? a) Benign glandular tissue b) Densely packed smocth muscle c) Densely packed striated muscle 4) Malignant glandular tissue Y © ©) Scattered mesenchymal cells in myxoid background Answer (Exanation | Other User's Explanation Report An Error Question Explanation: Atrial myxoma is the most common primary aduit cardiac tumor, which typicslly occurs as a single lesion in the left; atrinm that may intermitiently obstruct the mitral valve. Iris a benign mesenchymal tumor, histologically, these tumors are composed of scattered mesenchymal cells in a prominent myxoid background, Atrial myzomas in the left atrium can produce a ball valve effect that can block the mitral valve orifice end block diastolic filling of the left ventricle, thereby stimulating mitral valve stenosis. Benign glandular tissus suggests an adenoma which is not usually found in the heart. Densely packed smooth muscle suggests at Iciomyoma, also known as fibroid, most, commonly found in the uterus, Densely packed sitiated muscle suggests rhebdomyoma, whichis the most common primary cardiac tumor in children, not adults. Malignant glandular tissue suggests carcinoma, which can be metastatic to the hear: but does not usually cause a ball valve obstruction. ‘Mark this question eq => Question Id: 140322 Question 25 of 30 A. 66 year old man is broughtto the ER with chest pain and fever of 102°F (38.9°C) for the past 2 days. He was recently admited to the hospital for 2 weeks because of severe chest pan and was discharged on aspirin, ACE inhibitor, and a beta blocker. The most likely cause of his cureat presentation is a) Caseous Pericarditis +b) Fibrinous pericarditis, c) Hemorrhagic pericardtis 4) Punilent pericarditis €) Serous pericarditis Answer [UERINUIY oner Users Explanation Report An Evo Question Explanation: ‘The key to this question is to identify that this patient recently had a myocardial infarction (MI), as suggested by his admission following severe chest pain. Another clue is the initiation of cardiac medications typical for patients with recent MI. This man has presented with a common complication of myocardial infarction: acute pericarditis. Fibrincus and serofbrinous pericarditis (Dressler smérome) develops 2 to 10 weeks after an MI or heart surgery. It presents with a low grade fever, pleurttc chest pain that changes with respiration and bodily position percardial ftiction mb, and sometimes pericardial effusion. Treatment inciudes nonsteroidal anti- inflammatories. Remember that Drecsler eyndeome is a pericarditis of autoimamune origia that develops several weeks (8 to 10) after infarcticn. Cateous pericarditis ic generally due to tuberculosie. Hemorthagic pericarditie can be seen with tuberculecie with malignant tumors, in paticnts with bleeding diatheses, and following chest surgery. Purulent pericardis is seen when pyogenic infections involve the pericardium, «.g. afler cardiothoracic surgery. Serous pericarditis is seen in non-infectious inflammations (cheumatic fever lupus, scleroderma, mmors uremia). ‘Mark this question eq => Question Id: 140322 Question 25 of 30 A.66 year old man is brought to the ER with chest pain and fever of 102°F (38.9°C) for the past 2 days. He was receatly admitted to the hospital for 2 weeks because of severe chest pain and was discharged on aspirin, ACE inhibitor, and a beta blocker. The most likely cause of his current presentation is a) Caseous Pericarditis Y © b) Fibrinous pericarditis c) Hemorrhagic pericardtis 4) Punlent pericarditis €) Serous pericarditis Answer [UERINUIY oner Users Explanation Report An Evo Question Explanation: ‘The key to this question is to identify that this patient recently had a myocardial infarction (MI), as suggested by his admission following severe chest pain. Another clue is the initiation of cardiac medications typical for patients with recent MI. This man has presented with a common complication of myocardial infarction: acute pericarditis. Fibrincus and serofbrinous pericarditis (Dressler smérome) develops 2 to 10 weeks after an MI or heart surgery. It presents with a low grade fever, pleurttc chest pain that changes with respiration and bodily position percardial ftiction mb, and sometimes pericardial effusion. Treatment inciudes nonsteroidal anti- inflammatories. Remember that Drecsler eyndeome is a pericarditis of autoimamune origia that develops several weeks (8 to 10) after infarcticn. Cateous pericarditis ic generally due to tuberculosie. Hemorthagic pericarditie can be seen with tuberculecie with malignant tumors, in paticnts with bleeding diatheses, and following chest surgery. Purulent pericardis is seen when pyogenic infections involve the pericardium, «.g. afler cardiothoracic surgery. Serous pericarditis is seen in non-infectious inflammations (cheumatic fever lupus, scleroderma, mmors uremia). 2/24/2014 6:59:58 AM ‘Mark this question q => Question Td : 140332 Question 26 of 30 A.62 year old man with a history of arrhythmia is found dead at home. His heart at autopsy showed a large chimp of adherent red ‘material is seen that has thin white laminations composed of platelets. Tae most likely material is a) Atherosclerotic plaque bb) Infectious endocarditis vegetation c) Marantic vegetation 4) Postmortem thrombus €) Premortem thrombus Question Explanation: ‘The material ilustrated is a Premortem (before death) thrombus, To identify this as Premortem, look for the lines of Zahn (thin white laminations composed mostly of plateleis). The patient probably had atrial fibrillation prior to death: this causes a diated, nonfunctional atrium in which clots can form, Atherosclerotic plaques do not usually form in the cardiac chambers and are composed. ofa cellalar material with froblasts and a superficial enclothelinm visible on microscopy. The vegetations of infectious endocarditis usualy invelve the valves, but can involve the endocardium ofthe cardiac chambers, These vegetations form fable white lesions containing many neutrophils. A marantic vegetation is a small, noninfectious, acelular vegetation found on cardiac valve leaflets, often along the line of closure A postmortem thrombus would not contain lines of Zahn, as it does not form over a period of time. 2/24/2014 6:59:58 AM ‘Mark this question q => Question Td : 140332 Question 26 of 30 A.62 year old man with a history of arrhythmia is found dead at home. His heart at autopsy showed a large chimp of adherent red ‘material is seen that has thin white laminations composed of platelets. Tae most likely material is a) Atherosclerotic plague ') Infectious endocarditis vegetation c) Marantic vegetation 4) Postmortem thrombus ¥ © €) Premortem thrombus Question Explanation: ‘The material ilustrated is a Premortem (before death) thrombus, To identify this as Premortem, look for the lines of Zahn (thin white laminations composed mostly of plateleis). The patient probably had atrial fibrillation prior to death: this causes a diated, nonfunctional atrium in which clots can form, Atherosclerotic plaques do not usually form in the cardiac chambers and are composed. ofa cellalar material with froblasts and a superficial enclothelinm visible on microscopy. The vegetations of infectious endocarditis usualy invelve the valves, but can involve the endocardium ofthe cardiac chambers, These vegetations form fable white lesions containing many neutrophils. A marantic vegetation is a small, noninfectious, acelular vegetation found on cardiac valve leaflets, often along the line of closure A postmortem thrombus would not contain lines of Zahn, as it does not form over a period of time. 2/24/2014 7:00:16 AM ‘Mark this question = => ‘Question Td : 174266 Question 27 of 30 EKG of a61 year old man reveals QRS intervals of 0.14 seconds with distinctly abnomal configurations. Exam is sigificant for paradoxical spliting of the second heart sound. The conduction defects that is likely inthis patient is a) Complete AV block 1b) First-degree AV block ©) Mobitz Type LAY block 4) Mobitz Type IL AV block ¢) Wolf Parkinson-White syndrome Question Explanation: This patient has bundle branch block, as implied by the QRS interval greeter than 0.12 seconds and by paradoxical splitting of the second heart sound Mobitz Tl block is frequentiy associated with bundle branch block. Mobitz I block is characterized on EKG by a constant PR interval before failure of AY conduction occurs. The anatomic site of this type of block is usually below the AV node Complete AV block represents the failure of any impulses to be conducted from the airia to the ventricles. The ventricles are depolarized by an AV nodal or vertricular escape thythm. Itis manifested by a slow ventricular rate, wide pulse pressure, a variable first heart sound, and prominent jugular venous pulsations. First degree AV block represents a delay in conduction of the impulse from the atria to the ventricles, due most commonly to abnormalities in the AV nods, Ibis reflected by a prolonged PR interval, usually exceeding 0.20 seconds. Mobitz Type TAY block (Wenckebach) is usually due to a problem in the AV node and can result from a variety of cardiac or systemic disorders, inciuding myocerdial infarction, Wolff Parkinson White syndrome is also known as pte-excitation syndrome because conduction occurs by way of an ausliary pathway between the atria and ventricles. Findings on EKG include a short FR interval and a delta wave (slurred QRS upstroke), 2/24/2014 7:00:16 AM ‘Mark this question = => ‘Question Td : 174266 Question 27 of 30 EKG ofa 61 year old man reveals QRS intervals of 0.14 seconds with distinctly abnormal configurations. Exam is significant for paradoxical splitting of the second heast sound. The conduction defects that is likely in this patient is a) Complete AV block b) First-degree AV block ) Mobitz Type LAY block Y © d) Mobitz Type I AV block ¢) Wolf-Parkinson-White syndrome Question Explanation: This patient has bundle branch block, as implied by the QRS interval greeter than 0.12 seconds and by paradoxical splitting of the second heart sound Mobitz Tl block is frequentiy associated with bundle branch block. Mobitz I block is characterized on EKG by a constant PR interval before failure of AY conduction occurs. The anatomic site of this type of block is usually below the AV node Complete AV block represents the failure of any impulses to be conducted from the airia to the ventricles. The ventricles are depolarized by an AV nodal or vertricular escape thythm. Itis manifested by a slow ventricular rate, wide pulse pressure, a variable first heart sound, and prominent jugular venous pulsations. First degree AV block represents a delay in conduction of the impulse from the atria to the ventricles, due most commonly to abnormalities in the AV nods, Ibis reflected by a prolonged PR interval, usually exceeding 0.20 seconds. Mobitz Type TAY block (Wenckebach) is usually due to a problem in the AV node and can result from a variety of cardiac or systemic disorders, inciuding myocerdial infarction, Wolff Parkinson White syndrome is also known as pte-excitation syndrome because conduction occurs by way of an ausliary pathway between the atria and ventricles. Findings on EKG include a short FR interval and a delta wave (slurred QRS upstroke), ‘Mat this question & => Question Td: 178523 Question 28 of 30 A.45 year old woman has sharp, stabbing pain in her chest for the past 12 hours. She had MI 3 years ago. She refuses to lie down during exam and leens forward, stating that it allows her to breathe more easily. ECG reveals diffuse ST segment elevations with upright T waves. CXR is normal. CK-MB is normal. What is the likely diagnosis? a) Acute pericardtis +b) Disseoting aortic aneurysm c) Myocardial infarction 4) Stable angina ©) Unsteble angina Answer | Explanation Other User's Explanation Report An Error Question Explanation: ‘All of the answer choices reprecent common cardiovascular causes of chest pain. However, only pericarditis and dissecting aortic ancurisms wil produce sharp, life like pains. Patients wath pericarditis relieve their pain by sitting and leaning forward. The characteristic ECG patterns of pericarditis include diffiase ST clevation with upright T waves. Although a pericardial rub is diggnostis of pericarditis, its presence is not necessary for diagnosis, and the physical examination may well be unrevealing Typically, in uncomplicated pericarditis, both chest radiographs and cardiac isoenzyme levels appear normal, Pericarditis can be differentiaved from dissecting 2ortic aneurysms on the basis of clinical fndings. The pain associated with dissecting aortic aneurysins is usually unrelated to breathing, whereas the pain associated with pericarditis is related to breathing, Myocardial infarcis, as well as stable and unstable angina, typically produce more visceral types of pan. ‘Mat this question & => Question Td: 178523 Question 28 of 30 4.45 year old woman has sharp, stabbing pain in her chest for the past 12 hours. She had MI 3 years ago. She refuses to ie down during exam and leans forward, stating that it allows her to breathe more easily. ECG reveals diffuse ST segment elevatiors with upright T waves. CXCRis normal. CK-MB is normal, What's the lcely diagnosis? Y © a) Acute pericarditis 'b) Dissecting aortic aneurysm c) Myocardial infarction 4) Stable angina €) Urstable angina Answer | Explanation Other User's Explanation Report An Error Question Explanation: ‘All of the answer choices reprecent common cardiovascular causes of chest pain. However, only pericarditis and dissecting aortic ancurisms wil produce sharp, life like pains. Patients wath pericarditis relieve their pain by sitting and leaning forward. The characteristic ECG patterns of pericarditis include diffiase ST clevation with upright T waves. Although a pericardial rub is diggnostis of pericarditis, its presence is not necessary for diagnosis, and the physical examination may well be unrevealing Typically, in uncomplicated pericarditis, both chest radiographs and cardiac isoenzyme levels appear normal, Pericarditis can be differentiaved from dissecting 2ortic aneurysms on the basis of clinical fndings. The pain associated with dissecting aortic aneurysins is usually unrelated to breathing, whereas the pain associated with pericarditis is related to breathing, Myocardial infarcis, as well as stable and unstable angina, typically produce more visceral types of pan. 2/24/2014 7:00:46 AM ‘Mark this question & => Question Id : 181144 Question 29 of 30 Ina car accident a female's chest was hit by the steering wheel In the ER her blood pressure is 120/90 mmHg Whea she inhales, her systolic blood pressure drops to 100 mun Hg, This finding defines which one of the following termns? a) Pulsus alternans ') Pulsus bisferiens ©) Pulsus paradoxus 4) Pulsus parvus ©) Pulsus tardus Question Explanation: Pulsus paradomis is defined as a fall in systolic blood pressure > 10 mm Hg on inspiration. Tt can be associated with cardiac tamponade and chronic obstructive pulmonary disease (COPD). Pulsus alternans is a repeated variation in the amplitude of the pulse pressure, It can be associated with profound left ventricular dysfunction, Pulsus bisfenens is a double pulsation occurring during systole. It can be associated with aortic regurgtation and hypertrophic carciomyopathy. Pulsus parvus is a weak pulse upstroke caused by diminished stroke volume. It can be associated with hypovolemia, aortic stenosis, mitral stenosis, and left ventricular falure, Pulsus tardusis a delayed pulse upstroke. It can be associated with aortic stenosis 2/24/2014 7:00:46 AM ‘Marc this question & => Question Id : 181144 Question 29 of 30 Tna car accident a female‘s chest was hit by the steering wheel In the ER her blood pressure is 120/20 mm Hg When she inhales, her systolic blood pressure drops to 100 mm Hg, This finding defines which one of the following terms? a) Pulsus alternans 'b) Pulsus bisferiens ¥ © 6) Pulsus paradoxus d) Pulsus parvus e) Pulsus tardus Question Explanation: Pulsus paradomis is defined as a fall in systolic blood pressure > 10 mm Hg on inspiration. Tt can be associated with cardiac tamponade and chronic obstructive pulmonary disease (COPD). Pulsus alternans is a repeated variation in the amplitude of the pulse pressure, It can be associated with profound left ventricular dysfunction, Pulsus bisfenens is a double pulsation occurring during systole. It can be associated with aortic regurgtation and hypertrophic carciomyopathy. Pulsus parvus is a weak pulse upstroke caused by diminished stroke volume. It can be associated with hypovolemia, aortic stenosis, mitral stenosis, and left ventricular falure, Pulsus tardusis a delayed pulse upstroke. It can be associated with aortic stenosis Mark this question —& Question Td : 216812 Question 30 of 30 A 42-year-old man has slow-tising pulse without increased TYP. Apex beat is at fth intercostal space, midazillary line along a systelic thrill A harsh mideystolis murmuris heard best at the 2nd right intercostal space, radiating to the carotids that decreases in intensity on Valsalva maneuver and increases on squatting, Whatis the most common presentation associated with this disorder? a) Angina b) Dyspnea c) Light-headedness 4) Palpitations ©) Panic attacks £) Sudden death 2) Pulsating liver Question Explanation: ‘This patient hac aortio stenosic, ac indicated by the ejection cystolic raurmur heard bect at the 2nd sight intercostal space and radiating to the carotids, the slow-rising pulse, the systolic thril, and the left ventricular hypertrophy. This patient probably hes developed sonic stenosis from a congenital bicuspid valve (due to his age). The nmrmaur of aortic stenosis is very similar to the murmur of hypertrophic obstructive cardiomyopathy (HOCM). The murmur of aortic sienosis gets better with preload reduction (Valsalva maneuver) ‘because there is less blood going through the valve, and gets worse with an increase in preload because there is more blood going through the stenotic valve end thus more turbulence and a greater intensity of the murmur. The opposite is tue of HOCM. Preload reduction makes the murmur of HOCM worse and aa increase in preload makes the murmur better. Dyspnea on exertion is the most common presenting symptom of patients with aortic stenosis ‘As the stenosis worsens, angina syncopal episodes and heart failure become a more common forms of presentation. This is because the stenotic valve is obstructing the outflow through the coronary miseries during diastole. Also, the increased pressure that must be generated to overcome the stenosis increases the cardiac workload, thus increasing the demand for oxygen during a time when blood supply is not optimal. This results in cardiac ischemia, which manifests itself as chest pain. Once symptomatology develops, the survival rate without valve replacement is less than 5 years. This is due to a high incidence of sudden death attributed to myocardial ischemia and arrhythmias, 15 to 20%. Valvular replacement is indicated as soon as possible in all patients, Light-headedness is not the most common presentation for aortic stenosis. Palpitations are not the most common presentation for aortic stenosis Panic attacks are notreally associated with aortic stenosis Pulsatirg liver ie seen in TR. Mark this question —& Question Td : 216812 Question 30 of 30 A 42-year-old man has slow-tising pulse without increased TYP. Apex beat is at fth intercostal space, midazillary line along a systelic thrill A harsh mideystolis murmuris heard best at the 2nd right intercostal space, radiating to the carotids that decreases in intensity on Valsalva maneuver and increases on squatting, Whatis the most common presentation associated with this disorder? a) Angina Y © b)Dyspnea c) Light-headedness 4) Palpitations ©) Panic attacks £) Sudden death 2) Pulsating liver Question Explanation: ‘This patient hac aortio stenosic, ac indicated by the ejection cystolic raurmur heard bect at the 2nd sight intercostal space and radiating to the carotids, the slow-rising pulse, the systolic thril, and the left ventricular hypertrophy. This patient probably hes developed sonic stenosis from a congenital bicuspid valve (due to his age). The nmrmaur of aortic stenosis is very similar to the murmur of hypertrophic obstructive cardiomyopathy (HOCM). The murmur of aortic sienosis gets better with preload reduction (Valsalva maneuver) ‘because there is less blood going through the valve, and gets worse with an increase in preload because there is more blood going through the stenotic valve end thus more turbulence and a greater intensity of the murmur. The opposite is tue of HOCM. Preload reduction makes the murmur of HOCM worse and aa increase in preload makes the murmur better. Dyspnea on exertion is the most common presenting symptom of patients with aortic stenosis ‘As the stenosis worsens, angina syncopal episodes and heart failure become a more common forms of presentation. This is because the stenotic valve is obstructing the outflow through the coronary miseries during diastole. Also, the increased pressure that must be generated to overcome the stenosis increases the cardiac workload, thus increasing the demand for oxygen during a time when blood supply is not optimal. This results in cardiac ischemia, which manifests itself as chest pain. Once symptomatology develops, the survival rate without valve replacement is less than 5 years. This is due to a high incidence of sudden death attributed to myocardial ischemia and arrhythmias, 15 to 20%. Valvular replacement is indicated as soon as possible in all patients, Light-headedness is not the most common presentation for aortic stenosis. Palpitations are not the most common presentation for aortic stenosis Panic attacks are notreally associated with aortic stenosis Pulsatirg liver ie seen in TR. 2242018 AN “Mak this question Question 1 of 30 A 35-year-old male presents with episodes of breathlessness on exertion Examination reveals aloud P2 and fixed splitting of the second sound, Which of the following may be responsible for these signs? a) 47 XXY karyotype b) Homocystinuria ©) Matemal chicken poxinfection 4) Excess maternal alcohol coxsuaption ©) Maternal thalidomide therapy Question Explanation: Fetal alcohol syndrome, Down syncrome anc congenital rubella syndrome are associated with an atrial septal defect (ASD) with a loud second sound plus fixed spltting, as described in this case. 2242018 AN “Mak this question Question 1 of 30 A 35-year-old male presents with episodes of breathlessness on exertion Examination reveals aloud P2 and fixed splitting of the second sound, Which of the following may be responsible for these signs? a) 47 XXY karyotype b) Homocystinuria ©) Matemal chicken poxinfection Y © 4) Excess matemal alechol consumption ©) Maternal thalidomide therapy Question Explanation: Fetal alcohol syndrome, Down syncrome anc congenital rubella syndrome are associated with an atrial septal defect (ASD) with a loud second sound plus fixed spltting, as described in this case. ‘Mack this question = => Question Ta : 22498 Question 2 of 30 A 60 year old black male was recently diagnosed with an abdominal aortic aneurystn. A lipid profile performed a few months ago revealed an LDL level of 125 mg/dL. You would now advise tin that his goal LDL level should now be? a) < 130 mg/dL. b)< 150 mf ) < 100 mg/dl. od) < 160 mg/dL. e) < 180mg/dL. Anewer (UBRRARY) othe: teers Explanation Report An Enor Question Explanation: ‘Most physicians realize that the goal LDL level for patients with diabetes melitus or coronary artery disease is < 100 mg/dL. Many may not realize that this goal extends to people with CAD-equvalent diseases, including peripheral artery disease, symptomatic carotid artery disease, and abdominal aortic aneurysm, ‘Mack this question = => Question Ta : 22498 Question 2 of 30 A 60 year old black male was recently diagnosed with an abdominal aortic aneurystn. A lipid profile performed a few months ago revealed an LDL level of 125 mg/dL. You would now advise tin that his goal LDL level should now be? a) < 130 mg/dL. b)< 150 mf Y © 0) < 100 mela. od) < 160 mg/dL. e) < 180mg/dL. Anewer (UBRRARY) othe: teers Explanation Report An Enor Question Explanation: ‘Most physicians realize that the goal LDL level for patients with diabetes melitus or coronary artery disease is < 100 mg/dL. Many may not realize that this goal extends to people with CAD-equvalent diseases, including peripheral artery disease, symptomatic carotid artery disease, and abdominal aortic aneurysm, 2/24/2014 7:10:24 AM ‘Mark this question & => Question Id : 23904 Question 3 of 30 Out of the following, which finding is not consistent with cardiac tamponade? a) Pulsus paradoxus b) Tachycardia c) Jugular venous distension (TVD) 4) Distant heart sounds e) Eussmaul sign (Question Explanation: Cardiac tamponade is the compression of the heart caused by blood of fluid accumulation in the space berween the myocardium (the muscle of the heart) and the pericardium (the outer covering sac of the heart). Beck’s triad is a collection of three medical signs associated with acute cardiac tamponade, an emergeacy condition wherein duid accumulates around the heart and impairs its ability to pump blood, ‘The result is the trad oflow arterial blood pressure, increased central venous pressure (eadngto JVD), and distant heart sounds. 2/24/2014 7:10:24 AM ‘Mark this question & => Question Id : 23904 Question 3 of 30 Out of the following, which finding is not consistent with carvliac tamponade? a) Pulsus paradosus b) Tachycardia c) Jugular venous distension (TVD) 4) Distant heart sounds Y © e) Kussmaul sign (Question Explanation: Cardiac tamponade is the compression of the heart caused by blood of fluid accumulation in the space berween the myocardium (the muscle of the heart) and the pericardium (the outer covering sac of the heart). Beck’s triad is a collection of three medical signs associated with acute cardiac tamponade, an emergeacy condition wherein duid accumulates around the heart and impairs its ability to pump blood, ‘The result is the trad oflow arterial blood pressure, increased central venous pressure (eadngto JVD), and distant heart sounds. 2/24/2014 7:10:38 AM ‘Marke this question & => Question Td: 46668 Question 4 of 30 A.70 year old person comes to you for assessment one week afier experiencing a brief episode of left arm and left facial weekness His blood pressure is 140/80 mmHg. A CT scan showed no evidence of hemorthage or space occupyingles.on. An EKG shows normal sins rhythm. The most important investigation to order at this time is a) A magnetic resonance imaging (MRD of the brain 'b) An echocardiogram c) Carotid artery ultrasound 4) Cerebral angiography ©) Chest x-ray Answer (Explanation) Other User's Explanation Report An Error Question Explanation: ‘A transient ischemic attack (TIA) is focal brain ischemia producing sudden neurologic deficits that last < 1 hour. Most TIAs are caused by emboli, usuelly from carctid or vertebral arteries, although most of the causes of ischemic stroke can also result in TAS: The cause ofa TLA is sought as for that of ischemic strokes, including tests for carotid stenosis via a carotid ultrasound, cardiac sources of emboli, ariel fibrillation, and hematologic abnonnelities and screering for stroke risk factors. Because risk of subsequent ischemic stroke is high and immediate, evaluation proceeds rapidly, usually on an inpatient basis. Iris not clear which patients, ifany, can be safely discharged from the emergency department. 2/24/2014 7:10:38 AM ‘Marke this question & => Question Td: 46668 Question 4 of 30 A.70 year old person comes to you for assessment one week afier experiencing a brief episode of left arm and left facial weekness His blood pressure is 140/80 mmHg. A CT scan showed no evidence of hemorthage or space occupyingles.on. An EKG shows normal sins rhythm. The most important investigation to order at this time is a) A magnetic resonance imaging (MRD) of the brain 'b) An echocardiogram Y © c) Carotid artery ultrasound 4) Cerebral angiography ©) Chest x-ray Answer (Explanation) Other User's Explanation Report An Error Question Explanation: ‘A transient ischemic attack (TIA) is focal brain ischemia producing sudden neurologic deficits that last < 1 hour. Most TIAs are caused by emboli, usuelly from carctid or vertebral arteries, although most of the causes of ischemic stroke can also result in TAS: The cause ofa TLA is sought as for that of ischemic strokes, including tests for carotid stenosis via a carotid ultrasound, cardiac sources of emboli, ariel fibrillation, and hematologic abnonnelities and screering for stroke risk factors. Because risk of subsequent ischemic stroke is high and immediate, evaluation proceeds rapidly, usually on an inpatient basis. Iris not clear which patients, ifany, can be safely discharged from the emergency department. 272472014 7:10:56 AM ‘Marke this question <=> Question Ta : 51347 Question 5 of 30 AAG year old fernale with a history of deep venous thrombosis is taking warfarin (Coumadin), 10 mg once daily, and is maintain an Intemational Normalized Ratio (MTR) of 275. She stared taking high doses of garlic and would like to know if she can continue to take it, She hes blood in her nares and mutiple bruises on her exsremities that began after she started taking the garlic. The correct statement is a) She can continue to take the garlic because her INR is appropriate ') Interaction with gevlic should be suspected despite an appropriate INR ©) Garlic in combination with warfarin poses serious risk of cerebral hemorthage and should be stopped immediately 4) Gatlic does not pose a risk of drug interctions and she can continue taking it Question Explanation: Gare is thought 1o provide several cardiovascular benefits, such as blood pressure lowering, serum lipid lowering, and antithrombotic activity. Garlic oil has been reported to interrupt thromboxanes synthesis, thereby inhibiting platelet Aanction, There have been reports, of spontaneous epidaral hematoma after ingesting approximately 2000 mg of gartic daily (equivalent to about four cloves) for an undetermined petiod, The available information suggests that a serious interaction is possible, Patients taking warfarin should be advised to avoid garlic supplements. However, they should also be aware thet regular ingestion of food products containing small amounts of garlic should not pose a problem. If excessive garlic consumption and warfarin use occur concomitantly, the patient’s INK should be closely monitored, 272472014 7:10:56 AM ‘Marke this question <=> Question Ta : 51347 Question 5 of 30 AAG year old fernale with a history of deep venous thrombosis is taking warfarin (Coumadin), 10 mg once daily, and is maintain an Intemational Normalized Ratio (MTR) of 275. She stared taking high doses of garlic and would like to know if she can continue to take it, She hes blood in her nares and mutiple bruises on her exsremities that began after she started taking the garlic. The correct statement is a) She can continue to take the garlic because her INR is appropriate ') Interaction with gevlic should be suspected despite an appropriate INR Y © ©) Garlic in combination with warfarin poses serious tisk of cerebral hemorthage and should be stopped immediately 4) Gatlic does not pose a risk of drug interctions and she can continue taking it Question Explanation: Gare is thought 1o provide several cardiovascular benefits, such as blood pressure lowering, serum lipid lowering, and antithrombotic activity. Garlic oil has been reported to interrupt thromboxanes synthesis, thereby inhibiting platelet Aanction, There have been reports, of spontaneous epidaral hematoma after ingesting approximately 2000 mg of gartic daily (equivalent to about four cloves) for an undetermined petiod, The available information suggests that a serious interaction is possible, Patients taking warfarin should be advised to avoid garlic supplements. However, they should also be aware thet regular ingestion of food products containing small amounts of garlic should not pose a problem. If excessive garlic consumption and warfarin use occur concomitantly, the patient’s INK should be closely monitored, 2/24/2014 7: 7 AM ‘Mark tis question ez ‘Question Td = 51589 Question 6 of 30 A 22 year old student comes to the student health center because of marked fatigue, Temperature is 28.3°C (101.0°F). Physical examination shows striking pallor of skin, nail beds and conjunctivae. There are petechial hemorrhages in the skin of his legs. A soft, blowing systole murmur is present over the precordium, Mo other abnormalities ate present, Whatis the most eppropriate investigation at this point? a) Complete blood count ) Determination of bleeding and clotting time ©) Examination of bone marrow aspirate ) Hemoglobin electrophoresis €) Serological testing for infectious mononucleosis (Question Explanation: The description of this patient incides a number of signs and symptoms consistent with anemia: ie, marked fatigue, striking pallor, and soft blowing systolic murmur (flow imum). Petechial hemorthages may suggest a platelet disorder or vasculitis. The most inp ortant first study at this time would be a CBC, which would reveal his hemetocrit as well as his platelet count. 2/24/2014 7: 7 AM ‘Mark tis question ez ‘Question Td = 51589 Question 6 of 30 A 22 ycar old student comes to the student health center because of marked fatigue. Temperature is 38.3°C (101.0°F). Physical examination shows striking pallor of skin, nail beds and conjunctivae. There are petechial hemorrhages in the skin of his legs. A soft, blowing systolic murmur is present over the precordium. No other abnormalities are present. Whatis the most appropriate investigation at this point? Y © a) Complete blood count 'b) Determination of bleeding and clotting time c) Examination of bone marrow aspirate d) Hemoglobin electrophoresis e) Serological testing for infectious mononucleosis (Question Explanation: The description of this patient incides a number of signs and symptoms consistent with anemia: ie, marked fatigue, striking pallor, and soft blowing systolic murmur (flow imum). Petechial hemorthages may suggest a platelet disorder or vasculitis. The most inp ortant first study at this time would be a CBC, which would reveal his hemetocrit as well as his platelet count. ‘Mark this question & => (Question Td : 51739 Question 7 of 30 Preferred medication for chronic treatment of congestive heart failure due to let ventricular systolic dysfimction is a) Dinvetics b) Digoxin c) Calcium channel blockers 4) ACE inhibitors ©) Hydralarine (Apresoline) pulse isoserbide dinitrate (sordil, osbitrate) Answer | Beanation Other User's Explanation Report An Error Question Explanation: ACE inhibitors are the prefirred drugs for congestive heart failure due to left ventricular systolic dysfunction, because they arc associated with the lower mortality. The combination of hydralazine/isosorbide dinitrate is a seasonable akernative, and diuretics should be used cautiously. Ibis not known whether digoxin affects mortalty, although it can help with symptoms ‘Mark this question & => (Question Td : 51739 Question 7 of 30 Preferred medication for chronic treatment of congestive heart failure due to let ventricular systolic dysfimction is a) Dinvetics b) Digoxin c) Calcium channel blockers Y © 4) ACE inhibitors ©) Hydralarine (Apresoline) pulse isoserbide dinitrate (sordil, osbitrate) Answer | Beanation Other User's Explanation Report An Error Question Explanation: ACE inhibitors are the prefirred drugs for congestive heart failure due to left ventricular systolic dysfunction, because they arc associated with the lower mortality. The combination of hydralazine/isosorbide dinitrate is a seasonable akernative, and diuretics should be used cautiously. Ibis not known whether digoxin affects mortalty, although it can help with symptoms 2/24/2014 7:11:33 AN ‘Mat this question & => Question Id : 55522 Question 8 of 30 A contraindication to the use of Beta-blockers for congestive heart failure is a) Mild asthma +b) Symptomatic heart biock c) New York Heart Association (NYHA) class II heart failure 4) NYHA Clas I heart faire in a patient with ahietory of a previous myocardial infarction ©) An ejection fraction <30% Question Explanation: According to several randomized, controlled trials, mortality rates are improved in patients with heart failure, who receive beta- blockers in addition to diuretics, ACE inhibitors, and occasionelly, digoxin, Contraindications to Beta-blockeer use include hemodynamic instability, heast block, bradycardia, and severe asthma, Beta-blockers may be tried in patients with mild asthma or COPD as long as they are monitored for potential exacerbations. Beta-blocker use has been shown to be effective in pationts with NYHA Class Il or IT heart failure. There is no absolute threshold ejection fraction, Beta-blockers have also been shown to decrease morvalty in patients with a previous history of myocardial infarction, regardless of their N'YELA. classification, 2/24/2014 7:11:33 AN ‘Mat this question & => Question Id : 55522 Question 8 of 30 A contraindication to the use of Beta-blockers for congestive heart failure is a) Mild asthma © b) Symptomatic heart block c) New York Heart Association (NYHA) class II heart failure 4) NYHA Clas I heart faire in a patient with ahietory of a previous myocardial infarction ©) An ejection fraction <30% Question Explanation: According to several randomized, controlled trials, mortality rates are improved in patients with heart failure, who receive beta- blockers in addition to diuretics, ACE inhibitors, and occasionelly, digoxin, Contraindications to Beta-blockeer use include hemodynamic instability, heast block, bradycardia, and severe asthma, Beta-blockers may be tried in patients with mild asthma or COPD as long as they are monitored for potential exacerbations. Beta-blocker use has been shown to be effective in pationts with NYHA Class Il or IT heart failure. There is no absolute threshold ejection fraction, Beta-blockers have also been shown to decrease morvalty in patients with a previous history of myocardial infarction, regardless of their N'YELA. classification, 2/24/2014 7:11:45 AM ‘Matte this question & => Question 9 of 30 Fora 50 year old sedentary, normotensive, non-smoking white male who isnot overweight, drinks 60 mL of whiskey per day and plays golf occasionally, the condition is most likely to cause death within the next 10 years is a) Motor vehicle accident b) Cerebrovascular disease ©) Suicide 4d) Iechemic heart disease €) Citthosis of the liver Question Explanation: Tamale over the age of 50 with low risk factors, the most common cause of morbidity is due to ischemic heart disease. Ischemic heart disease (HD), or myocardial ischemia, is a disease characterized by reduced blood supply to the heart muscle, usually clue to coronary artery disease (atherosclerosis of the coronary arteries). Its risk increases with age, stoking, hypercholesterolemia (high cholesterol levels), clabetes, hypertension (high blood pressure) and is more common in men and those who have close relatives with ischemic heart disease Tis the most common cause of death inmost Westem countries end a major cause of hospital admissions. There is limited evidence for population screening, but prevention (with a healthy diet and sometimes medication for diabetes, cholesterol and high blood pressure is used both to prevent THT) and to decrease the risk of compilations. 2/24/2014 7:11:45 AM ‘Matte this question & => Question 9 of 30 For a 50 year old sedentary, normotensive, non-smoking white male who is not overweight, drinks 60 ml. of whiskey per day and plays golf occasionally, the condition is most likely to cause death within the next 10 years is a) Motor vehicle accident ) Cerebrovascular disease ©) Suicide Y © d) Ischemic heart disease ©) Citrhosis of the liver Question Explanation: Tamale over the age of 50 with low risk factors, the most common cause of morbidity is due to ischemic heart disease. Ischemic heart disease (HD), or myocardial ischemia, is a disease characterized by reduced blood supply to the heart muscle, usually clue to coronary artery disease (atherosclerosis of the coronary arteries). Its risk increases with age, stoking, hypercholesterolemia (high cholesterol levels), clabetes, hypertension (high blood pressure) and is more common in men and those who have close relatives with ischemic heart disease Tis the most common cause of death inmost Westem countries end a major cause of hospital admissions. There is limited evidence for population screening, but prevention (with a healthy diet and sometimes medication for diabetes, cholesterol and high blood pressure is used both to prevent THT) and to decrease the risk of compilations. ‘Mack this question = => Question Id : 65776 Question 10 of 30 Inpairment of oxygen transport occurs in all of the following EXCEPT a) Carbon monoxide intoxication b) Tetralogy of Fallot c) Anemia. 4) Cyanide poisoning. ©) Edema. Answer (Expionation | Other User's Explanation Report An Error Question Explanation: Cyenide poisoning results in a paradoxical state of high oxygen tension in venous blood. This results from celhilar imparment of the electron-transfer fiction of cytochrome oxidase, which prohibits these cells from utilizing oxygen. Anemia is ascociated with a corresponding decline in the oxygen-carrying capacity of the blood. Athough the PaO2 remains normal, the absolute emount of ‘oxygen trensported per unit volume of blood is decreased. When hemoglobin is combined with carbon monexide, the resultant carboxyhemoglobin is unavailable for oxygen transport Any condition, such as tetralogy of Fallot, which causes right-to-left shunting, causes hypoxia Edema, by increasing the distance, through which oxygen must diffise prior to reaching the cell, can also cause hypoxia ‘Mack this question = => Question Id : 65776 Question 10 of 30 Inpairment of oxygen transport occurs in all of the following EXCEPT a) Carbon monoxide intoxication b) Tetralogy of Fallot c) Anemia. Y © &) Cyanide poisoning ©) Edema. Answer (Expionation | Other User's Explanation Report An Error Question Explanation: Cyenide poisoning results in a paradoxical state of high oxygen tension in venous blood. This results from celhilar imparment of the electron-transfer fiction of cytochrome oxidase, which prohibits these cells from utilizing oxygen. Anemia is ascociated with a corresponding decline in the oxygen-carrying capacity of the blood. Athough the PaO2 remains normal, the absolute emount of ‘oxygen trensported per unit volume of blood is decreased. When hemoglobin is combined with carbon monexide, the resultant carboxyhemoglobin is unavailable for oxygen transport Any condition, such as tetralogy of Fallot, which causes right-to-left shunting, causes hypoxia Edema, by increasing the distance, through which oxygen must diffise prior to reaching the cell, can also cause hypoxia ‘Marke this question eq => Question Td : 82146 Question 11 of 30 “Which one of the following EEG characteristic is abnomnal in first degree AV block? a) P-wave )PR interval ©) QRS complex 4) ST segment ©) T-wave Anewer EMAAR) ote: uesr Explanation Repo An Err Question Explanation: ‘This is an EKG of Fust-degree AV block defined as a PR interval (B) greater than 0,20 seconds. Right arial enlargement causes aa increase voltage of the P-wave (A) on the ECG. Myocarcial ischemia classically causes ST-segment (D) depression. Pericardial effusion wil increase the distance between the source of electrical activty (myocardurn) and the votage sensor ECG leads), decreasing the amount of voltage detecied and therefore causing a low-voltage QRS complex (C). Eaily, mild hyperkalemia cavses peaked T-waves (B), which can progress to PR segment prolongation and QRS widering, a a mit ‘Olnterface Education ‘Marke this question eq => Question Td : 82146 Question 11 of 30 ‘Which one of the folowing EKG characteristic is abnormal in first degree AV block? a) P-wave YM © b)PR interval ©) QRS complex d) ST segment e) T-wave Anewer EMAAR) ote: uesr Explanation Repo An Err Question Explanation: ‘This is an EKG of Fust-degree AV block defined as a PR interval (B) greater than 0,20 seconds. Right arial enlargement causes aa increase voltage of the P-wave (A) on the ECG. Myocarcial ischemia classically causes ST-segment (D) depression. Pericardial effusion wil increase the distance between the source of electrical activty (myocardurn) and the votage sensor ECG leads), decreasing the amount of voltage detecied and therefore causing a low-voltage QRS complex (C). Eaily, mild hyperkalemia cavses peaked T-waves (B), which can progress to PR segment prolongation and QRS widering, a a mit ‘Olnterface Education 2/24/2014 AM. ‘Mark this question e => (Question Id : 82341 Question 12 of 30 “What is the most common cause of hypertension in adults? a) Hyperthyroidism. ) Renal artery stenosis. 6) Idiopathic (essential) 4) Cushing’s syndrome. «) Coarctation of the aorta Ancwer [RESPIR Other Users Explanation Repos An Exor Question Explanation: Essential hypertension accounts for approximately 95% of all cases. Other etiologies include endocrinopathies [hyperthyroidism (A), Cushing's syncrome (D), hyperaldosteronism, renal faiure], structural [renal artery stenosis (B), coarctation (E)], and pharmacological (oral contraceptives) 2/24/2014 AM. ‘Mark this question e => (Question Id : 82341 Question 12 of 30 “What is the most common cause of hypertension in adults? a) Hyperthyroidism. ) Renal artery stenosis. Y © 6) Iiiopathic (essential) 4) Cushing’s syndrome. «) Coarctation of the aorta Ancwer [RESPIR Other Users Explanation Repos An Exor Question Explanation: Essential hypertension accounts for approximately 95% of all cases. Other etiologies include endocrinopathies [hyperthyroidism (A), Cushing's syncrome (D), hyperaldosteronism, renal faiure], structural [renal artery stenosis (B), coarctation (E)], and pharmacological (oral contraceptives) ‘Marie this question <= => (Question Id : 85929 Question 13 of 30 An adul from South America develops perpheral edema, breathlessness, and decreased exercise tolerance. Echocardiographic studies demonstrate that the ventricles are not expanding normally during diastole. Biopsy from endocardal tissue demonstrates fibrosis of the endocardiam and focal myocarcal necrosis in the adjacent myocardum, An eosinophilic infiltrate is prominent in the biopsy, and peripheral blood eosinophilia is also present. The most likely diagnosis in this patient is a) Alcoholic cardiemyopthy b) Cardiac amyloidosis c) Endocardial fibroelastosis 4) Idiopathic subaortic stenosis ) Loeliler endocarditis Question Explanation: ‘Trouble filing the ventricles indicates that the patient has a restrictive cardiomyopathy The presence of endomyocardial fibrosis with myocyte necrosis and a prominent eosinophilic infltrate is most characteristic of Loeffler endocarditis, This condition, also sometimes called endomyocardial fbrosis with hypereosinophilia syndrome, appears to be the result ofa direct toxicity to the heart by proteins (eosinophil ribomuclease and eosinophil major basic protein) in eosinophil granules designed to kill large parasites. Why only a few patients with hypereesinophilia develop cardiac disease is unclear. The underlying hypercosinophilia is often idiopathic, although parasites or other causes of hypercosinophiia have occasionally been reported. Death can occur cue to heast failure, arrhythmias, or massive emboli (clots form on the damaged endometrium), Formerly the prognosis was very poor, but carly diagnosis (often during investigation of kypereosinophilia) and open heatt surgery to resect the Sbrous tissue have markedly improved the prognosis, Alcoholic cardiomyopathy produces a dilated heast, Severe cardiac amyloidosis does produce a restrictive pattem but the biopsy would show red extracellular deposits. Endocardial fbroelestosis, procuces a restrictive pattern butis a disease of young children, Idiopathic subaonic stenosis produces a hypertrophic cardiomyopathy. ‘Marie this question <= => (Question Id : 85929 Question 13 of 30 An adul from South America develops perpheral edema, breathlessness, and decreased exercise tolerance. Echocardiographic studies demonstrate that the ventricles are not expanding normally during diastole. Biopsy from endocardal tissue demonstrates fibrosis of the endocardiam and focal myocarcal necrosis in the adjacent myocardum, An eosinophilic infiltrate is prominent in the biopsy, and peripheral blood eosinophilia is also present. The most likely diagnosis in this patient is a) Alcoholic cardiemyopthy b) Cardiac amyloidosis c) Endocardial fibroelastosis 4) Idiopathic subaortic stenosis ¥ © &) Loefiler endocarditis Question Explanation: ‘Trouble filing the ventricles indicates that the patient has a restrictive cardiomyopathy The presence of endomyocardial fibrosis with myocyte necrosis and a prominent eosinophilic infltrate is most characteristic of Loeffler endocarditis, This condition, also sometimes called endomyocardial fbrosis with hypereosinophilia syndrome, appears to be the result ofa direct toxicity to the heart by proteins (eosinophil ribomuclease and eosinophil major basic protein) in eosinophil granules designed to kill large parasites. Why only a few patients with hypereesinophilia develop cardiac disease is unclear. The underlying hypercosinophilia is often idiopathic, although parasites or other causes of hypercosinophiia have occasionally been reported. Death can occur cue to heast failure, arrhythmias, or massive emboli (clots form on the damaged endometrium), Formerly the prognosis was very poor, but carly diagnosis (often during investigation of kypereosinophilia) and open heatt surgery to resect the Sbrous tissue have markedly improved the prognosis, Alcoholic cardiomyopathy produces a dilated heast, Severe cardiac amyloidosis does produce a restrictive pattem but the biopsy would show red extracellular deposits. Endocardial fbroelestosis, procuces a restrictive pattern butis a disease of young children, Idiopathic subaonic stenosis produces a hypertrophic cardiomyopathy. 014 7:12:58 AM ‘Mat this question qerz Question Td : 89659 Question 14 of 30 ‘Which of the following is NOT a cause of angina? a) Hypertrophic cardiomyopathy. b) Acttic stenosis. c) Thrombocytopenia, 4) Vesculits. ©) Coronary atherosclerosis, Question Explanation: Coronary atherosclerotis is the most common cause of angina. The fixed narrowing ofthe coronary arteries produces angina when increased oxygen requirements cannot be met, Likewise, vasculiis may couse nerrowing of the coronary vessels or may resuit in coronary emboli from marantic endocardits. Conditions that cause a marked disproportion betwreea the coronary perfusion pressure and the heart's oxygen requirements also produce angina, In addition, in aortic stenosis, the coronary ostia may be occluded. Thrombotic thrombocytopenic purpura, thrombocytosis, leukernia, and hyperviscosity syndromes may all cause angina by producing thrombosis ofthe coronary artery. 014 7:12:58 AM ‘Mat this question qerz Question Td : 89659 Question 14 of 30 ‘Which of the following is NOT a cause of angina? a) Hypertrophic cardiomyopathy. +b) Acrtic stenosis. Y © c) Thrombocytopenia 4) Vesculits. #) Coronary atherosclerosis Question Explanation: Coronary atherosclerotis is the most common cause of angina. The fixed narrowing ofthe coronary arteries produces angina when increased oxygen requirements cannot be met, Likewise, vasculiis may couse nerrowing of the coronary vessels or may resuit in coronary emboli from marantic endocardits. Conditions that cause a marked disproportion betwreea the coronary perfusion pressure and the heart's oxygen requirements also produce angina, In addition, in aortic stenosis, the coronary ostia may be occluded. Thrombotic thrombocytopenic purpura, thrombocytosis, leukernia, and hyperviscosity syndromes may all cause angina by producing thrombosis ofthe coronary artery. 22472014 7:13:14 AM ‘Mark this question & => Question Td : 94474 Question 15 of 30 A.22 year old drug addict is being treated with cloxacillin for confirmed acute staphylococcal endocarditis involving the aortic value His fever has persisted, According to the nurse recently he has become quite distressed and is complaining of shormess of breath. On examination, the first heart sound is barely audible and the previous fill ength diastolic murmur is now of very short duration, less than Jy diastole. There are bibasilar rales. The most appropriate management would be which of the following? 2) Emergency aartic valve replacement ) Continued observation with present therapy ©) Introduction of duretic and aitetioad-reduction therapy 4) Antibiotic change to vancomycin «) Repeat enchocardiograrn Answer (Botehaion | Other User's Explanation Report An Error Question Explanation: Infective endocarditis is infection of the endocardium, usualy with bacteria (commonly streptococci and staphylococci) or fungi. Tt produces ever, heart murmurs, petechiae, anemia, embclic phenomena, and endocardial vegetations. Vegetations may result in valvular incompetence or obstruction, myocardial abscess, or mycotic aneurysm. Diagnosis requires demonstration of microorganisms in blood and usually echocardiography. Treatment consists of prolonged antimicrobial treatment andl sometimes surgery. Surgery (debridement, velve repair or replacement) is frequently required for abscess, persistent infection despite antinicrobidl therapy Ge, persistent positive blood cultures or recurrent embol), or severe valvular regurgitation. Timing of surgery requites experienced clinical judgment. IPheart failure caused by a correctable lesion is worsening (particularly when the orgenisi is S. aureus, a gram-negative bacillus, or a fiangus), surgery may be required after only 24 to 72h of antimicrobial therapy. In patients with prosthetic valves, surgery may be required when TEE shows valve dehiscence on a paravalvular abscess, when valve dysfunction precipitates heart falure, when recurrent embol ate detected, or when the infection is caused by an antimicrobial resistant organssm, 22472014 7:13:14 AM ‘Mark this question & => Question Td : 94474 Question 15 of 30 A.22 year old drug addict is being treated with cloxacillin for confirmed acute staphylococcal endocarditis involving the aortic value His fever has persisted, According to the nurse recently he has become quite distressed and is complaining of shormess of breath. On examination, the first heart sound is barely audible and the previous fill ength diastolic murmur is now of very short duration, less than Jy diastole. There are bibasilar rales. The most appropriate management would be which of the following? JY © 2) Emergency aortic valve replacement ) Continued observation with present therapy ©) Introduction of duretic and aitetioad-reduction therapy 4) Antibiotic change to vancomycin «) Repeat enchocardiograrn Answer (Botehaion | Other User's Explanation Report An Error Question Explanation: Infective endocarditis is infection of the endocardium, usualy with bacteria (commonly streptococci and staphylococci) or fungi. Tt produces ever, heart murmurs, petechiae, anemia, embclic phenomena, and endocardial vegetations. Vegetations may result in valvular incompetence or obstruction, myocardial abscess, or mycotic aneurysm. Diagnosis requires demonstration of microorganisms in blood and usually echocardiography. Treatment consists of prolonged antimicrobial treatment andl sometimes surgery. Surgery (debridement, velve repair or replacement) is frequently required for abscess, persistent infection despite antinicrobidl therapy Ge, persistent positive blood cultures or recurrent embol), or severe valvular regurgitation. Timing of surgery requites experienced clinical judgment. IPheart failure caused by a correctable lesion is worsening (particularly when the orgenisi is S. aureus, a gram-negative bacillus, or a fiangus), surgery may be required after only 24 to 72h of antimicrobial therapy. In patients with prosthetic valves, surgery may be required when TEE shows valve dehiscence on a paravalvular abscess, when valve dysfunction precipitates heart falure, when recurrent embol ate detected, or when the infection is caused by an antimicrobial resistant organssm, Mark this question. —= => Question Td : 95452 Question 16 of 30 ‘An elderly patient with no significant past medical history died suddenly. An autopsy is performed, Examination of cardiac tissues reveals perivascular deposits of red extracelhilar material within the atria. Congo red stain shows a bright green fue to the deposits when viewed with polarization. The type of protein most Hkely be found in these deposits is a AA b) AB o) Ap2M, DACA ©) AL of ATTR Answer (| Explanation Other User's Explanation Report An Error Question Explanatio: The extracellular deposits with apple green le on polarization after staining with Congo red are suggestive of amyloid dep orition. “Amyloidosis is the accumulation of insoluble fibrilar material in tissues. Cardiac amyloidosis is usually a primary amyloidosis, and it results from deposition of AL type or ATTR type amyloid. Cardiac amyloidosis is the most typical restrictive cardiomyopathy, offen known as "sti heart syndrome." It may manifest with symptoms of palpitations leg swelling shortness of breath and/or fatigue. However the diagnosis is difficul: to make since findings fiom examination are not specific. Nevertheless, the disease may be complicated by atrial frillation or ventricular arrhythmias, which in some cases may be fatal [uae BA Xeeoe sted Digearel \Clinical Manifestations Itype [AA(choice |[Secondary amyloidosis (antoimmnne|[Deposttion in kidney, spleen, liver, most commonly presents as renal | lA) Jtisease, chronic infection) lnsuffciency/nephrotic syndrome BE noice Ua eheimer Dementia B) 2M ae [Most commonly musculoskeletal, with deposits in muscles, igaments, Kchoice C) i 3 synovium, bone, resulting in arthropathy, carpal tunnel, fractures [A Cal (choice||Medullary thyroid carcinoma 5 rece [Depostion in the tumor; no known clirical censequence [aL or ATTR [AL: light chain deposition; systemic, includes heart, kidney ATTR; mostly | (erie eos lcardiac deposition [Amylin \Type 2 diabetes [Deposition inislet cells in pancreas, unclear clinical consequence Vchoice FE) |9P* . Pees Meee ole pare a Mark this question. —= => Question Td : 95452 Question 16 of 30 ‘An elderly patient with no significant past medical history died suddenly. An autopsy is performed, Examination of cardiac tissues reveals perivascular deposits of red extracelhilar material within the atria. Congo red stain shows a bright green fue to the deposits when viewed with polarization. The type of protein most Hkely be found in these deposits is a AA b) AB o) Ap2M, DACA ¥ © e)ALor ATTR Answer (| Explanation Other User's Explanation Report An Error Question Explanatio: The extracellular deposits with apple green le on polarization after staining with Congo red are suggestive of amyloid dep orition. “Amyloidosis is the accumulation of insoluble fibrilar material in tissues. Cardiac amyloidosis is usually a primary amyloidosis, and it results from deposition of AL type or ATTR type amyloid. Cardiac amyloidosis is the most typical restrictive cardiomyopathy, offen known as "sti heart syndrome." It may manifest with symptoms of palpitations leg swelling shortness of breath and/or fatigue. However the diagnosis is difficul: to make since findings fiom examination are not specific. Nevertheless, the disease may be complicated by atrial frillation or ventricular arrhythmias, which in some cases may be fatal [uae BA Xeeoe sted Digearel \Clinical Manifestations Itype [AA(choice |[Secondary amyloidosis (antoimmnne|[Deposttion in kidney, spleen, liver, most commonly presents as renal | lA) Jtisease, chronic infection) lnsuffciency/nephrotic syndrome BE noice Ua eheimer Dementia B) 2M ae [Most commonly musculoskeletal, with deposits in muscles, igaments, Kchoice C) i 3 synovium, bone, resulting in arthropathy, carpal tunnel, fractures [A Cal (choice||Medullary thyroid carcinoma 5 rece [Depostion in the tumor; no known clirical censequence [aL or ATTR [AL: light chain deposition; systemic, includes heart, kidney ATTR; mostly | (erie eos lcardiac deposition [Amylin \Type 2 diabetes [Deposition inislet cells in pancreas, unclear clinical consequence Vchoice FE) |9P* . Pees Meee ole pare a ‘Mat this question a> ‘Question Td: 95813 Question 17 of 30 4.65 year old woman is brought-to the emergency department with complains of substemal chest pain radiating down her lef. arm. ECG showed elevated ST-T wave segments in the anteriorlateral leads. She is diagnosed as having a myocardial infarction. This patient is post menopausal, diabetic and hypertensive. Which of the following treatment is NOT recommended after a heart attack in this patient? a) Enteric coated aspirin b) Control of gucose wih HbA less than 8% ©) Postmenopausal estrogen, 9) Blood pressure control («130/ 85) €) Lowering the LDL cholesterol level less than 180 mg/l Question Explanation: the patient has two or more risk factors for atherosclerosis, like this patient does, then the LDL cholesterol level should be lowered to less than 130 mg/dl and in some cases even less than 100 mg/dl, Aspirin is highly recommended for prophylanis after a stroke or myocardial infarction to reduce the incidence of further thrombosis, Tight control of diabetes mellitus in a patient with coronary artery cisease is necessary to prevent further atherosclerosis and recurrent MI. Postmenopausal estrogen has been shown to increase the HDL and thus decrease the risk for coronary artery disease. Hypertension is a very important risk factor for coronary artery disease, and good control of blood pressure will decrease the risk for further events. Smoking is also a risk factor for coronary artery disease. ‘Mat this question a> ‘Question Td: 95813 Question 17 of 30 4.65 year old woman is brought-to the emergency department with complains of substemal chest pain radiating down her lef. arm. ECG showed elevated ST-T wave segments in the anteriorlateral leads. She is diagnosed as having a myocardial infarction. This patient is post menopausal, diabetic and hypertensive. Which of the following treatment is NOT recommended after a heart attack in this patient? a) Enteric coated aspirin b) Control of gucose wih HbA less than 8% ©) Postmenopausal estrogen, 9) Blood pressure control («130/ 85) Y © #) Lowering the LDL cholesterol level less than 180 mg/dl Question Explanation: the patient has two or more risk factors for atherosclerosis, like this patient does, then the LDL cholesterol level should be lowered to less than 130 mg/dl and in some cases even less than 100 mg/dl, Aspirin is highly recommended for prophylanis after a stroke or myocardial infarction to reduce the incidence of further thrombosis, Tight control of diabetes mellitus in a patient with coronary artery cisease is necessary to prevent further atherosclerosis and recurrent MI. Postmenopausal estrogen has been shown to increase the HDL and thus decrease the risk for coronary artery disease. Hypertension is a very important risk factor for coronary artery disease, and good control of blood pressure will decrease the risk for further events. Smoking is also a risk factor for coronary artery disease. 2242014 AM. ‘Mark this question & => Question 18 of 30 ‘All of the following are associated with Torsades de pointes, EXCEPT a) Quinidine b) Hypokalemia ©) Phenytoin 4) Liquid protein dicts €) Congenital prolongation of the QT syndrome (Question Explanation: Torsades de pointes (twisting of the points) describe ventricular tachycardia with polymorphic QRS complexes, which change in amplitude and cycle length. The direction of the points of the QRS complex appears to revolve around an imaginary isoelectric ine. By definition, the syndrome is associated with QT prolongation Drugs, such as quinidine, which prolong the QT interval, may produce the syndrome, while drugs that shorten the QT snterval are used therapeutically. Metabolic disorders, such as hypokalemia and hypomagnesemia, which prolong the QT interval also produce the syndrome, The multiple metabolic abnormalities reported to occur with liquid protein diets probably explain the association of this therapy with Torsades de pointes 2242014 AM. ‘Mark this question & => Question 18 of 30 All of the following are associated with Torsades de pointes, EXCEPT a) Quinidine 'b) Hypokalemia Y © 0) Phenytoin d) Liquid protein diets e) Congenital prolongation of the QT syndrome (Question Explanation: Torsades de pointes (twisting of the points) describe ventricular tachycardia with polymorphic QRS complexes, which change in amplitude and cycle length. The direction of the points of the QRS complex appears to revolve around an imaginary isoelectric ine. By definition, the syndrome is associated with QT prolongation Drugs, such as quinidine, which prolong the QT interval, may produce the syndrome, while drugs that shorten the QT snterval are used therapeutically. Metabolic disorders, such as hypokalemia and hypomagnesemia, which prolong the QT interval also produce the syndrome, The multiple metabolic abnormalities reported to occur with liquid protein diets probably explain the association of this therapy with Torsades de pointes ‘Mark this question & => Question Id: 101432 Question 19 of 30 AS1 year od woman with a 10 year history oftype 2 diabetes mellitus is found to have a blood pressure of 145/90 mmilg and significant Microalbuminuria on routine checkup. Which statement regarding the use of an ACE inhibitor in this patient is not correct? a) A previous history of angioneurotic edema ») Renal insufficiency ©) Asthma 4d) A history of recent myocardial infarction 2) A cardiac ejection faction <25% Question Explanation: Angioneurotic edema can be life threatening, and ACE inhibitors should not be given to patients with a history of this condition from ary cause. Elevated crectinine levels are not an absolute contraindication to ACE inhibitor therapy. Myocardial infarction and a reduced cardiac ejection frection are indications for ACE inhibitor therapy ACE inhibitors do not affect asthina ‘Mark this question & => Question Id: 101432 Question 19 of 30 AS1 year od woman with a 10 year history oftype 2 diabetes mellitus is found to have a blood pressure of 145/90 mmilg and significant Microalbuminuria on routine checkup. Which statement regarding the use of an ACE inhibitor in this patient is not correct? Y © a) A previous history of angicneurotic edema ») Renal insufficiency ©) Asthma d) A history of recent myocardial infarction 2) A cardiac ejection fraction <25% Question Explanation: Angioneurotic edema can be life threatening, and ACE inhibitors should not be given to patients with a history of this condition from ary cause. Elevated crectinine levels are not an absolute contraindication to ACE inhibitor therapy. Myocardial infarction and a reduced cardiac ejection frection are indications for ACE inhibitor therapy ACE inhibitors do not affect asthina 2/24/2014 7:15:39 AM ‘Mark this question & => Question Td : 109561 Question 20 of 30 A male of 77 years age presents to the emergency room with syncope, palpitations, and hypotension. He has been taking quinidine for arrhythmias in the past. An old ECG reveals a prolonged QT interval, Now his ECG reveals ventricular tachycardia. The treatment of choice is a) Procainamide b) Lidocaine ©) Veraparril 4) Adenosine €) Magnesiuin sulfate Question Explanation: ‘This patient has torsades de pointes, which means twisting around the points. The refractory time is prolonged and mulnerable to ventricular excitation, which causes ventricular tachycardia and subsequent hemodynamic instability. Procainamide, quinidine, amitriptine, and hypocalcemia can all cause prolongation of the QT interval and subsequent torsades. The treatment is pacemaker overdrive or magnesium sulfate, Procainamide is a type I antiarrythmic drug which can cause torsades. Lidocaine is usually used for ventricular tachycardia from ischemia or other causes. Verapamil is a calcum channel blocker used to treat supraventricular tachycerdias. Adenosine is also used to treat supraventricular tachycardias. Itis very short acting 2/24/2014 7:15:39 AM ‘Mark this question & => Question Td : 109561 Question 20 of 30 Amale of 77 years age preseats to the emergency room with syncope, palpitations, and hypotension. He has been taking quinidine for arrhythmias in the past. An old ECG reveals a prolonged QT interval. Now his ECG reveals ventricular tachycardia. The treatment of choice is a) Procanamide ) Lidocaine ©) Veraparril 4) Adenosize Y¥ © ©) Magnesium sulfate Question Explanation: ‘This patient has torsades de pointes, which means twisting around the points. The refractory time is prolonged and mulnerable to ventricular excitation, which causes ventricular tachycardia and subsequent hemodynamic instability. Procainamide, quinidine, amitriptine, and hypocalcemia can all cause prolongation of the QT interval and subsequent torsades. The treatment is pacemaker overdrive or magnesium sulfate, Procainamide is a type I antiarrythmic drug which can cause torsades. Lidocaine is usually used for ventricular tachycardia from ischemia or other causes. Verapamil is a calcum channel blocker used to treat supraventricular tachycerdias. Adenosine is also used to treat supraventricular tachycardias. Itis very short acting 2/24/2014 7:15:50 AM ‘Mat this question & => Question Td: 113152 Question 21 of 30 ‘Treatment of hyperkalemia associated with acute renal failure opposes the cardiotoxic effect of hyperkalemia is which one ofthe following? a) Calcium chloride ) Glucoselinsulin ©) Sodium bicarbonate ) Kayexalate resin ©) Dialysis Question Explanation: Calcium chloride opposes the carciotoxic effects of hyperkalemia and should be infused! in patients with acute ECG changes, Giucose/insuln and sodium bicarbonate work by inducing intracelilar potassium shit's. Kayexalate binds potassim in the gut and dialysis removes potassium by diffusion. 2/24/2014 7:15:50 AM ‘Mat this question & => Question Td: 113152 Question 21 of 30 ‘Treatment of hyperkalemia associaied with acute renal failure opposes the cardiotoxic effect of hyperkalemia is which one ofthe following? Y¥ © a) Calcium chloride 'b) Ghacoselinsulin ©) Sodium bicarbonate d) Kayexalate resin €) Dialysis Question Explanation: Calcium chloride opposes the carciotoxic effects of hyperkalemia and should be infused! in patients with acute ECG changes, Giucose/insuln and sodium bicarbonate work by inducing intracelilar potassium shit's. Kayexalate binds potassim in the gut and dialysis removes potassium by diffusion. 2/24/2014 7:16:04 AM ‘Mark this question & => Question Td : 132460 Question 22 of 30 A 62 year old woman with renal failure presents with chest pain. ECG reveeled peaked T-waves in all the leads. The most common electrolyte abnormality leading to this condition is which one of the following? a) Hypercalcemia b) Hyperkalemia c) Hypokalemia 4) Hypomagnesemia €) Hypocalcemia Question Explanation: Peaked T waves become evident when the serum potassium level exceeds 6.5 mEq per iter. This peaking of the T waves is a manifestation of the accelerated repolarization of the cardiac action potential produced by hyperkalemia. Hypercalcemia causes shortening of the QT interval on the electrocardiogram, and incidences of bradycardia and first degree heart block have been. reported, Hypokalemia and hypomagnesemia both cause segging of the ST segment, depression of the T wave, and elevation of the ‘U wave Both electrolyte abnormalities may precipitate serious arriythmias. Hypocalcemia causes prolongation of the QT interval and this a possibilty of torsades de pointes (a malignant ventricular acrhythmia) 2/24/2014 7:16:04 AM ‘Mark this question & => Question Td : 132460 Question 22 of 30 A 62 year old woman with renal failure presents with chest pain. ECG reveeled peaked T-waves in all the leads. The most common electrolyte abnormality leading to this condition is which one of the following? a) Hypercalcemia Y © b) Hyperkalemia c) Hypokalemia 4) Hypomagnesemia €) Hypocalcemia Question Explanation: Peaked T waves become evident when the serum potassium level exceeds 6.5 mEq per iter. This peaking of the T waves is a manifestation of the accelerated repolarization of the cardiac action potential produced by hyperkalemia. Hypercalcemia causes shortening of the QT interval on the electrocardiogram, and incidences of bradycardia and first degree heart block have been. reported, Hypokalemia and hypomagnesemia both cause segging of the ST segment, depression of the T wave, and elevation of the ‘U wave Both electrolyte abnormalities may precipitate serious arriythmias. Hypocalcemia causes prolongation of the QT interval and this a possibilty of torsades de pointes (a malignant ventricular acrhythmia) ‘Mark this question = => Question Td: 139416 Question 23 of 30 4.55 year old man came to emergency room with severe chest pain radiating to the left arm. Which ofthe following serum marker levels would best aid in the evaluation of this individual's chest pein? a) Aspartate aminotransferase (AST) ) Creating kinase-MB isozyme c) Lactate dehydrogenase-1 isozyme (LDH-1) 4) Total creatine kinase €) Troponin Question Explanation: Cardiac specific forms of troponin T and troponin Tare not normally detectable in the blood but may increase 20 fold following a myocardial infarction. Slight amino acid differences between cardiac and skeletal muscle fortes of troponin allow specific association ofthe troponin with heart muscle damage, rather than skeletal muscle damage. Troponin is the best serum merker for myocardial infarction for the frst 8 hours and because levels of the cardiac troponins remain elevated for 7 to 10 days it may be useful for evaluation of small CK. negative infarctions for several days after the event ASTis a nonspecific marker for cardiac liver and skeletal muscle, Due to its lack of specificity, itis much less useful as 2 marker of myocardial infarction than CK-MB or tropotin, Both total creatine kinase and ts more cardiac specific form CK-MB are most useful from 8 to 24 hours after infarction typically with peales at 12 to 18 hours, LDH 1, the cardiac specific form of lactic dehydrogenase, is the test of choice 2 to 7 days after a suspected myocardial infarction, ‘Mark this question = => Question Td: 139416 Question 23 of 30 A.55 year oldmen came to emergency room with severe chest pain radiating to the lef arm, Which ofthe following serum marker levels would best aid in the evaluation of this individval’s chest pein? a) Aspartate aminotransferase (AST) ) Creating kinase-MB isozyme c) Lactate dehydrogenase-1 isozyme (LDH-1) 4) Total creatine kinace Y © €) Troponin Question Explanation: Cardiac specific forms of troponin T and troponin Tare not normally detectable in the blood but may increase 20 fold following a myocardial infarction. Slight amino acid differences between cardiac and skeletal muscle fortes of troponin allow specific association ofthe troponin with heart muscle damage, rather than skeletal muscle damage. Troponin is the best serum merker for myocardial infarction for the frst 8 hours and because levels of the cardiac troponins remain elevated for 7 to 10 days it may be useful for evaluation of small CK. negative infarctions for several days after the event ASTis a nonspecific marker for cardiac liver and skeletal muscle, Due to its lack of specificity, itis much less useful as 2 marker of myocardial infarction than CK-MB or tropotin, Both total creatine kinase and ts more cardiac specific form CK-MB are most useful from 8 to 24 hours after infarction typically with peales at 12 to 18 hours, LDH 1, the cardiac specific form of lactic dehydrogenase, is the test of choice 2 to 7 days after a suspected myocardial infarction, ‘Mark this question & => Question 24 of 30 A 69 year old male is brought to the emergency department because of the sudden onset of teasing chest pain waking hin from the sleep. The pain seers to originate in the anterior chest and radiate to the back in the interscepular region notrelieved by rest. Patient has hyperlipidemia and hypertension and a strong fanily history for cardiac disease. Auscultation reveals diastolic murtnur along the left sternal border. ECGis normal CXR shows widening of the mediastinum, ‘The most licely diagnosis is a) Aortic dissection 'b) Acute mediastinitis ©) Acute Pericarditis 4) Cardiac tamponade €) Myocardial infarction Question Explanation: ‘A tearing, excnuciating chest pain that radiates to the back should always generate the clinical suspicion of aottic dissection involving the aortic arch The patient presents with extreme signs of distress. Aortic insufficiency with ts associated diastolic murmur, frequently develops. A discrepancy in blood pressure or pulse between the right and left arms is an additional supporting sign Mediastinal widening is often seen on chest X-ray. But the diagnosis should be confirmed by CT or MRI scans. Hypertension is the most common predisposing factor, but Marfan syndrome is a classic condition associated with aortic dissection. Acute mediastinitis is a rare infectious complication due to extension of suppurative processes from adjacent cervical organs (¢.g,, peritonsilitis, thyroiditis) or perforation of esophagus or trachea. The petient has chest pain but lacks fever and other systemic signs of infections. The history ‘of arecent dental abscess should not deceive you Acute pericarditis produces chest pain, which is gradual in onset and usuelly accompanied by a fiiction rub, Tt pericardial effusion is particularly abundant, cardiac tamponade may ensue. The latter will result in acute signs and syrnptoms of cardiac failure, necessitating emergency pericardiocentesis to relieve the pressure on the heart ‘Myocardial infarction is probably the most important differential diagnosis to consider in case of aortic dissection, but the absence of ECG changes suggesting myocardial ischemia argues against it in this patient, ‘Mark this question & => Question 24 of 30 4 69 year old male is brought to the emergency department because of the sudden onset of tearing chest pain waking him from the sleep. The pain seems to originate in the anterior chest and radiate to the back in the interscapular region aot relieved by rest. Patient ‘has hyperlipidemia and hypertension and a strong family history for cardiac disease. Auscultation reveals diastolic murmur along the left sternal border. ECGis normal CXR shows widening of the mediastinum. The most licely diagnosis is o¥ © a) Aortic dissection b) Acute mediastinitis c) Acute Pericarditis 4) Cardiac tamponade €) Myocardial infarction Question Explanation: ‘A tearing, excnuciating chest pain that radiates to the back should always generate the clinical suspicion of aottic dissection involving the aortic arch The patient presents with extreme signs of distress. Aortic insufficiency with ts associated diastolic murmur, frequently develops. A discrepancy in blood pressure or pulse between the right and left arms is an additional supporting sign Mediastinal widening is often seen on chest X-ray. But the diagnosis should be confirmed by CT or MRI scans. Hypertension is the most common predisposing factor, but Marfan syndrome is a classic condition associated with aortic dissection. Acute mediastinitis is a rare infectious complication due to extension of suppurative processes from adjacent cervical organs (¢.g,, peritonsilitis, thyroiditis) or perforation of esophagus or trachea. The petient has chest pain but lacks fever and other systemic signs of infections. The history ‘of arecent dental abscess should not deceive you Acute pericarditis produces chest pain, which is gradual in onset and usuelly accompanied by a fiiction rub, Tt pericardial effusion is particularly abundant, cardiac tamponade may ensue. The latter will result in acute signs and syrnptoms of cardiac failure, necessitating emergency pericardiocentesis to relieve the pressure on the heart ‘Myocardial infarction is probably the most important differential diagnosis to consider in case of aortic dissection, but the absence of ECG changes suggesting myocardial ischemia argues against it in this patient, 2/24/2014 7:17:01 AM ‘Mark this question = => Question Id ; 140363 Question 25 of 30 A.36 year old man has progressive shoriness of breath, orthopaca, and occasional dizziness and confusion, On examination pulmonary crackles, a laterally displaced apical beat, and a harsh systolic murmur are audible, CXR shows cardiomegaly, intersitial edema, and Kerly B lines. The pathologic changes most likely associated with this condition is a) Bicuspid aortic valves ) Caleification of tricuspid aortic valves c) Dissection of ascending aorta 4) Rupture of aortic valve leaflets €) Vegetations on the aortic valve Question Explanation: Bicuspid aortic valves manifest aortic stenosis and, secondacily, heart failure at clinically earlier ages (in the 4th decade), Aortic stenosis causes increased afterload, lypertrophy, and subsequent dilatation of the left ventricle. Signs consistent with left heart failure are pulmonary edema, pleural effusions, cyanosis, cardiomegaly, displaced apical beat, and Kerly B lines Symptoms of left heart failure include shormess of breath, orthopnea, paroxysmal nocturnal dyspnea, dizziness, confusion, cool extremities at rest, and weakness, The charecteristic murmur is a crescendo decrescendo systolic murmur best heard in the second right intercostal space. ‘An ejection click may be heard, mosily associated with bicuspid aortic valves. Cabification of the aonic valve usually occurs in the elderly population. Aortic dissection does not cause aortic stenosis, It presents with chest pain of acute onset, which must be differentiated from chest pain of carciac ischemic origin (Le., angina or myocardial infarction). Cystic medial degeneration (fagnentation of elastic laminge with formation of cystic spaces and deposition of myoid matrix) is the underlying pathologic change. -Aorte dissection is frequently (but not exclusively) encountered in association with Marfan syndrome. Rupture of valve leaflets and. vegetations on the valve leaflets are seen when the aottic valve is affected by infectious endocarditis, Bacteria grow on the endothelial surface, promoting acute inflammation and deposition of fibrin-rich vegetations, Enzymes released by neutrophils cause tissue damage and may lead to perforation and rupture of valve leaflets. This resuits m aorte insufficiency (Le., regurgitation) not stenosis. Farthermore, infectious endocarditis follows an acute or subacute course, Echocardiogram is very sensitive in demonstrating aortic vegetations and structural abnormalities 2/24/2014 7:17:01 AM ‘Mark this question = => Question Id ; 140363 Question 25 of 30 A.36 year old man has progressive shortness of breath, orthopaca, and occasional dizsiness and confusica, On examination pulmonary crackles, a laterally displaced apical beat, and a harsh systolic murmur are audible, CKR shows cardiomegaly, intersitil edema, and Kerly B lines. The pathologic changes most likely associated with this condition is Y © a) Bicuspid aortic valves ') Cakification of tricuspid aortic valves c) Dissection of ascending aorta 4) Rupture of aortic valve leaflets ©) Vegetations on the aonic valve Question Explanation: Bicuspid aortic valves manifest aortic stenosis and, secondacily, heart failure at clinically earlier ages (in the 4th decade), Aortic stenosis causes increased afterload, lypertrophy, and subsequent dilatation of the left ventricle. Signs consistent with left heart failure are pulmonary edema, pleural effusions, cyanosis, cardiomegaly, displaced apical beat, and Kerly B lines Symptoms of left heart failure include shormess of breath, orthopnea, paroxysmal nocturnal dyspnea, dizziness, confusion, cool extremities at rest, and weakness, The charecteristic murmur is a crescendo decrescendo systolic murmur best heard in the second right intercostal space. ‘An ejection click may be heard, mosily associated with bicuspid aortic valves. Cabification of the aonic valve usually occurs in the elderly population. Aortic dissection does not cause aortic stenosis, It presents with chest pain of acute onset, which must be differentiated from chest pain of carciac ischemic origin (Le., angina or myocardial infarction). Cystic medial degeneration (fagnentation of elastic laminge with formation of cystic spaces and deposition of myoid matrix) is the underlying pathologic change. -Aorte dissection is frequently (but not exclusively) encountered in association with Marfan syndrome. Rupture of valve leaflets and. vegetations on the valve leaflets are seen when the aottic valve is affected by infectious endocarditis, Bacteria grow on the endothelial surface, promoting acute inflammation and deposition of fibrin-rich vegetations, Enzymes released by neutrophils cause tissue damage and may lead to perforation and rupture of valve leaflets. This resuits m aorte insufficiency (Le., regurgitation) not stenosis. Farthermore, infectious endocarditis follows an acute or subacute course, Echocardiogram is very sensitive in demonstrating aortic vegetations and structural abnormalities ‘Mark this question ez Question Td : 175132 Question 26 of 30 A 25 year old male has headache, dizziness: and claudication Exarrination reveals hypertension in the upper imbs and hypotension in the lower limbs. Which additional finding would be most likely present? a) Aortic valvular stenosis b) Notching of inferior margins of ribs c) Patent ductus arteriosus ) Pulmonary valvular stencsis, ©) Vesculitis involving the aortic arch. Answer ( Bepianatin) Other User's Explanation Report An Error Question Explanation: ‘The adult form of aortic coarctationis caused by stenosis in the aortic arch just distal to the left subclavian artery (postductal). This leads to hypertension proximal to the obstruction, due to more blood being stalled because of the coarctation, and hypotension distal to, the stenotic segment. Hypertension in the upper part of the body manifests with headache, dizziness, and other neurologic symptoms. Hypotension in the lower part of the body results in signs and symptoms of ischemia, mest often claudication, ie., recurrent pain due to ischemia of leg muscles, In addition, collateral arteries between the precoarctation and posicoarciation aoria 2g, the intercostal and internal mammary arteries) enlarge and establish communication between aortic segments proximal and distal to stenosis, Enlarged intercostal arteries produce notching of the inferior margins of the ribs, which can be detected on X-ray and is iagnostic of this condition. Remember thet the infantile form of aortic coarctation is associated with patent ductus arteriosus, whereas the adult form is not. Hypertension can also be found due to the decreased blood flow to the kidneys, which will cause activation of the senin/angiotensin system, Aortic valnular stenosis at this age would most licely be caused by a congenitally malformed valve, usually a valve with two cusps or a single cusp. Aortic stenosis manifests with systolic hypotension, recurrent syncope, and hyperirophydilatation of the left ventricle. Low systolic pressure is present in the entire body. The isolated form of fratent ductus arteriosus leads to shunting of blood from the aorta (high-pressure vessel) to the pulmonary artery (low-pressure vessel). Eventualy. chronic cor pulmonale develops with resultant right-sided heart failure. Pulmonary valvular stenosis is a rare form of congenital heart disease that leads to chroric cor pulmonale and heart failure. Vasculitis involving the aortic arch is found in Takayasu arteritis, in which chronic inflammatory changes develop in the aortic arch and its branches, (brachiocephalic trunk, left conemon carctid, and left subclavian arteries). This comition cases sterasis of these arteries; therefore, there will be signs and symptoms ofischemia to the upper part ofthe body. Because, the radial pulees are very weak or absent, this disorder is aleo known as puleelece diceace ‘Mark this question ez Question Td : 175132 Question 26 of 30 A 25 year old male has headache, dizziness: and claudication Exarrination reveals hypertension in the upper imbs and hypotension in the lower limbs. Which additional finding would he most likely present? a) Acrtic valvular stenosis Y © b) Notching of inferior margins of ribs c) Patent ductus arteriosus 4) Pulmonary valular stencsis ) Vasculitis involving the aortic arch, Answer ( Bepianatin) Other User's Explanation Report An Error Question Explanation: ‘The adult form of aortic coarctationis caused by stenosis in the aortic arch just distal to the left subclavian artery (postductal). This leads to hypertension proximal to the obstruction, due to more blood being stalled because of the coarctation, and hypotension distal to, the stenotic segment. Hypertension in the upper part of the body manifests with headache, dizziness, and other neurologic symptoms. Hypotension in the lower part of the body results in signs and symptoms of ischemia, mest often claudication, ie., recurrent pain due to ischemia of leg muscles, In addition, collateral arteries between the precoarctation and posicoarciation aoria 2g, the intercostal and internal mammary arteries) enlarge and establish communication between aortic segments proximal and distal to stenosis, Enlarged intercostal arteries produce notching of the inferior margins of the ribs, which can be detected on X-ray and is iagnostic of this condition. Remember thet the infantile form of aortic coarctation is associated with patent ductus arteriosus, whereas the adult form is not. Hypertension can also be found due to the decreased blood flow to the kidneys, which will cause activation of the senin/angiotensin system, Aortic valnular stenosis at this age would most licely be caused by a congenitally malformed valve, usually a valve with two cusps or a single cusp. Aortic stenosis manifests with systolic hypotension, recurrent syncope, and hyperirophydilatation of the left ventricle. Low systolic pressure is present in the entire body. The isolated form of fratent ductus arteriosus leads to shunting of blood from the aorta (high-pressure vessel) to the pulmonary artery (low-pressure vessel). Eventualy. chronic cor pulmonale develops with resultant right-sided heart failure. Pulmonary valvular stenosis is a rare form of congenital heart disease that leads to chroric cor pulmonale and heart failure. Vasculitis involving the aortic arch is found in Takayasu arteritis, in which chronic inflammatory changes develop in the aortic arch and its branches, (brachiocephalic trunk, left conemon carctid, and left subclavian arteries). This comition cases sterasis of these arteries; therefore, there will be signs and symptoms ofischemia to the upper part ofthe body. Because, the radial pulees are very weak or absent, this disorder is aleo known as puleelece diceace ‘Mark this question eq => Question Td : 176994 Question 27 of 30 An unconscious male in the ICU undergoes a2 hour period of severe hypotension, Serum chemistries indicate rising CK-MB. fiaction, peaking at 5 times the upper limit of normal ECG findings are equivocal, with some degree of las ST-segment depression. over several leads. The most lkely diagnosis is a) Prinametal angina ) Stable angina c) Subendocardial infarction 4) Transmural infarction €) Unstable angina Question Explanation: ‘The high serum CK-MB indicates that the patient has sustained an infarction rather than angina, Subendocardial, rather than transmural, infarction is most ley in the serting of known, prolonged severe hypotension, and the equivocal nanure of the ECG findings confirms this diagnosis. Subendocardial infarction occurs in seniings of generalized poor perfusion complicared by increased demand or transient vasospasm, Subendocardial muscle tissue is especially vulnerable because itis farthest from the arterial supply. Tn Prinzmetal and stable angina, the CK-MB would not be expected to increase significarely. Transmural infarction is not specifically expected in the setting of shock. It produces characteristic ECG changes that are usually localized (anless a very lange infarct has occurred) to afew leads. In unstable angina an increase in cardiac enzymes maybe seen, but is usuelly less than two times the upper limit of normal. ‘Mark this question eq => Question Td : 176994 Question 27 of 30 An unconscious male in the ICU undergoes a2 hour period of severe hypotension. Serum chemistries indicate rising CK-MB fraction, peaking at 5 times the upper limit of normal ECG findings are equivecal, with some degree of flat ST-segment depression over several leads. The most likely diagnosis is a) Prinzmetal angina 'b) Stable angina ¥ © ©) Subendorardial infarction ) Trensmnural infarction e) Unstable angina Question Explanation: ‘The high serum CK-MB indicates that the patient has sustained an infarction rather than angina, Subendocardial, rather than transmural, infarction is most ley in the serting of known, prolonged severe hypotension, and the equivocal nanure of the ECG findings confirms this diagnosis. Subendocardial infarction occurs in seniings of generalized poor perfusion complicared by increased demand or transient vasospasm, Subendocardial muscle tissue is especially vulnerable because itis farthest from the arterial supply. Tn Prinzmetal and stable angina, the CK-MB would not be expected to increase significarely. Transmural infarction is not specifically expected in the setting of shock. It produces characteristic ECG changes that are usually localized (anless a very lange infarct has occurred) to afew leads. In unstable angina an increase in cardiac enzymes maybe seen, but is usuelly less than two times the upper limit of normal. 2/24/2014 7: 8 AM ‘Mark this question e-= Question Id : 178721 Question 28 of 30 Aman whose CVS was operating at the intersection of the solid curves (point A) has a cardiac output of SL/min and a right atrial pressure of 0 Hg Which condition would likely cause his CVS to operate at the intersection of the dashed lines (point B)? mae | / am mt i *] 0 “42 0 24 6 a a 12 Righy Atrial Pressure (mm Ha) © irterace Me kon a) Blood transfusion 'b) Heart failure ©) Hemorrhage 4) Spinal anesthesia e) Sympathetic block Anewer (ERTREEERY shor teers Exptna Question Explanation: ‘This individual has compensated heart failure. The circulatory system ss stable with a normal resting cardiac output of 3 Limin and an elevated right atrial pressure of Jmmig. The cardiac output curve (cardiac function curve) is depressed because of myocardial damage sustained during a heart attack. Retention of salt and water have increased the mean circulatory filing pressure (MFP) from anonmal value of +7 mm Hg to +12 mm Hg, shown on the figure. The resting cardiac output is normal, however, a further increase in MSEP cannot increase the cardiac output because the heart is operating on the plateau of the carctac output curve. Blood transfusion increases cardiac output by increasing MSEP and decreasing the resistance to venous return, Hemorrhage lowers blood volume, which decreases MSEP and shifts the venous retum curve to the left. The effects of spinal anesthesia on the cardiac output venous return curves resembles blockade ofthe syiapathetic nervous system. Blocking the syenpathetic nervous system lowers MSEP (and thus shifts the venous retum curve to the left) and decreases myocardial contractility (which shifis the cardiac output curve downward), Report An Error 2/24/2014 7: 8 AM ‘Mark this question e-= Question Id : 178721 Question 28 of 30 Aman whose CVS was operating at the intersection of the solid curves (point A) has a cardiac ouput of SLimin and a right atrial pressure of 0 Hg. Which condition would likely cause his CVS to operate at the intersection of the dashed lines (point B)? mes ae cl ae i | G3 024 8 8 Righy Atrial Pressure (mm Hg) a) Blood transfusion Y © b) Heart failure ) Hemorrhage d) Spinal anesthesia ¢) Sympathetic block Anewer (ERTREEERY shor teers Exptna Question Explanation: ‘This individual has compensated heart failure. The circulatory system ss stable with a normal resting cardiac output of 3 Limin and an elevated right atrial pressure of Jmmig. The cardiac output curve (cardiac function curve) is depressed because of myocardial damage sustained during a heart attack. Retention of salt and water have increased the mean circulatory filing pressure (MFP) from anonmal value of +7 mm Hg to +12 mm Hg, shown on the figure. The resting cardiac output is normal, however, a further increase in MSEP cannot increase the cardiac output because the heart is operating on the plateau of the carctac output curve. Blood transfusion increases cardiac output by increasing MSEP and decreasing the resistance to venous return, Hemorrhage lowers blood volume, which decreases MSEP and shifts the venous retum curve to the left. The effects of spinal anesthesia on the cardiac output venous return curves resembles blockade ofthe syiapathetic nervous system. Blocking the syenpathetic nervous system lowers MSEP (and thus shifts the venous retum curve to the left) and decreases myocardial contractility (which shifis the cardiac output curve downward), 02 Me Report An Error ‘Mark this question =z Question 1d: 206777 Question 29 of 30 A 30 year old 31 weeks pregnant female is receiving low molecular weight hepasin at treatment doses due to a pulmonary etubclism 3 months prior to conception, All foetal scans have been normal, and her BP is 126/80 munElg in the left lateral position, The correct statement is which one of the following? a) The dose of Clexane should be increased in the third trimester b) Clexane treatment needs no monitoring in pregnancy ©) Prothrombin time is an indicator of anti-factor Xa activity ) Breasifceding is not advised €) Ibis safe for her to receive NSATDs perinatally Question Explanation: There is no recommencation that the dose of LMWH should be increased in the 3rd trimester. Increases in prothrombin time and ACT are not nearly correlated with increasing LMWH antithrombotic activity and therefore are unsuitable and unreliable for monitoring LMWH activity. NSATD treatment increases the risk of haemorrhage in both mother and foetus. Ttis not known whether unchanged enoxeparin sodum is excreted in human breast milk, The oral absorption of enoxaparin sodium is unlikely. However, as a precaution, lactating mothers receiving enoxaparin sodium should be advised to avoid breast feeding, ‘Mark this question =z Question 1d: 206777 Question 29 of 30 2.30 year old 31 weeks pregnant female is receiving low molecular weight heparin at teaiment doses due to a pulmonary embelism 3 months prior to conception, All foetal scans have been normal, and her BP is 126/80 munFly in the left lateral position. The correct statement is which one of the following? a) The dose of Clexane should be increased in the third trimester ) Clexane treatment needs no monitoring in pregnancy ©) Prothrombin time is an indicator of anti-factor Xa activity VY © ABreasifeeding is not advised 6) Ibis safe for her to receive NSAIDs perinatally Question Explanation: There is no recommencation that the dose of LMWH should be increased in the 3rd trimester. Increases in prothrombin time and ACT are not nearly correlated with increasing LMWH antithrombotic activity and therefore are unsuitable and unreliable for monitoring LMWH activity. NSATD treatment increases the risk of haemorrhage in both mother and foetus. Ttis not known whether unchanged enoxeparin sodum is excreted in human breast milk, The oral absorption of enoxaparin sodium is unlikely. However, as a precaution, lactating mothers receiving enoxaparin sodium should be advised to avoid breast feeding, ‘Mark this question e Question Td : 217086 Question 30 of 30 A.42-year-old IV doug abuser has fever, malaise, and cough. A cardiac murmur exists. Cultures grow methicilin-resistant Staphylococcus aureus and a chest radiograph reveals multiple bilateral nodular densities. Treatment with intravenous vancomycin is continued, What the murmur will be most ikely like? a) Contimious machinery murmur throughout the cardiac cycle b) Decrescendo diastolic murmur heard best at mid precordium c) Faint smunmur that increases in intensity with inspiration 4) Holosystolic murmur with radiation of the murmur to the axilla ©) Opening snap and mid-diastolic rumbling murmur Answer (Banat) Other User's Explanation Report An Error Question Explanation: Most cases of right-sided endocardtis occur among injection drug users. Blood culmures are usually positive in these patients, but the ‘smucnur may be difficult to hear. Further, other stigmata of infective endocarditis usvally are not present, making a high index of suspicion extremely important, Septc pulmonary emboli, seen on this patient's radiograph, indicate that the lesion is on the right side ofthe heart. The characteristic murmur of tricuspid regurgitation is one that is faint increases with inspiration (Carvallo sign), end usually is heard best with the diaphragm placed over the lower sternal border Further, intravenous drug abusers are at higher risk than non-IV drug users for right-sided endocardis. ‘A. continuous machinery murmur starts in systole and extends through diastole without interruption. The cause of a continuous murmur is the rapid and turbulent flow of blood from a higher-pressure chamber or vessel to one of lower pressure, where the pressure gradient contines throughout the cardiac cycle. The most common cause in aduts is a patent ductus arteriosus ‘A. decrescendo diastolis reunmur heard best at the mid precordiwa (choice B) describes the murmur of aortic regurgitation. Thic atient's sick factors (injection drug use) and presertation (ceptic pulmonary emboli) suggest a right-sided heart lesion, ‘A holosystolic murmur with radiation of the murmur to the axilla suggests mitral regurgitation. This is a common presentation of let- sided endocarditis. However, this patient's risk factors (injection drug use) and presenlation (septic pulmonary emboli) suggest a right-sided heart lesion. An opening snap and mid-diastolic rumbling murmur suggest mitral stenosis. It is uncommon for a vegetation to cause @ clinically apparent stenotic lesion, and in any case, this patient presents with right-sided cardiac pathology ‘Mark this question e Question Td : 217086 Question 30 of 30 A.42-year-old IV deug abuser has fever, malaise, and cough, A cardiac murmur exists. Cultures grow methicilln-resistant Staphylococcus aureus and a chest sadiograpia reveals multiple bilateral nodular densities. Treatment with intravenous vancomycin is continued. What the murmur will be most icely lke? a) Continious machinery murmur throughout the cardiac cycle b) Decrescendo diastolic murmur heard best at mid precordium Y © c)Faint murmur that increases in intensity with inspiration 4) Holosystolic murmur with radiation of the murmur to the axilla e) Opening snep and mid-diastolic rumbling murmur Answer (Banat) Other User's Explanation Report An Error Question Explanation: Most cases of right-sided endocardtis occur among injection drug users. Blood culmures are usually positive in these patients, but the ‘smucnur may be difficult to hear. Further, other stigmata of infective endocarditis usvally are not present, making a high index of suspicion extremely important, Septc pulmonary emboli, seen on this patient's radiograph, indicate that the lesion is on the right side ofthe heart. The characteristic murmur of tricuspid regurgitation is one that is faint increases with inspiration (Carvallo sign), end usually is heard best with the diaphragm placed over the lower sternal border Further, intravenous drug abusers are at higher risk than non-IV drug users for right-sided endocardis. ‘A. continuous machinery murmur starts in systole and extends through diastole without interruption. The cause of a continuous murmur is the rapid and turbulent flow of blood from a higher-pressure chamber or vessel to one of lower pressure, where the pressure gradient contines throughout the cardiac cycle. The most common cause in aduts is a patent ductus arteriosus ‘A. decrescendo diastolis reunmur heard best at the mid precordiwa (choice B) describes the murmur of aortic regurgitation. Thic atient's sick factors (injection drug use) and presertation (ceptic pulmonary emboli) suggest a right-sided heart lesion, ‘A holosystolic murmur with radiation of the murmur to the axilla suggests mitral regurgitation. This is a common presentation of let- sided endocarditis. However, this patient's risk factors (injection drug use) and presenlation (septic pulmonary emboli) suggest a right-sided heart lesion. An opening snap and mid-diastolic rumbling murmur suggest mitral stenosis. It is uncommon for a vegetation to cause @ clinically apparent stenotic lesion, and in any case, this patient presents with right-sided cardiac pathology 2/24/2014 1:06:38 PM ‘Mark this question Question 1 of 30 A T1-year-old man presents with an episode of syncope due to marked postural hypotension He takes Felodipine for hypertension for anumber of years and he also takes aspirin. He appears to have taken up a new healthier lifestyle. Which ofthe following health supplements might be responsible? a) Cranberry juice ) Grapefiut juice c) Cod liver oil capsules 4) Ginseng ©) Vitamin C Question Explanation: CGrapefinit juice interacts with drugs. The basis for this interaction hes been diligently explored and appears to relate to both flavanoid and nonflavanoid components of grapefruit juice interfering with enterocyte CYP3A4 activity Of the calcium channel blockers felodipine in particular is affected. 2/24/2014 1:06:38 PM ‘Mark this question Question 1 of 30 A T1-year-old man presents with an episode of syncope due to marked postural hypotension He takes Felodipine for hypertension for anumber of years and he also takes aspirin. He appears to have taken up a new healthier lifestyle. Which ofthe following health supplements might be responsible? a) Cranberry juice Y © b) Grapefiuit juice ¢) Cod liver oil capsules @) Ginseng. e) Vitamin C Question Explanation: CGrapefinit juice interacts with drugs. The basis for this interaction hes been diligently explored and appears to relate to both flavanoid and nonflavanoid components of grapefruit juice interfering with enterocyte CYP3A4 activity Of the calcium channel blockers felodipine in particular is affected. 2/24/2014 1:06:56 PM ‘Mark this question & => Question Td : 48752 Question 2 of 30 The effect of warfarin (Coumadin) is reversed by which of the following? a) Vitamin & ) Vitarnin C ©) Vitemin K 4) Vitasnin D ) Vitamin E Question Explanation: Inpatients whose INR becomes elevated while taking warférin, the INR can be lowered either by withhelding warlaria or by oral or parenteral administration of vitamin K. ation Report An Error 2/24/2014 1:06:56 PM ‘Mark this question & => Question Td : 48752 Question 2 of 30 The effect of warfarin (Coumadin) is reversed by which of the following? a) Vitamin & ) Vitarnin C Y © ©) Vitamin K 4) Vitasnin D ) Vitamin E Question Explanation: Inpatients whose INR becomes elevated while taking warférin, the INR can be lowered either by withhelding warlaria or by oral or parenteral administration of vitamin K. ation Report An Error 2242014 1:07:11 PM ‘Mark this question & => Question 3 of 30 4.52 year old man suffered a myocardil infarction 2 years ago. He has been well since. His non fasting cholesterol level is mmol. The acat step in his management is a) No firthes action ) Obtain a total cholesterol after a 12-hour fast ©) Initiate a cholesterol-lowering diet 4) Obtain a lipid profile (Cholesteral, triglycerides end HDL cholestercl) after a 12 hour fest €) Obtain a lpoprotein electrophoresis Question Explanation: ‘A fil lipid panel is a group of tests that are often ordered together to determine risk of coronary heart disease, The tests that make up alipid profile are tests that have been shown to be good incicators of whether someone is liely to have a heart attack or stroke caused by blockage of blood vessels (hardening of the arteries). The full lipid profile includes total cholesterol, HDL-cholesterol (often called good cholesterol), LDL-cholesterol (often called bad cholesterol), and trigycerides, Sometimes the report will include additional calculated values such as the Cholestero/EDL ratio or a risk score based on bpid profile results, age, sex, and other risk factors. 2242014 1:07:11 PM ‘Mark this question & => Question 3 of 30 4.52 year old man suffered a myocardil infarction 2 years ago. He has been well since. His non fasting cholesterol level is mmol. The acat step in his management is a) No firthes action ) Obtain a total cholesterol after a 12-hour fast ©) Initiate a cholesterol-lowering diet Y © 4) Obtain a lipid profile (Cholesterol, triglycerides and HDL cholestercl) after a 12 hour fost €) Obtain a lpoprotein electrophoresis Question Explanation: ‘A fil lipid panel is a group of tests that are often ordered together to determine risk of coronary heart disease, The tests that make up alipid profile are tests that have been shown to be good incicators of whether someone is liely to have a heart attack or stroke caused by blockage of blood vessels (hardening of the arteries). The full lipid profile includes total cholesterol, HDL-cholesterol (often called good cholesterol), LDL-cholesterol (often called bad cholesterol), and trigycerides, Sometimes the report will include additional calculated values such as the Cholestero/EDL ratio or a risk score based on bpid profile results, age, sex, and other risk factors. 2/24/2014 10727 PM ‘Mark this question & => ‘Question Td : 54309 Question 4 of 30 Ayyoung female presents complains of chest pam, palpitation and fainting, She has supraventricular tachycardia. The best initial management is a) Cardioversion b) IV Verapamil c) Increased vagal tone 4) Beta blocker Question Explanation: Episodes of paroxysmal supraventricular tachycardia often can be stopped by one of several maneuvers that stimulate the vagus nerve and thus decrease the heart rate, These maneuvers are usually conducted or supervised by a doctor, but people who repeatedly experience the arrhythmia often leam to perform the maneuvers themselves. Mansuvers include siraining as if having a dificult bowel movement, rubbing the neck just below the angle ofthe jaw (Which stimulates a sensitive arca onthe carotid artery called the carotid sinus), and plunging the face into a bow! of ice-cold water These maneuvers are most effective when they are used, shortly after the arrhythnnia starts. [Pthese maneuvers are not effective, ifthe arrhythmia produces severe symptoms, or ifthe episode lasts more than 20 minutes, people are advised to seek medical intervention to stop the episode. Doctors can usually stop an episode promptly by giving an intravenous injection of a drug, usvally adenosine or verapamil 2/24/2014 10727 PM ‘Mark this question & => ‘Question Td : 54309 Question 4 of 30 Ayyoung female presents complains of chest pam, palpitation and fainting, She has supraventricular tachycardia. The best initial management is a) Cardioversion Y¥ © b)IV Verapamil ) Increased vagal tone 4) Beta blocker Question Explanation: Episodes of paroxysmal supraventricular tachycardia often can be stopped by one of several maneuvers that stimulate the vagus nerve and thus decrease the heart rate, These maneuvers are usually conducted or supervised by a doctor, but people who repeatedly experience the arrhythmia often leam to perform the maneuvers themselves. Mansuvers include siraining as if having a dificult bowel movement, rubbing the neck just below the angle ofthe jaw (Which stimulates a sensitive arca onthe carotid artery called the carotid sinus), and plunging the face into a bow! of ice-cold water These maneuvers are most effective when they are used, shortly after the arrhythnnia starts. [Pthese maneuvers are not effective, ifthe arrhythmia produces severe symptoms, or ifthe episode lasts more than 20 minutes, people are advised to seek medical intervention to stop the episode. Doctors can usually stop an episode promptly by giving an intravenous injection of a drug, usvally adenosine or verapamil 2/24/2014 1:07:41 PM. ‘Marre this question << => (Question Id : 54399 Question 5 of 30 A young hypertensive patient is found to have on physical exam bilaeral femoral bruits. A secondary cause for her hypertension is suspected, The most likely canse is a) Hyperaldosteronis. b) Cushing syndrome ©) Aortic Coarctation ) Renal artery stenosis ©) Phaeochromocytoma Question Explanation: Renal artery stenosis (RAS) is the narrowing of the fining of the main artery that supplies the kidney Depending on the degree of narrowing, patient's can develop hypertension celled renal vascular hypertension (RVE). This form of hypertension is the most common cause of secondary hypertension. There may be the presence of a bruit (sound or murmur heard with a stethoscope) in the abdomen (¢.g, groin), neck, or other area Renal ultrasound may indicate that the kidney has decreased in size, or there is a decrease in blood flow through the artery because the artery has narrowed. 2/24/2014 1:07:41 PM. ‘Marre this question << => (Question Id : 54399 Question 5 of 30 A young hypertensive patient is found to have on physical exam bilaeral femoral bruits. A secondary cause for her hypertension is suspected, The most likely canse is a) Hyperaldosteronis. b) Cushing syndrome ©) Aortic Coarctation o © d) Renal artery stenosis ©) Phaeochromocytoma Question Explanation: Renal artery stenosis (RAS) is the narrowing of the fining of the main artery that supplies the kidney Depending on the degree of narrowing, patient's can develop hypertension celled renal vascular hypertension (RVE). This form of hypertension is the most common cause of secondary hypertension. There may be the presence of a bruit (sound or murmur heard with a stethoscope) in the abdomen (¢.g, groin), neck, or other area Renal ultrasound may indicate that the kidney has decreased in size, or there is a decrease in blood flow through the artery because the artery has narrowed. 2/24/2014 1:07:54 PM ‘Mark this question & => Question Td : 54661 Question 6 of 30 A.25 year old man has 5 year history of hypertension that is refractory to medical therapy. He became increasingly lethargic and confused over past few days. He was found in his home lying on floor unresponsive and was brought to the ER. His Na is 112mmolfL ON. 135-147mmoVL), K is 5. mmol QV. 3.5-Smmol/L) and osmolality is 230mmolkg QV. 280-300mmolkg). Substance causing hyponatremia ss a) Aldosterone ) Antidiuretic hormone (ADED) ©) Cottisel ) Somatostatin ¢) Thyroid stimulating hormene (TSE) Question Explanation: Hyponatremiais the most commen electrolyte abnormality encountered in the hospital. This patient is clearly hyponatremic, ADH works at the collecting ducis of the nephron in the kidney to bring back water from the urine into the bloodstream. This causes both the serum osmolality and serum sodium to drop as the serum g becomes more diute Recall that aldosterone brings back sodium from the urine into the bloodstream and kicks out serum potassium into the urine ‘Therefore, high levels of aldosterone would present as hypokalemia with hypematremia, 2/24/2014 1:07:54 PM ‘Mark this question & => Question Td : 54661 Question 6 of 30 4.25 year old man has 5 year history of hypertension that is refractory to medical therapy. He became increasingly lethargic and confused over past few days. He was found in his home lying on oor unresponsive and was brought to the ER. His Na is 112mmoV/L QW: 135-14?mmolL), K is 5. mmol (N: 3.5-Smmol/L) and osmolality is 230mmol/kg (W: 280-300mmol/kg). Substance causing hyponatremia is a) Aldosterone Y¥ © ’) Antidiuretic hormone (ADH) ©) Cortisol d) Somatostatin ¢) Thyroid stimulating hormone (TSH) Question Explanation: Hyponatremiais the most commen electrolyte abnormality encountered in the hospital. This patient is clearly hyponatremic, ADH works at the collecting ducis of the nephron in the kidney to bring back water from the urine into the bloodstream. This causes both the serum osmolality and serum sodium to drop as the serum g becomes more diute Recall that aldosterone brings back sodium from the urine into the bloodstream and kicks out serum potassium into the urine ‘Therefore, high levels of aldosterone would present as hypokalemia with hypematremia, 2/24/2014 1:08:08 PM ‘Maric this question <& => i Question 7 of 30 70 year male presents to your clinical in atrial Gibrillation with a rate of 132beatsimin, He has hypertension, but no history of congestive heart failure or structural heart disease. He is otherwise healthy and active. What would be the best initial appropriate management of his atrial brillation? a) Rhythm control with antiarrhythmics and warfarin (Coumadin) only fhe cannot be consistently maintained in sinus rythm b) Rhythm control with antiarrythmics and warfarin regardless of maintenance of sinus shyt ©) Ventricular rate control with digoxin, and warfarin for anticoagulation 4) Veatricular rate control with digoxin ard aspirin for anticoagulation ©) Ventricular rate control with a calcium channel blocker or Beta blocker, and warfarin for anticoagulation Question Explanation: ‘ive recent randomized, controlled trials have ndicated that in most patients with atrial fibrillation, an initial approach of rate control is best. Patients who were stratified to the rhythm control arm of the trials did NOT have a morbidity or mortality benefit and were more likely to suffer from adverse drug effects and increased hospitalzatons. ‘The most efficacious drugs for rate control are calcium charnel blockers and Beta blockers. Digoxin is less effective for rate control and should be reserved as an add-on option for those not controled with a Beta blocker or calcium channel blocker, or for patients with significant leit ventricular systolic dysfimction. In patients 65 years of age or older or with one of more risk factors for stroke, the best choice for anticoagulation to prevent thromboembolic disease is warfarin. Ofnote, in patients who are successfully rhythm controlled and maintained in sinus rhythm, the thromboembolic rate is equivalent to those managed with arate control strategy. Thus, the data suggest that patients who choose a rhythm control strategy should be maintained on anticoagulation regardless of whether they are consistently in sinus rhythm, 2/24/2014 1:08:08 PM ‘Maric this question <& => i Question 7 of 30 70 year male presents to your clinical in atrial Gibrillation with a rate of 132beatsimin, He has hypertension, but no history of congestive heart failure or structural heart disease. He is otherwise healthy and active. What would be the best initial appropriate management of his atrial brillation? a) Rhythm control with antiarrhythmics and warfarin (Coumadin) only fhe cannot be consistently maintained in sinus rythm b) Rhythm control with antiarrythmics and warfarin regardless of maintenance of sinus shyt ©) Ventricular rate control with digoxin, and warfarin for anticoagulation 4) Veatricular rate control with digoxin ard aspirin for anticoagulation Y © e) Ventricular rate control with a calcium channel blocker or Beta blocker, and warfarin for anticoagulation Question Explanation: ‘ive recent randomized, controlled trials have ndicated that in most patients with atrial fibrillation, an initial approach of rate control is best. Patients who were stratified to the rhythm control arm of the trials did NOT have a morbidity or mortality benefit and were more likely to suffer from adverse drug effects and increased hospitalzatons. ‘The most efficacious drugs for rate control are calcium charnel blockers and Beta blockers. Digoxin is less effective for rate control and should be reserved as an add-on option for those not controled with a Beta blocker or calcium channel blocker, or for patients with significant leit ventricular systolic dysfimction. In patients 65 years of age or older or with one of more risk factors for stroke, the best choice for anticoagulation to prevent thromboembolic disease is warfarin. Ofnote, in patients who are successfully rhythm controlled and maintained in sinus rhythm, the thromboembolic rate is equivalent to those managed with arate control strategy. Thus, the data suggest that patients who choose a rhythm control strategy should be maintained on anticoagulation regardless of whether they are consistently in sinus rhythm, ‘Mark this question <> Question Id : 66813 Question 8 of 30 An ECG strip seveals a shythm with iegularly regular ventricular complexes, which are narrow in nature (QRS duration 0.08 sec) There is an undulating baseline with an arial rate somewhere in the 300-500 beat per minute range. The most likely rhythm for this strip is a) Atrial Futter. b) Muttifocal atrial tachycardia c) Atvial brillation 4) Ventricular tachycardia, ©) Ventricular flutter. Answer | Belanation | Other User's Explanation Report An Error Question Explanation: “Atrial Gbrllation presents with irregularly irregular ventricular complexes that are narrow in nature (QRS duration 0 .08 sec). There is an undulating baseline with an attial rate somewhere in the 300-500beat per minute range. P waves may or may not be presenting (Coarse vs. fine a fib), Atrial fitter is usually a regular rhythm (unless there is varying block) and usually has an atriel rate of -250. ‘Multifocal tachycardia usually has an atrial rate in the low 100s, ‘The ventricular rhythms mentioned would not have narrow QRS complexes. ‘Mark this question <> Question Id : 66813 Question 8 of 30 An ECG strip seveals a shythm with iegularly regular ventricular complexes, which are narrow in nature (QRS duration 0.08 sec) There is an undulating baseline with an arial rate somewhere in the 300-500 beat per minute range. The most likely rhythm for this strip is a) Atrial Futter. b) Muttifocal atrial tachycardia Y © ©) Atrial fibrillation. 4) Ventricular tachycardia, ©) Ventricular flutter. Answer | Belanation | Other User's Explanation Report An Error Question Explanation: “Atrial Gbrllation presents with irregularly irregular ventricular complexes that are narrow in nature (QRS duration 0 .08 sec). There is an undulating baseline with an attial rate somewhere in the 300-500beat per minute range. P waves may or may not be presenting (Coarse vs. fine a fib), Atrial fitter is usually a regular rhythm (unless there is varying block) and usually has an atriel rate of -250. ‘Multifocal tachycardia usually has an atrial rate in the low 100s, ‘The ventricular rhythms mentioned would not have narrow QRS complexes. ‘Mark this question = => Question Td : 78458 Question 9 of 30 A.28-year-old man presents with palpitations and his EKG shows a PR interval of 0.08 seconds. What syndrome does this patient have? a) Hypokalemia. 6) Acute theumatic fever ©) Romano-Ward syndrome 4) Wolff Parkinson-White syndrome ¢) First Degree Heart Block, Ancwer [REIHIRRINY Other Users Explanation Repos An Exon (Question Explanation: Romano werd is a prolonged QT syndrome associated with producing severe ventricular dysrhythmias. First degree heatt block, hypokalemia and acute rheumatic fever, all increase PE interval, Wlff-Parkinson- White syndrome is caused by an accessory bypass tract (Bundle of Kent) and may cause supraventricular tachycardias, a delta wave is seen, along with a shoriened PR snterval (normal is 0.12-0.20 seconds) ‘Mark this question = => Question Td : 78458 Question 9 of 30 A 28-year-old maa presents wih palpitations and his EKG shows aPR interval of 0.08 seconds, What syndrome dors this patient hevet? a) Hypokalemia ») Acute rheumatic fever. ©) Romano-Ward syndrome © & Woif-Parkinson-White syndrome ¢) First Degree Heart Block. Ancwer [REIHIRRINY Other Users Explanation Repos An Exon (Question Explanation: Romano werd is a prolonged QT syndrome associated with producing severe ventricular dysrhythmias. First degree heatt block, hypokalemia and acute rheumatic fever, all increase PE interval, Wlff-Parkinson- White syndrome is caused by an accessory bypass tract (Bundle of Kent) and may cause supraventricular tachycardias, a delta wave is seen, along with a shoriened PR snterval (normal is 0.12-0.20 seconds) 2/24/2014 1:08:55 PM ‘Mare this question = => Question Td : 81046 Question 10 of 30 “Which one of the following is not indicated in the treatment of acute right ventricular myocardial infarction? a) Diwetics b) Dobutamine, c) Dopamine. 4) Volume expansion. ©) Aspirin. Answer | Explanation Other User's Explanation Report An Error Question Explanation: Right ventricular (RV) infarction causes poor RV function, which decreases LV fillng and thus drops cardiac output, Diuretics will firther decrease right-sided fillng pressures, causing a further crop in cardiac output, and are absolutely contraindicated, Dobutamnine (@) and dopamine (C) are inotropic agents and are usefUl in maintaining cardiac ouput. Volume expansion increases right-sided fillng pressures and increases cardiac output (D). Aspirin (E) has been shown to be beneficial in all acute myocardial infarchons (in the ISIS-2 trial, mortality was reduced by 25%). 2/24/2014 1:08:55 PM ‘Mare this question = => Question Td : 81046 Question 10 of 30 “Which one of the followng is not indicated in the treatment of acute right ventricular myocarsial infarction? Y © a) Dinwetics b) Dobutamine, c) Dopamine. ) Votume expension ©) Aspirin. Answer | Explanation Other User's Explanation Report An Error Question Explanation: Right ventricular (RV) infarction causes poor RV function, which decreases LV fillng and thus drops cardiac output, Diuretics will firther decrease right-sided fillng pressures, causing a further crop in cardiac output, and are absolutely contraindicated, Dobutamnine (@) and dopamine (C) are inotropic agents and are usefUl in maintaining cardiac ouput. Volume expansion increases right-sided fillng pressures and increases cardiac output (D). Aspirin (E) has been shown to be beneficial in all acute myocardial infarchons (in the ISIS-2 trial, mortality was reduced by 25%). ‘Maric this question << => Question Td : 82290 Question 11 of 30 For which of the following groups the initial screening of serum cholesterol is recommended and is most usefil? a) Newborns b) Patients who have had a recent myocardial infraction ©) All adults 4) Postmenepansal women ©) Adolescents Question Explanation: The current recommencation for cholesterol screening is measurement of non-fasting serum cholesterol in all adults. Testing newboms (A) has not been found to be helpful. Patients with a recent MI (B) should be tested, but these patents should not be recetving their initial screening at this point, which is too late to modify their risks for the development of atherosclerosis, Postmenopausal women (D) should not be undergoing their intial screening. Adolescents are too young (E) unless they have avery strong family history of atherosclerosis or have other signficant risk factors. ‘Maric this question << => Question Td : 82290 Question 11 of 30 For which of the following groups the initial screening of scram cholesterol is recommended and is most useful? a) Newborns 'b) Patients who have had a recent myocardial infraction Y © 6) All adults d) Postmenopausal women €) Adolescents Question Explanation: The current recommencation for cholesterol screening is measurement of non-fasting serum cholesterol in all adults. Testing newboms (A) has not been found to be helpful. Patients with a recent MI (B) should be tested, but these patents should not be recetving their initial screening at this point, which is too late to modify their risks for the development of atherosclerosis, Postmenopausal women (D) should not be undergoing their intial screening. Adolescents are too young (E) unless they have avery strong family history of atherosclerosis or have other signficant risk factors. 272472014 1:09:22 PM ‘Mark this question & => Question Td : 84584 Question 12 of 30 “Which of the following is a major risk factor for atherosclerosis? a) Penpheral arterial disease ») High EDL ©) Hypertension 4) Low homocysteine levels ¢) Increases estrogen Avewor (NEQNONRANY other UcersExptanation Report An Enos Question Explanation: ‘Hypertension is major risk factor for the development of atherosclerosis and the consequent ischemic heart disease. Other major risk factors include hyperipidemia, smokmg, and diabetes. Minor sisk factors: male gender obesity sedentary Ife stress (type A. personaity), elevated homocysteine levels oral contraceptive use, increasing age, and familal/genetic factors, Peripheral arteral disease (choice A) is a consequence of atherosclerosis and therefore is not a risk factor because the disease is aiready present. High HDL levels (choice B) is protective against atherosclerosis, “Ahigh (aotlow, choice D) homocysteine level is considered a risk factor for atherosclerosis. Betrepeies Clegion i ceronsi Gorell peco-chige apiinet dihernseiars ida on inchenve tear deeace: 272472014 1:09:22 PM ‘Mark this question & => Question Td : 84584 Question 12 of 30 “Which of the following is a major risk factor for atherosclerosis? a) Penpheral arterial disease ») High EDL JY © ©) Hypertension 4) Low homocysteine levels ¢) Increases estrogen Avewor (NEQNONRANY other UcersExptanation Report An Enos Question Explanation: ‘Hypertension is major risk factor for the development of atherosclerosis and the consequent ischemic heart disease. Other major risk factors include hyperipidemia, smokmg, and diabetes. Minor sisk factors: male gender obesity sedentary Ife stress (type A. personaity), elevated homocysteine levels oral contraceptive use, increasing age, and familal/genetic factors, Peripheral arteral disease (choice A) is a consequence of atherosclerosis and therefore is not a risk factor because the disease is aiready present. High HDL levels (choice B) is protective against atherosclerosis, “Ahigh (aotlow, choice D) homocysteine level is considered a risk factor for atherosclerosis. Betrepeies Clegion i ceronsi Gorell peco-chige apiinet dihernseiars ida on inchenve tear deeace: 272472014 1:09:40 PM ‘Maric this question & => Question Td : 86884 Question 13 of 30 A55 year old women presents te ths emergency department after a fall, No fractures are presert on skeletal roentgenograms. Serum chemistry studies reveal thet her aspartate aminotransferase (AST) is markedly elevated, while her alanine aminotransferase (ALT), gamma-gliamyl transpeptidase (GGT), and alkaline phosphatase are all within normal limits. This pattern of serum enzymes is most likely caused by which one ofthe following? a) Colitis b) Duodenal tear ©) Hepatitis 4) Ischemic heart disease e) Pancreatitis Anewor [NEQGIRAN) otherucors explanation Repost An Eros Question Explanation: ‘Myocardial infarction (MII) can cause AST elevation without accompanying elevation of ALT or other liver enzymes. This is an important fact to remember because it may be the first clue for heart disease in a patient who has a typical presentation of MI (as is common in women with Ml). Ml can be confirmed with measurement of the ME fraction of creatine phosphokinase (CPK-MB) Unfortunately, diseases of the tubular organs of the gastrointestinal tract, including the colon and duodenum, do not produce distinctive serum enzyme patterns. Hepatitis will have abnormally elevated levels of AST, ALT, and bilirubin. Damage to the pancreas is associated with elevated amylase and lipase levels. 272472014 1:09:40 PM ‘Maric this question & => Question Td : 86884 Question 13 of 30 A.55 year old women presents to the emergency department after a fal. No fractures are present oa skeletal rocntgenograms. Serum chemistry studies reveal thet her aspartate aminotransferase (AST) is markedly elevated, while her alanine aminotransferase (ALT), gamma-glutamyl transpeptidase (GGT), and alkaline phosphatase are all within normal limits. This pattern of serum enzymes is most likely caused by which one of the following? a) Colitis b) Duodenal tear ©) Hepattis UV © Alschemic heart disease ©) Pancreatitis Anewor [NEQGIRAN) otherucors explanation Repost An Eros Question Explanation: ‘Myocardial infarction (MII) can cause AST elevation without accompanying elevation of ALT or other liver enzymes. This is an important fact to remember because it may be the first clue for heart disease in a patient who has a typical presentation of MI (as is common in women with Ml). Ml can be confirmed with measurement of the ME fraction of creatine phosphokinase (CPK-MB) Unfortunately, diseases of the tubular organs of the gastrointestinal tract, including the colon and duodenum, do not produce distinctive serum enzyme patterns. Hepatitis will have abnormally elevated levels of AST, ALT, and bilirubin. Damage to the pancreas is associated with elevated amylase and lipase levels. 2/24/2014 1:10:01 PM ‘Maz this question = => Question Td : 87603 Question 14 of 30 A newbom infant's X-ray shows enlargement of the left ventricle and left atrium as well as dilatation ofthe aorta, Echocardiography demonstrates volume overloading of the left ventricle. Presence ofa continuous murmur is noted on cardiac auscultation, The infant is ‘most likely suffering irom which one of the following? a) Atrial septal detect 'b) Patent ductus arteriosus ©) Pulnonic stenosis 4) Tetralogy of allot ©) Ventricular septal detect Question Explanation: Patent ductus arteriosus (PDA) is a congenital cardiac disorder in which blood traveling in the corta is shunted through the ductus arteriosus to the pulmonary arteries. On X-ray, the left ventricle and left atrium may be enlarged and pulmonary hypertension may be observed PDA is characterized by @ continuous ‘machinery" rmarmur on auscultation. TFthe ductus is widely patent, pulmonary hypertension may eventually develop and the initielly left to sight shunt is reversed sending deoxygenated blood through the descending acrta and producing cyanosis Gisenmenger syndrome). Since the deoxygenated blood enters the descending aorta, the toes can be cyanotic but the fingers are generally not, In atrial septal defect, left to right shunting causes volume overloading of the ight ventricle, the increased flow across the pulmonic valve producing a midsystolic pulmonary ejection murmur. The second heart sound is widely split, A diastolic rmurmur may also be heard, reflecting increased flow from the right atrium into the right ventuicle. Pulmonic stenosis typically produces a harsh systolic ejection murmur best heard at the upper left sternal border, often preceded by a systolic ejection sound, Tetralogy of Falotis a formn of cyanotic congenital heart disease characterized by ventricular septal defect, ‘ght ventricular outflow tract cbstruction, an overriding aorta, and right ventiicular hypertrophy. The heart is often described as “boot shaped’ on chest X-ray A ventricular septal defect would produce an initial left-to-right shunt, characterized by a holosystolc ‘murmur, and increased pulmonary vescularty on chest X-ray. A mid-diastolic rumble may also be heard. 2/24/2014 1:10:01 PM ‘Maz this question = => Question Td : 87603 Question 14 of 30 A newbom infant's X-ray shows enlargement of the left ventricle and left atrium as well as dilatation ofthe aorta, Echocardiography demonstrates volume overloading of the left ventricle. Presence ofa continuous murmur is noted on cardiac auscultation, The infant is ‘most likely suffering irom which one of the following? a) Atrial septal detect Y © b) Patent ductus arteriosus ©) Pulnonic stenosis 4) Tetralogy of allot ©) Ventricular septal detect Question Explanation: Patent ductus arteriosus (PDA) is a congenital cardiac disorder in which blood traveling in the corta is shunted through the ductus arteriosus to the pulmonary arteries. On X-ray, the left ventricle and left atrium may be enlarged and pulmonary hypertension may be observed PDA is characterized by @ continuous ‘machinery" rmarmur on auscultation. TFthe ductus is widely patent, pulmonary hypertension may eventually develop and the initielly left to sight shunt is reversed sending deoxygenated blood through the descending acrta and producing cyanosis Gisenmenger syndrome). Since the deoxygenated blood enters the descending aorta, the toes can be cyanotic but the fingers are generally not, In atrial septal defect, left to right shunting causes volume overloading of the ight ventricle, the increased flow across the pulmonic valve producing a midsystolic pulmonary ejection murmur. The second heart sound is widely split, A diastolic rmurmur may also be heard, reflecting increased flow from the right atrium into the right ventuicle. Pulmonic stenosis typically produces a harsh systolic ejection murmur best heard at the upper left sternal border, often preceded by a systolic ejection sound, Tetralogy of Falotis a formn of cyanotic congenital heart disease characterized by ventricular septal defect, ‘ght ventricular outflow tract cbstruction, an overriding aorta, and right ventiicular hypertrophy. The heart is often described as “boot shaped’ on chest X-ray A ventricular septal defect would produce an initial left-to-right shunt, characterized by a holosystolc ‘murmur, and increased pulmonary vescularty on chest X-ray. A mid-diastolic rumble may also be heard. 2/24/2014 1:10:26 PM ‘Mack this question = => ‘Question Id : 88108 Question 15 of 30 An autopsy specimen from a Mexican immigrant demonstrates a heart with massive dilation of the aortic root and adjacent aortic arch. Opening the aorta reveals a distinctive wrinkling of the intimal surface. On making a histological section through the aortic wall which of the following would most lkely be seen? a) Aheavy eosinophilic infiltrate +b) Fibrinoid necrosis with a neutrophilic infitration c) Focal fragmentation of elastic elements d) Obliterative endartentis of the vasa vasorum c) Ringlike calcification of the vessel media Question Explanation: Massive dilation of the aostic root with an absence of atherosclerotic vessel lesions strongly suggests a syphiltic aneurysm. These aneurysms are a manifestation of tertiary syphilis and have become very uncommon now, probably due to a combination of deliberate therapy and therapy of undiagnosed disease when antibiotics are given for some other condition, The tistological hallmark of the syphilitc eneurysm is a plasma-cell lesion of the small blood vessels (the vasa vasorum) that supply the aorta and eventually obliterate the small vessel lamina, Tree-barking is another name for the wrinkling of the aorta that can occur as a consequence of post- inflemmatory scarring in syphilitic aortic lesions heavy eosinophilic infiltrate ic a feature of Churg-Strauss eyndcome, which ie a veriant of polyarteritic nodosa that involves vessels smaller than the aorta. Fibsincid necrosis with a neutrophilic inflation is a fecture of polyerteritis nodosa, which involves vessels smaller than the aorta Focal fragmentation of elastic elements is a feature of cystic mecial necrosis, which can cause acrtic dissection, Ringlike calcification of the vessel media is @ feanure of Monckeberg arteriosclerosis, which involves vessels, smaller than the aorta 2/24/2014 1:10:26 PM ‘Mack this question = => ‘Question Id : 88108 Question 15 of 30 An autopsy specimen from a Mexican immigrant demonstrates a heart with massive dilation of the aortic root and adjacent aortic arch. Opening the aorta reveals a distinctive wrinkling of the intimal surface. On making a histological section through the aortic wall which of the following would most lkely be seen? a) Aheavy eosinephilic inflate +p) Fibtinoid necrosis with a neutrophilic infitration c) Focal fragmentation of elastic elements SY © A) Obliterative endarteritis of the vasa vasorum c) Ringlike calcification of the vessel media Question Explanation: Massive dilation of the aostic root with an absence of atherosclerotic vessel lesions strongly suggests a syphiltic aneurysm. These aneurysms are a manifestation of tertiary syphilis and have become very uncommon now, probably due to a combination of deliberate therapy and therapy of undiagnosed disease when antibiotics are given for some other condition, The tistological hallmark of the syphilitc eneurysm is a plasma-cell lesion of the small blood vessels (the vasa vasorum) that supply the aorta and eventually obliterate the small vessel lamina, Tree-barking is another name for the wrinkling of the aorta that can occur as a consequence of post- inflemmatory scarring in syphilitic aortic lesions heavy eosinophilic infiltrate ic a feature of Churg-Strauss eyndcome, which ie a veriant of polyarteritic nodosa that involves vessels smaller than the aorta. Fibsincid necrosis with a neutrophilic inflation is a fecture of polyerteritis nodosa, which involves vessels smaller than the aorta Focal fragmentation of elastic elements is a feature of cystic mecial necrosis, which can cause acrtic dissection, Ringlike calcification of the vessel media is @ feanure of Monckeberg arteriosclerosis, which involves vessels, smaller than the aorta ‘Mack this question e& => Question Td : 90466 Question 16 of 30 All of the following are associated with erythrocytosis, EXCEPT a) Tetralogy of Fallot b) Citthosis of the liver c) Chronic mountain sickness 4) Renal cell carcinoma €) Psoriasis Question Explanation: Excessive ust of coal tar derivatives, which cause production of abnormal hemoglobin, sesults in erythrocytosis. However, in the absence of such therapy psoriasis does not result in erythrocytosis. Exythrocytosis refers to an increase in the hemoglobin concentration which is ouside the normal range. Chronic mourtain sickness may occur following several years of residence at high altindes. Hyposta resus from alveolar hypoventilation superimposed on a lowered inspired ©2 concentration. Hyperventlation, erythrocytosis, increased pulmonary capillary biood volume, increased diffusing capecity and an increased cardiac output occur to compensate for the hypoxia In cardiac diseases associated with right to left shunting of blood, erythrocytosis develops in response to the hypoxic stimulus. Although rare, intrapulmonary shunts of right to left shunts from the portal to the pulmonary veins are associated, with hypozia, Any tumor which produces erythropoietin or erythropoietin like substances, such as hemangoblastomas, renal tumors, vtenne myomas, hepatocellular carcinomas, and pheochromocytomas, are associated with erythrocytosis. ‘Mack this question e& => Question Td : 90466 Question 16 of 30 Al ofthe following are associated with erythrocytosis, EXCEPT a) Tetralogy of Fallot 'b) Cirrhosis of the liver c) Chronic mountain sickness 4) Renal call carcinoma Y © e)Pooriasis Question Explanation: Excessive ust of coal tar derivatives, which cause production of abnormal hemoglobin, sesults in erythrocytosis. However, in the absence of such therapy psoriasis does not result in erythrocytosis. Exythrocytosis refers to an increase in the hemoglobin concentration which is ouside the normal range. Chronic mourtain sickness may occur following several years of residence at high altindes. Hyposta resus from alveolar hypoventilation superimposed on a lowered inspired ©2 concentration. Hyperventlation, erythrocytosis, increased pulmonary capillary biood volume, increased diffusing capecity and an increased cardiac output occur to compensate for the hypoxia In cardiac diseases associated with right to left shunting of blood, erythrocytosis develops in response to the hypoxic stimulus. Although rare, intrapulmonary shunts of right to left shunts from the portal to the pulmonary veins are associated, with hypozia, Any tumor which produces erythropoietin or erythropoietin like substances, such as hemangoblastomas, renal tumors, vtenne myomas, hepatocellular carcinomas, and pheochromocytomas, are associated with erythrocytosis. 2/24/2014 1:10:54 PM ‘Mare this question = => Question Td : 90854 Question 17 of 30 “Which one of the followmng statements is FALSE regarding transient bacteremia predisposing to bacterial endocarditis? a) There is alow number of organisms per milliliter of blood. +) Its intensity i related to the degree of trauma, c) Ibis usually less than 30 minutes in uretion. 4) It may result om brushing of the teeth. ©) The intensity is unrelated to the presence ofinflammation at the ste of reucosal injury Question Explanation: ‘Transient bacteremia is one of many factors which predispose to bacterial infection. This bacteremia is typified by a low mmber of ongenistas per mililiter of blood, The bacteremia characteristically lasts between 15 and 30 minutes. The intensity of the bacteremia is directly related to the degree of trauma to the mucosal or skin surface, the density of the microbial flora, and the presence of inflammation ot infection at the site of injary Minor trauma to the gingival crevice, resulting from eating herd candy or brushing the teeth, is thought to be a predisposing factor in the majority of persons with endocarditis who are unable to recall a traumatic event. 2/24/2014 1:10:54 PM ‘Mare this question = => Question Td : 90854 Question 17 of 30 “Which one of the followmng statements is FALSE regarding transient bacteremia predisposing to bacterial endocarditis? a) There is alow number of organisms per milliliter of blood. +) Its intensity i related to the degree of trauma, c) Ibis usually less than 30 minutes in uretion. 4) It may result om brushing of the teeth. Y © ®) The intensity is unrelated to the presence of inflammation at the site of mucosal inuury. Question Explanation: ‘Transient bacteremia is one of many factors which predispose to bacterial infection. This bacteremia is typified by a low mmber of ongenistas per mililiter of blood, The bacteremia characteristically lasts between 15 and 30 minutes. The intensity of the bacteremia is directly related to the degree of trauma to the mucosal or skin surface, the density of the microbial flora, and the presence of inflammation ot infection at the site of injary Minor trauma to the gingival crevice, resulting from eating herd candy or brushing the teeth, is thought to be a predisposing factor in the majority of persons with endocarditis who are unable to recall a traumatic event. 2/24/2014 1:11:13 PM ‘Mark this question & => Question 18 of 30 A6 year old man comes to you for anew patient vist. He is asymptomatic and has aot scen a doctor in 10 years. Hs does not smoke or dink and takes no medication. He says he has a history of "mild high blood pressure" but has never been treated for this His blood pressure today is 180/90 mmilg, He hes a decreased arteriovenous ratio on findoscopic examination, his point of maximal intensity is displaced laterally, and he has decreased pedal pulses. What is the most appropriate management at this point? a) Perform two blood pressure measurements 1 week apart to establish the diagnosis of hypertension b) Order ambulatory blood pressure monitoring ©) Order a laboratory workup to rule out causes of secondary hypertension 9) Prescribe a duretic, Question Explanation: Elevated blood pressure along with physical findings of cardiovascular disease establishes the diagnosis of hypertension in this patient, so itis not necessary to take follow up blood pressure readings prior to starting treatment, Since he has no symptoms or physical findings suggestive of secondary hypertension it is also not necessary to perform a laboretory workup prior to treatment. Because he has Stage 3 hypertension with evidence of end-organ disease, treatment with antihypertensives is indicated at this point. At least three Jarge clinical tials, including tale European Working Party on High Blood Pressure in the Elderly (EWPHE) trial, have shown that duretics are the most effective single agents for hypertension in the elderly. A low sodium dist can be added, as can a beta blocker if the hypertension fails to respond to diuretics alone. nn Report An Error 2/24/2014 1:11:13 PM ‘Mark this question & => Question 18 of 30 A 66 year old man comes to you for a new patient visit. He is asymptomatic and has act seen a dector in 10 years Hs does not smoke or drink and takes no medication He says he has a history of “mild high blood pressure" but has never been treated for this His blood pressure today is 180/90 mul, He has a decreased arteriovenous ratio on findoscopic examination, his point of maximal intensity is displaced laterally, and he has decreased pedal pulses. What is the most appropriate management at this point? a) Perform two blood pressure measurements 1 week apart to establish the diagnosis of hypertension b) Order ambulatory blood pressure monitoring ©) Order a lcboratory workup to nile out causes of secondary hypertension Y © dD) Prescribe a diuretic Question Explanation: Elevated blood pressure along with physical findings of cardiovascular disease establishes the diagnosis of hypertension in this patient, so itis not necessary to take follow up blood pressure readings prior to starting treatment, Since he has no symptoms or physical findings suggestive of secondary hypertension it is also not necessary to perform a laboretory workup prior to treatment. Because he has Stage 3 hypertension with evidence of end-organ disease, treatment with antihypertensives is indicated at this point. At least three Jarge clinical tials, including tale European Working Party on High Blood Pressure in the Elderly (EWPHE) trial, have shown that duretics are the most effective single agents for hypertension in the elderly. A low sodium dist can be added, as can a beta blocker if the hypertension fails to respond to diuretics alone. nn Report An Error 2/24/2014 1:11:28 PM. ‘Mark this question & => Question Td : 94363 Question 19 of 30 A young patient has a ventricular Sbrilation arrhythmia, He is treated with Procainamide. His blood pressure then decreases to 80/60 mmilg, You ry another time with Procainamide, his blood pressure sill decreases. The appropriate step at this time is which of the following? a) Benzopine ) Saline perfusion ©) Digitalis ) Intubation e) Defibrillation Anewor (NEQISIRN) other Users Explanation Report An Eror Question Explanation ‘Ventricular fiorilation produces uncoordinated quivenng of the ventricle with no useful contractions, It causes immediate syncope and death within minutes. Treatment is with cardiopulmonary resuscitation, including immediate defibrillation. 2/24/2014 1:11:28 PM. ‘Mark this question & => Question Td : 94363 Question 19 of 30 ‘A young patienthas a ventricular Gibrlation arrhythmia. He is treated with Frocainanide, His blood pressure then decreases to 80/60 mmilg You iry another tine with Procainamide, his blood pressure stil decreases. The appropriate step at this time is which ofthe following? a) Benziropine +) Saline perfision ©) Digitalis Intubation Y © e) Defibrillation Anewor (NEQISIRAN) Other Users Explanation Report An Eror Question Explanation ‘Ventricular fiorilation produces uncoordinated quivenng of the ventricle with no useful contractions, It causes immediate syncope and death within minutes. Treatment is with cardiopulmonary resuscitation, including immediate defibrillation. ‘Marie this question m= (Question Id : 95472 Question 20 of 30 A.40) year old man presents to emergency department because of several hours of progressively increasing chest pain, History of hhypercholesterolemia, angixal episodes, and diabetes is present. Although previous episodes were relieved by sublngval nitroglycerin, the man states that it has nct helped with the pain durng this episode. EKG does not show any ST or T-wave changes, and CK-MB and troponns are within normal limits, Which of the following caused this condition? a) Atherosclerosis alone b) Coronary artery embolism c) Coronary artery spasm. 4) Thrombosis with or without underlying atherosclerosis ©) Vasculitis, Question Explanation: This patient presents with unstable angina, which is defined as recurrent episodes of angina on minimal effort or at rest that is no longer relieved by nitroglycerin, It can be referred to as a crescendo angina, and may or may not be associated with ST or T wave depression Cardiac enzymes (CK-MB.As and troponins) do not become clevated in patient's with this type of angna. Unstable angina is thought to be duc to a slowly developing, thrombosis in 2 coronary ariery branch. The thrombosis may or may not occur over an area of the vessel involved by atherosclerotic plaque, Atherosclerosis alone usually causes stable angina occurring with, exenion. Coronary artery embolism is uncommon but can occur ita plaque at the aortic otifice fragments and is driven into @ coronary artery. It would present with signs and symptoms of an acute myocardial infarction, with associated ST segment changes and T-wave changes. Coronary artery spasm is thought to cause angina at rest (Prinzmetal angina). This type of angina is typically severe but does not have a crescendo pattem, and often occurs in younger patients Coronary artery vasculitis is important in the pathogenesis of Kawasald disease. ‘Marie this question m= (Question Id : 95472 Question 20 of 30 A.40) year old man presents to emergency department because of several hours of progressively increasing chest pain, History of hhypercholesterolemia, angixal episodes, and diabetes is present. Although previous episodes were relieved by sublngval nitroglycerin, the man states that it has nct helped with the pain durng this episode. EKG does not show any ST or T-wave changes, and CK-MB and troponns are within normal limits, Which of the following caused this condition? a) Atherosclerosis alone b) Coronary artery embolism c) Coronary artery spasm V © 4) Thrombosis with or without underlying atherosclerosis ®) Vasculitis Question Explanation: This patient presents with unstable angina, which is defined as recurrent episodes of angina on minimal effort or at rest that is no longer relieved by nitroglycerin, It can be referred to as a crescendo angina, and may or may not be associated with ST or T wave depression Cardiac enzymes (CK-MB.As and troponins) do not become clevated in patient's with this type of angna. Unstable angina is thought to be duc to a slowly developing, thrombosis in 2 coronary ariery branch. The thrombosis may or may not occur over an area of the vessel involved by atherosclerotic plaque, Atherosclerosis alone usually causes stable angina occurring with, exenion. Coronary artery embolism is uncommon but can occur ita plaque at the aortic otifice fragments and is driven into @ coronary artery. It would present with signs and symptoms of an acute myocardial infarction, with associated ST segment changes and T-wave changes. Coronary artery spasm is thought to cause angina at rest (Prinzmetal angina). This type of angina is typically severe but does not have a crescendo pattem, and often occurs in younger patients Coronary artery vasculitis is important in the pathogenesis of Kawasald disease. ‘Mack this question = => Question Td : 96750 Question 21 of 30 ‘A-patient develops chest pain that is improved by sitting and worsened by reclining six week after a myocardial infarction. He also develops a temperature of 103°F and an elevated white tlocd cell count. Whats the most lkely diagnosis? a) Acute myocardial infarction b) Infectious endocarditis c) Pulmonary embolus. 4) Systemic Inpus erythematosus e) Dressler's syncrome, Answer | Bolanation | Other User's Explanation Report An Error Question Explanation: “About five percent of patients wil develop fever, leukocytosis, and pericarditis one to twelve weeks fellowing an acute myocardial infarction, This is an auroimmnne phenomenon and is known as Dressler’s syadrome, Treatment usually involves aspirin or nonsteroidals, and may even require corticosteroids. The other choices have symptoms in common, but the timing of the event following the MI makes Dressler’s syndrome the most likely. ‘Mack this question = => Question Td : 96750 Question 21 of 30 ‘A patient develops chest pain that is improved by siting and worsened by reclining six weeks after a myocardial infarction. He also develops a temperature of 103°F and an elevated white blood cell count. Whatis the most lkely dagnosis? a) Acute myocarcial infarction, b) Infectious endocarditis c) Pulmonary embolus. d) Systemic lupus erythematosus, SY © e)Dresdler's syndrome. Answer | Bolanation | Other User's Explanation Report An Error Question Explanation: “About five percent of patients wil develop fever, leukocytosis, and pericarditis one to twelve weeks fellowing an acute myocardial infarction, This is an auroimmnne phenomenon and is known as Dressler’s syadrome, Treatment usually involves aspirin or nonsteroidals, and may even require corticosteroids. The other choices have symptoms in common, but the timing of the event following the MI makes Dressler’s syndrome the most likely. 2/24/2014 1:12:17 PM ‘Mark this question e& => Question Id : 101442 Question 22 of 30 A70 year oldman with class IIT congestive heart failure (CHE) due to systolic dysfimction asks about the use of ibuprofen for his "pains and aches" Appropriate counseling regarding NSAID use and heart failure should include which one of the following? a) NSAIDs are a good choice for pain relief as they decrease systemic vascular resistance b) NSAIDs are a good choice for pain reliefthey angment the effect of his diuretic c) NSAIDs, including high dose aspirin, should be avoided in CHF patients because they can cause fluid retention 4) High-dose aspirin (325 mg/day) is preferable to other NSAIDs for patients taking ACE inhibitors Question Explanation: Ifpossible, NSAIDs should be avoided in patients with heart failure, They cause sodiurn and water retention as well as an increase in systemic vascular resistance which may lead to cardiac decompensation, Patients with heart failure who take NSAIDs have a tenfold increased risk of hospitalization for exacerbation of their CHF. NSAIDs alone in patients with normal ventricular finction have not been associated with intial episodes of heart failure NSAIDs, including high-dose aspirin (325 mg/day) may decrease of negate entirely the beneficial unloading effects of ACE inhibition. “They have been shown to have a negative impact on the long-term morbidity and mortality benefits that ACE inhibitors provide ‘Sulindac and low dose aspirin (81 mg/day) are less likely to cause these negative effects. 2/24/2014 1:12:17 PM ‘Mark this question e& => Question Id : 101442 Question 22 of 30 A70 year oldman with class IIT congestive heart failure (CHE) due to systolic dysfimction asks about the use of ibuprofen for his "pains and aches" Appropriate counseling regarding NSAID use and heart failure should include which one of the following? a) NSAIDs are a good choice for pain relief as they decrease systemic vascular resistance b) NSAIDs are a good choice for pain reliefthey angment the effect of his diuretic Y © 0) NSAIDs, including high dose aspirin, should be avoided in CHE patients because they can canse fluid retention 4) High-dose aspirin (325 mg/day) is preferable to other NSAIDs for patients taking ACE inhibitors Question Explanation: Ifpossible, NSAIDs should be avoided in patients with heart failure, They cause sodiurn and water retention as well as an increase in systemic vascular resistance which may lead to cardiac decompensation, Patients with heart failure who take NSAIDs have a tenfold increased risk of hospitalization for exacerbation of their CHF. NSAIDs alone in patients with normal ventricular finction have not been associated with intial episodes of heart failure NSAIDs, including high-dose aspirin (325 mg/day) may decrease of negate entirely the beneficial unloading effects of ACE inhibition. “They have been shown to have a negative impact on the long-term morbidity and mortality benefits that ACE inhibitors provide ‘Sulindac and low dose aspirin (81 mg/day) are less likely to cause these negative effects. 22472014 1:12:34 PM. ‘Mavk this question & => Question Td: 113479 Question 23 of 30 A.A4 year old man complains of chest pain. The pain is reproducible on palpation and is alleviated with sitting forwerd in a chair Paticnt also has low grade fever and his ESR is 100 mmfhr. ECG reveals diffe ST-T segment elevations in all leads. The mostllikely diagnosis is a) Unstable angina b) Viral pericardiis ©) Myocardial infarction 4) Pericardial tamponade ©) Costochondrits, Question Explanation: This patient has symptoms of viral pericarditis. His pain is aleviated with siting forward and itis not radiating He may also have a pericardial effusion associated with his infection A pericardial effusion can be caused by metastatic cancer, lupus, or tuberculosis, A viral pericarditis usually resolves on its own, Unstable angina would present as crushing chest pain at rest and a fever, and pain upon palpation would be unnsual, as well as reproducible pain Ina myocardial infarction, the pain would not be reproducible and one would see ST-T wave elevations in certain leads only. Pericarditis can develop into tamponade ifthe effusion accunmuletes very rapidly In that case, the patient would be hemodynamically unstable and the EKG would show low voltage across the precordium, A. pulsus paradoxus would be ellicited. Costochondrtis presents as pleuritis and is rb inflammation alleviated with Tylenol or nonsteroidal anti-inlammatory drugs. S-T elevation would not be present. 22472014 1:12:34 PM. ‘Mavk this question & => Question Td: 113479 Question 23 of 30 A.44 year old man complains of chest pain. The pain is reproducible on palpation and is alleviated with sitting forward in a chair. Patient also has low grade fever and his ESR is 100 mmyhr. ECG reveals diffuse ST-T segment clevaticns in all leads. The most likely agnosis is a) Unstable angina Y © b) Viral pericarditis ©) Myocardial infarction 4) Pericardial tamponade €) Costochondritis Question Explanation: This patient has symptoms of viral pericarditis. His pain is aleviated with siting forward and itis not radiating He may also have a pericardial effusion associated with his infection A pericardial effusion can be caused by metastatic cancer, lupus, or tuberculosis, A viral pericarditis usually resolves on its own, Unstable angina would present as crushing chest pain at rest and a fever, and pain upon palpation would be unnsual, as well as reproducible pain Ina myocardial infarction, the pain would not be reproducible and one would see ST-T wave elevations in certain leads only. Pericarditis can develop into tamponade ifthe effusion accunmuletes very rapidly In that case, the patient would be hemodynamically unstable and the EKG would show low voltage across the precordium, A. pulsus paradoxus would be ellicited. Costochondrtis presents as pleuritis and is rb inflammation alleviated with Tylenol or nonsteroidal anti-inlammatory drugs. S-T elevation would not be present. 2242014 PM “Mark this question & => Question 24 of 30 An EKG shows a progressive increase in the PR interval until a QRS complex is dropped. In this situation what ECG diagnosis applies? a) First-degree AV block ») Second degree AV block, Motitz Type I ©) Second-dearee AW block, Mobitz Type IT. 4) Third-degree AV block «) Fourth-degree AV block Question Explanation: ‘Asecond degree AV block, Mobitz'Type I, ECG dagnoss would apply in this situation 2242014 PM “Mark this question & => Question 24 of 30 An EKG shows a progressive increase in the PR interval until a QRS complex is dropped. In this situation what ECG diagnosis applies? a) First-degree AV block Y © ») Second degree AV block, Motitz Type I ©) Second-dearee AW block, Mobitz Type IT. 4) Third-degree AV block «) Fourth-degree AV block Question Explanation: ‘Asecond degree AV block, Mobitz'Type I, ECG dagnoss would apply in this situation 2/24/2014 1:13:10 PM ‘Mark this question & => Question Id : 141366 Question 25 of 30 A woman aged 37 years is brought to the ER with a fever of 40°C (104°F). CAR shows muttiple patchy infitrates in both lungs, and examination reveals conjunctival petschiae and murmur along the lef: fourth intercosials space. Echocardiography suggests right sided, valvular damage. The most likely cause is a) Congenital heatt disease ) Ilicit drug use c) Rhenmatis fever 4) Rhoumatoid arthritis €) Systemic lupus erythematosus Question Explanation: ‘This patient has acute infective endocarditis. The most common cause of acute bacterial endocarditis is Stapkylococcus aureus, typically acquired by ict intravenous drug use, which can introduce skin organisms into the venous system that then aftack the tricuspid valve. Staphylococcus aureus accounts for between 60% and 90% of cases of endocarditis in intravenous drug users. The endocardits associated with congenital heart disease typically involves either damaged valves or atrial or ventricular septal defects “The tricuspid valve is not particularly vulnerable, Rheumatic fever most commonly damages the mitral and aortic valves, and tricuspid damage is usually less severe and seen only when the mitral and aortic valves are heavily involved, Rheumatoid arthritis is not associated with bacterial endocardits. Systemic lupus erythematosus can produce small, eseptic vegetations on valves, but is not associated with bacterial endocardits. It is important to recognize that the above patient has acute bacterial endocarditis (ABE) due to the use of intravenous drugs. The most common cause of ABE is Staphylococcus aureus. This can cause right sided valvular damage, mainly involving the tricuspid valve. 2/24/2014 1:13:10 PM ‘Mark this question & => Question Id : 141366 Question 25 of 30 A-woman aged 37 years is brought to the ER with a fever of 40°C (104°F). CXR shows multiple patchy infiltrates in both lungs, and examination reveals conjunctival petechiac and murmur along the lef fourth intercostals space. Echocardiography suggests right sided valvular damage. The most likely cause is a) Congenital heart disease oY © b) Dlcit drug use c) Rheumatic fever d) Rheumatoid arthritis €) Systemic lupus erythematosus Question Explanation: ‘This patient has acute infective endocarditis. The most common cause of acute bacterial endocarditis is Stapkylococcus aureus, typically acquired by ict intravenous drug use, which can introduce skin organisms into the venous system that then aftack the tricuspid valve. Staphylococcus aureus accounts for between 60% and 90% of cases of endocarditis in intravenous drug users. The endocardits associated with congenital heart disease typically involves either damaged valves or atrial or ventricular septal defects “The tricuspid valve is not particularly vulnerable, Rheumatic fever most commonly damages the mitral and aortic valves, and tricuspid damage is usually less severe and seen only when the mitral and aortic valves are heavily involved, Rheumatoid arthritis is not associated with bacterial endocardits. Systemic lupus erythematosus can produce small, eseptic vegetations on valves, but is not associated with bacterial endocardits. It is important to recognize that the above patient has acute bacterial endocarditis (ABE) due to the use of intravenous drugs. The most common cause of ABE is Staphylococcus aureus. This can cause right sided valvular damage, mainly involving the tricuspid valve. ‘Mark this question = => Question Td : 158610 Question 26 of 30 Following cardiac surgery a 68 year old man’s ECG shows absent P waves and tall, peaked T waves. Cardiac examination is unremarkable and vital signs are stable, though he is slightly bradycardic. What is contraindicated in him? a) Acetazolamide ) Furosemide c) Hydrochlorothiaride 4) Lisinopsil ©) Spironolactone Question Explanation: ‘An absence of P waves and tall, pecked T waves are pathognomonic for elevated potassium Patients with hyperkalemia are at risk for arrhythmia, and should not be treated with potassium sparing diuretics. Potassiumn sparing diuretics, such as spironelactone, amiloride, and tiamverene, act at the level of the collecting rubules and ducts, by countering the effects of aldosterone. Use of the medication will lead to an increase in potassium and a decrease in sodium. Acetezolamide is a carbonic anhydrase inhibitor that works at the level of the proximal tubule. It is not a potassiumn-sparing drug. Furosemide is a loop duretic that acts at the ascending loop of Henle and blocks the cotransport of sodium, potassium, and chloride, resulting in a net loss of potassium Hydrochlorothiazide acts at the distal tubule and inhibits the sodium/chloride cotransport system. Ttis also not potassium sparing ‘Lisinopnil an ACE inhibitor, acts by preventing the conversion of angiotensin Ito angiotensin Il, thereby reducing vasoconstriction and, aldosterone secretion, ‘Mark this question = => Question Td : 158610 Question 26 of 30 Following cardiac surgery a 68 year old man’s ECG shows absent P waves and tall, peaked T waves. Cardiac examination is unremarkable and vital signs are stable, though he is slightly bradycardic. What is contraindicated in him? a) Acetazolamide ) Furosemide c) Hydrochlorothiaride 4) Lisinopsil ¥ © €) Spironolactone Question Explanation: ‘An absence of P waves and tall, pecked T waves are pathognomonic for elevated potassium Patients with hyperkalemia are at risk for arrhythmia, and should not be treated with potassium sparing diuretics. Potassiumn sparing diuretics, such as spironelactone, amiloride, and tiamverene, act at the level of the collecting rubules and ducts, by countering the effects of aldosterone. Use of the medication will lead to an increase in potassium and a decrease in sodium. Acetezolamide is a carbonic anhydrase inhibitor that works at the level of the proximal tubule. It is not a potassiumn-sparing drug. Furosemide is a loop duretic that acts at the ascending loop of Henle and blocks the cotransport of sodium, potassium, and chloride, resulting in a net loss of potassium Hydrochlorothiazide acts at the distal tubule and inhibits the sodium/chloride cotransport system. Ttis also not potassium sparing ‘Lisinopnil an ACE inhibitor, acts by preventing the conversion of angiotensin Ito angiotensin Il, thereby reducing vasoconstriction and, aldosterone secretion, ‘Mark this question = => (Question Id: 178742 Question 27 of 30 A 56 year old man has crushing chest and left shoulder pain of 2 hours duration, not relieved by sublingual nitroglycerin, ECG shows ST elevation in several leacls. Aspirin and streptokinase therapy are initiated, Nest day, serum cardiac enzymes are elevated to 4 times the upper Lmit af normal. What is the likely diagnasis? a) Prinanetal’s angina ) Stable angina ©) Subendocardial infarction 4) Transmucal infarction e) Unstable angina Question Explanation ‘The elevated serum cardiac enzymes (CK-MB, tropotins) indicate that a myocardial infarction has occurred The setting (patient ‘brought in from community with typical myocardial pain) and limitation of ST elevation to a few leads are typical of transemural infarction due to occlusion ofa coronary artery. In contrast, hospitalized, severely hypotensive patients typically undergo the more generalized subendocardial infarction, Prinzmstal's angna would not cause a marked nse in serum enzymes. Stable angina would not cause a marked rise in serum enzymes. Subendocardial infarction usually occurs in the seting of shock and affects most EKG leads. ‘Unstable angina may be accompanied by small enzyme elevations up to 2 times the upper limit of normal ‘Mark this question = => (Question Id: 178742 Question 27 of 30 A 56 year old man has crushing chest and left shoulder pain of 2 hours duration, not relieved by sublingual nitroglycerin, ECG shows ST elevation in several leacls. Aspirin and streptokinase therapy are initiated, Nest day, serum cardiac enzymes are elevated to 4 times the upper Lmit af normal. What is the likely diagnasis? a) Prinanetal’s angina ) Stable angina ©) Subendocardial infarction Y © @) Transmural infarction e) Unstable angina Question Explanation ‘The elevated serum cardiac enzymes (CK-MB, tropotins) indicate that a myocardial infarction has occurred The setting (patient ‘brought in from community with typical myocardial pain) and limitation of ST elevation to a few leads are typical of transemural infarction due to occlusion ofa coronary artery. In contrast, hospitalized, severely hypotensive patients typically undergo the more generalized subendocardial infarction, Prinzmstal's angna would not cause a marked nse in serum enzymes. Stable angina would not cause a marked rise in serum enzymes. Subendocardial infarction usually occurs in the seting of shock and affects most EKG leads. ‘Unstable angina may be accompanied by small enzyme elevations up to 2 times the upper limit of normal 2/24/2014 1:13:59 PM. ‘Mark this question ez Question Id: 178807 Question 28 of 30 AAA year old female has a blood pressure of 200/140 mmETg during a routine cxam. Further examination reveals retinal hemorrhages and the ECG shows left axis deviation, Whatis likely to be decreased in her? a) Arteriolar density ') Atteriotar wall thickness ©) Arteriotar wall to lumen ratio 4) Capillary wall t> hamen ratio €) Total peripheral resistance Question Explanation: ‘This woman tras malignant hypertension, The hypertension has caused left ventricular hypertropky, indicated by the left axis deviation seen on the ECG, Another phenomenon that can occur in malignant hypertension is a loss of artericles, leading t> decreased attesioiar density. This loss of arterioles, termed arterioiar rarefaction, is poorly understood, butis believed to result from long term over perfusion of the tissues. Organs and tissues in which the vasculature has primarily a nutritive function (e.g., brain, heat, skeletal smuscle) regulate their blood flow in accordance with the metabolic needs of the tissues. These tissues exhibit short-term autoregulation of blood flow such that the increase in blood flow caused by elevated artenal pressure is minimized by atterioiar constriction, When the increased blood pressure persists for weeks to months, many of the constricted arterioles close off and are resorbed. Therefore, the arterioiar rarefaction that occurs in hypertensive ndiwiduals is an example of long term autoregulation “Arteriolar wall thickness increases in hypertension as an adaptation to the high pressure. ‘The artericlar well to lumea ratio increases in hypertensive individuals, Capilaries lack smooth muscle cells in their walls and the wall-to-lumen ratio of capillaries does aot change Total peripheral sesistance is increased in hypertensive individuals. 2/24/2014 1:13:59 PM. ‘Mark this question ez Question Id: 178807 Question 28 of 30 A.44 year old female has a blood pressure of 200/140 mmHg during a routine cxara. Further examination reveals retinal hemorshages and the ECG shows lef axis deviation. Whatis likely to be decreased in her? Y © a) Atteriolar density ') Arteriolar wall thickness ©) Arteriotar wall to lumen ratio 4) Copillary wall to hirnen ratio ©) Total peripheral resistance Question Explanation: ‘This woman tras malignant hypertension, The hypertension has caused left ventricular hypertropky, indicated by the left axis deviation seen on the ECG, Another phenomenon that can occur in malignant hypertension is a loss of artericles, leading t> decreased attesioiar density. This loss of arterioles, termed arterioiar rarefaction, is poorly understood, butis believed to result from long term over perfusion of the tissues. Organs and tissues in which the vasculature has primarily a nutritive function (e.g., brain, heat, skeletal smuscle) regulate their blood flow in accordance with the metabolic needs of the tissues. These tissues exhibit short-term autoregulation of blood flow such that the increase in blood flow caused by elevated artenal pressure is minimized by atterioiar constriction, When the increased blood pressure persists for weeks to months, many of the constricted arterioles close off and are resorbed. Therefore, the arterioiar rarefaction that occurs in hypertensive ndiwiduals is an example of long term autoregulation “Arteriolar wall thickness increases in hypertension as an adaptation to the high pressure. ‘The artericlar well to lumea ratio increases in hypertensive individuals, Capilaries lack smooth muscle cells in their walls and the wall-to-lumen ratio of capillaries does aot change Total peripheral sesistance is increased in hypertensive individuals. 22472014 1:14:15 PM ak tas question <> Question 29 of 30 A.44 year old male has chest pain and pain in his left shoulder. He kad spent the previous 2 hours shoveling snow off his driveway but had to stop because of the pain. He has had several similar episodes. ECG shows ST segment depression. Sublngual aitroglycerin relieves his pain, The following morning, serumn cardiac enzymes are within normal limits and no BCG changes are seen, What is the most likely diagnosis? a) Prinzmetal variant angina 'b) Stable (typical) angina ©) Suberdocardial infarction 4) Transmusal infarction ¢) Tastaale (crescendo) angina Question Explanation: “Myocardial infarction is excluded by the failure of serum cardiac enzymes to rise. The patient has angina rather than infarction. This is not his first episode, and it was triggered by heavy physical labor. This suggests that he has stable (typical) angina since Prinanetal angina is triggered at rest and unstable anginais characterized by progressively smaller triggers for angina pain, New onset angina is, by definition, unstable and should be monizored closely. Prinzmetal variant angina is ruled out because it oxcurs at rest. Subendocardlal infarction and transmural infarction can be ruled out because both would produce enzyme elevations. Unstable angina 4s a severe form that occurs with progressively less severe triggers. It may or may not produce smal elevations of cardiac enzymes up to twice the upper limit ofnormal possibly because of death of afew myocardial cells 22472014 1:14:15 PM ak tas question <> Question 29 of 30 A.44 year old male has chest pain and pain in his left shoulder. He kad spent the previous 2 hours shoveling snow off his driveway but had to stop because of the pain. He has had several similar episodes. ECG shows ST segment depression. Sublngual aitroglycerin relieves his pain, The following morning, serumn cardiac enzymes are within normal limits and no BCG changes are seen, What is the most likely diagnosis? a) Prinzmetal variant angina Y © b) Stable (typical) angina ©) Suberdocardial infarction 4) Transmusal infarction ¢) Tastaale (crescendo) angina Question Explanation: “Myocardial infarction is excluded by the failure of serum cardiac enzymes to rise. The patient has angina rather than infarction. This is not his first episode, and it was triggered by heavy physical labor. This suggests that he has stable (typical) angina since Prinanetal angina is triggered at rest and unstable anginais characterized by progressively smaller triggers for angina pain, New onset angina is, by definition, unstable and should be monizored closely. Prinzmetal variant angina is ruled out because it oxcurs at rest. Subendocardlal infarction and transmural infarction can be ruled out because both would produce enzyme elevations. Unstable angina 4s a severe form that occurs with progressively less severe triggers. It may or may not produce smal elevations of cardiac enzymes up to twice the upper limit ofnormal possibly because of death of afew myocardial cells 2/24/2014 1:14:32 PM ‘Mark this question e& Question Id: 181811 Question 30 of 30 A.61 year old malnourished alcoholic presents with shortness of breath and gasping for air on awakening. Cardiac exam reveals an S3 heart sound, a diastolic murmur, and jugular venous dstention, Pulmonary rales and peripheral edema are evident, What would be expected to be seen on an echocardiogram? a) A carotid pulse tracing with spike and dome configuration +) Bilateral atrial enlargement and ventricular thickening ©) Depressed left ventricular fanction with pericardial effusion 4) Left and right ventricular dilatation with poor contraction throughout ©) Left ventricular hypertrophy with asymmetric septal hypertrophy Answer | Bxplanation Other User's Explanation Report An Error Question Explanation: Dilated cardiomyopathy can be caused by ethanol abuse Malnourishment offen accompanies severe alccholism and implies thicmine deficiency, which can lead to heart disease (wet beriberi). Suspectthis diagnosis in any alcoholic presenting with symptoms and signs ‘of congestive heart failure, Ta this patient, an echocardiogram would bs expected to reveal bilateral veatriculer clatation with impaired contraction throughout both chambers. A carotid pulse tracing with spilce and dome configuration and lef ventricular kypertrophy with asyininetric septal hypertrophy are both associated with hypertrophic cardiomyopathy, also known as idiopathic hypertrophic subaottic stenosis, Bilateral atrial enlargement and vertricular thickening is associated with restrictive cardiomyopathy. Common, causes of this condition are amyloidosis (in the elderly) and sarcoidosis (in the young). Depressed left ventricular function with pericardial effusion would be consistent with myocarditis accompanied by pericarditis. 2/24/2014 1:14:32 PM ‘Mark this question e& Question Id: 181811 Question 30 of 30 A.61 year old malnourished alcoholic presents with shortness of breath and gasping for air on awakening. Cardiac exam reveals an S3 heart sound, a diastolic murmur, and jugular venous dstention, Pulmonary rales and peripheral edema are evident, What would be expected to be seen on an echocardiogram? a) A carotid pulse tracing with spike and dome configuration +) Bilateral atrial enlargement and ventricular thickening ©) Depressed left ventricular fanction with pericardial effusion Y © 4) Left and sight ventricular dilatation with poor contraction throughout ©) Left ventricular hypertrophy with asymmetric septal hypertrophy Answer | Bxplanation Other User's Explanation Report An Error Question Explanation: Dilated cardiomyopathy can be caused by ethanol abuse Malnourishment offen accompanies severe alccholism and implies thicmine deficiency, which can lead to heart disease (wet beriberi). Suspectthis diagnosis in any alcoholic presenting with symptoms and signs ‘of congestive heart failure, Ta this patient, an echocardiogram would bs expected to reveal bilateral veatriculer clatation with impaired contraction throughout both chambers. A carotid pulse tracing with spilce and dome configuration and lef ventricular kypertrophy with asyininetric septal hypertrophy are both associated with hypertrophic cardiomyopathy, also known as idiopathic hypertrophic subaottic stenosis, Bilateral atrial enlargement and vertricular thickening is associated with restrictive cardiomyopathy. Common, causes of this condition are amyloidosis (in the elderly) and sarcoidosis (in the young). Depressed left ventricular function with pericardial effusion would be consistent with myocarditis accompanied by pericarditis. 22472014 1:15:35 PM ‘Mark this question => ‘Question Id : 21729 Question 1 of 30 4.42 year old male with Marfan syndrome, aortis insufficiency and mitral regurgtation comes to the emergency department because of severe substernal chest pain for the past 3 hours. He describes the pain radiating to the neck and tearing in quality. He experienced similar, but less severe, chest pain one week earlier and treated himcelf with sepirin, The underlying cause of his worsening symptoms can be? a) Acute bacterial endocarditis, b) Acute myocardial infarction c) Esophageal reflux with spasin 4) Dissection of the aorta €) Perforated peptic ulcer Question Explanation: ‘The factor that immediately points to dissection of the aorta is that the patient has Marfan syndrome. Cardiovascular protlems are the ‘most common causes of morbidity and mortaity in such patients. Mitral valve prolapse often develops early, and are seen in more than 80% of adult Marfan syndrome patients, However, of most concemis disease of the ascending aorta, of which dlation of the aortic rootis most serious, since a possibly fatal subsequent dissection and rupture can occur. One must always think acute aortic dissection for a Marfan patient with sudden onset of severe chest pain. The “tearing” quality of the pain also tends to point toward aortic dissection. The pain results from stimulation of nerve endings in the adventitia, and it begins abruptly, rapidly becoming severe 22472014 1:15:35 PM ‘Mark this question => ‘Question Id : 21729 Question 1 of 30 A.42 year old mele with Marfan syndrcene, aortis insuficiency and mitral seguegtation comes to the emergency department because of cevere substeraal chest pain for the past 3 hows. He describes the pain radiating to the neck and tearing in quality. He experienced similar, butlece cevere, chest pain one weele ealer and treated himself with aepiin. The underlying cause of hic worsening eymptoras can be? a) Acute bacterial endocarditis 'b) Acute myocardial infarction c) Esophageal reflux with spasin SY © d) Dissection of the aorta ©) Perforated peptic ulcer Question Explanation: ‘The factor that immediately points to dissection of the aorta is that the patient has Marfan syndrome. Cardiovascular protlems are the ‘most common causes of morbidity and mortaity in such patients. Mitral valve prolapse often develops early, and are seen in more than 80% of adult Marfan syndrome patients, However, of most concemis disease of the ascending aorta, of which dlation of the aortic rootis most serious, since a possibly fatal subsequent dissection and rupture can occur. One must always think acute aortic dissection for a Marfan patient with sudden onset of severe chest pain. The “tearing” quality of the pain also tends to point toward aortic dissection. The pain results from stimulation of nerve endings in the adventitia, and it begins abruptly, rapidly becoming severe ‘Mare this question <= => (Question Td : 22324 Question? of 30 A.50 year old muse comes to your office complaining of severe bilateral buttock cramps and thigh fatigue during a tennis match and recent onset of impotence. The most likely diagnosis is a) Lumbosacral dise problem 'b) Multiple sclerosis c) Metastatic carcinoma of the spine 4) Lumbosacral stenosis ©) Letiche syndrome Question Explanation: Leriche’s syndrome is an atherosclerotic occlusive disease invelving the abdominal acrta and/or both of the iliac arteries. Clessically, itis described in male patients as a triad of symptoms consisting of absent or diminished femoral pulses, intermittent claudication (pain with walking) and penile impotence. This combination is known as Leriche syndrome, However, ary number of symptoms may present, depending on the distribution and severity ofthe disease. Variable, chronic ischemia involving the lower limbs is a common ‘presentation. ‘Mare this question <= => (Question Td : 22324 Question? of 30 A.50 year old muse comes to your office complaining of severe bilateral buttock cramps and thigh fatigue during a tennis match and recent onset of impotence. The most likely diagnosis is a) Lumbosacral dise problem 'b) Multiple sclerosis c) Metastatic carcinoma of the spine 4) Lumbosacral stenosis SY © )Leriche syndrome Question Explanation: Leriche’s syndrome is an atherosclerotic occlusive disease invelving the abdominal acrta and/or both of the iliac arteries. Clessically, itis described in male patients as a triad of symptoms consisting of absent or diminished femoral pulses, intermittent claudication (pain with walking) and penile impotence. This combination is known as Leriche syndrome, However, ary number of symptoms may present, depending on the distribution and severity ofthe disease. Variable, chronic ischemia involving the lower limbs is a common ‘presentation. 2/24/2014 1:16:04 PM. ‘Mark this question & => Question Td : 30018 Question 3 of 30 A.66 year old woman has smoked 50 cigarettes a day for 40 years. She has hed increasing dyspnea for several years and CXR findings show emphysema, She develops worsening peropheral edema over next few years. Vitals are: T 367°C, pulse 80/min, RR 1S/inin and BP 120/80 mm He Which cardiac finding is likely to be present? a) Constnetive pericartitis b) Right ventricular hypertrophy ©) Left ventricular (LV) aneurysm. ) Mitral valve stenosis ¢) Non-bacterial thrombotic endocarditis Avewor (UEQRISNAN) other Ucor's Explanation Report An Evror Question Explanation: ‘The most likely finding in this woman is pulmonary hypertension as a result of emphysema secondary to long term cigarette smoking Peripheral oedema is due to right heart dilatation and failure. Mitral stenosis is not supported by the history. Constntive pericarditis could be caused by a hung malignancy in this patient, but again there is no suggestion of this inthe history. Constructive pericarditis would be characterized by soft heart sounds, a diastolic ‘pericardial knock" and gross signs of right heart failure. LV aneurysm would lead to symptoms and signs of left heart failure and again is not the most licely finding suggested by the history. 2/24/2014 1:16:04 PM. ‘Mark this question & => Question Td : 30018 Question 3 of 30 4.66 ycar old woman has smoked 50 cigarettes a day for 40 ycars. She has had increasing dyspnea for several years and CXR findings show emphysema. She develops worsening peropheral edema over next few years. Vitals are: T36.7°C, pulse 80/min, RE. 15fmin and BP 120/80 mm Hyg Which cardiac finding is likely to be present? a) Consinctive pericarditis Y © b) Right ventricular hypertrophy c) Left ventricular (LV) aneurysm: d) Mitral valve stenosis &) Non-bacterial thrombotic endocarditis Avewor (UEQRISNAN) other Ucor's Explanation Report An Evror Question Explanation: ‘The most likely finding in this woman is pulmonary hypertension as a result of emphysema secondary to long term cigarette smoking Peripheral oedema is due to right heart dilatation and failure. Mitral stenosis is not supported by the history. Constntive pericarditis could be caused by a hung malignancy in this patient, but again there is no suggestion of this inthe history. Constructive pericarditis would be characterized by soft heart sounds, a diastolic ‘pericardial knock" and gross signs of right heart failure. LV aneurysm would lead to symptoms and signs of left heart failure and again is not the most licely finding suggested by the history. 2/24/2014 1:16:19 PM. ‘Marc this question & => Question Td : 49824 Question 4 of 30 ‘While performing cardiac resuscitation on an adult by two persons, the ratio of cardiac compressions to respirations recommended is which of the following? a) 51 b) 3:2 ©) 15:2 S101 (Question Explanation: “While previous versions of the adult Basic Life Suoport guidelines recommend a ratio of 15 compressions to 2 ventilations for one- rescuer CPR anda ratio of 5 compressions to 1 ventlation for two-rescuer CPR, current evidence suggests thet coronary perfusion pressure is higher after 15 uninterrupted chest compressions than itis after 5 chest compressions. ‘Thus the 15:2 ratio is now recommended for one or two rescuers, and applies to adult Basic Life Support provided by both laypersons and health care professionals 2/24/2014 1:16:19 PM. ‘Marc this question & => Question Td : 49824 Question 4 of 30 “While performing cardiac resuscitation on an adult by two persons, the ratio of cardiac compressions to respirations recommended is which ofthe following? a) 5 b) 3:2 Y Oo) 152 4) 101 (Question Explanation: “While previous versions of the adult Basic Life Suoport guidelines recommend a ratio of 15 compressions to 2 ventilations for one- rescuer CPR anda ratio of 5 compressions to 1 ventlation for two-rescuer CPR, current evidence suggests thet coronary perfusion pressure is higher after 15 uninterrupted chest compressions than itis after 5 chest compressions. ‘Thus the 15:2 ratio is now recommended for one or two rescuers, and applies to adult Basic Life Support provided by both laypersons and health care professionals “Mark this question €&c> Question Id: 49834 Question 5 of 30 AB year old woman comes to you with complains of painful, cold finger tips which tam white when she hanging out her laundry “Vii there ie no approved treatment for this condltion at this time, the drug that hac been shown to be usefulie a) Propranolol (inderal) ) Nifedipine Procardia) ©) Ergotamincloalfeine (Caforgot) ) Methysergide (Sansert) Question Explanation: At present there is no approved treatment for Raynaud's disease. However. patients with this disorder reportedly experience subjective symptomatic improvement with calcinm channel antagonists. Nifedipine is the calcium channel blocker of choice in patients with Raynaud's disease. Beta blockers can produce arterial insufficiency of the Raynaud type, so propranolol and atenolol would be contraindicated, Drugs such as ergotamine preparations and methysergide can produce cold sensitivity, and should therefore be avoided in patients with Raynaud's disease. “Mark this question €&c> Question Id: 49834 Question 5 of 30 A.48 year old woman comes to you with complains of painfil, cold finger tips which turn white when she hanging out her laundry. “While there is no approved treatment for this condition at this time, the drug that has been shown to be usefillis a) Propranolol (Inderal) Y¥ © b) Nifedipine Procardia) c) Ergotamine/caffeine (Cafergot) d) Methysergide (Sansert) Question Explanation: At present there is no approved treatment for Raynaud's disease. However. patients with this disorder reportedly experience subjective symptomatic improvement with calcinm channel antagonists. Nifedipine is the calcium channel blocker of choice in patients with Raynaud's disease. Beta blockers can produce arterial insufficiency of the Raynaud type, so propranolol and atenolol would be contraindicated, Drugs such as ergotamine preparations and methysergide can produce cold sensitivity, and should therefore be avoided in patients with Raynaud's disease. ‘Mark this question & => Question Td : Question 6 of 30 A.66-year-old women for the frst time had a 20 minute episode of difficulty speaking and weakness of fight side face and right arm. Her pulse is 90/tnin irregularly regular. ECG shows atrial Sibrillation with left ventricular hypertrophy but no Q waves or ST elevation, CT brain without contrast is negative Appropriate immediate step is a) Lowering blood pressure b) Antiplatelet therapy with clopidogrel (Plavix) ©) Anticoagulation with warfarin (Coumadin) 4) Blectrical or chemical cardioversion €) An MEI scan ofthe brain with contrast Ancwor [UEIABWBIAN) other Users Explanation Report An Evan Question Explanation: ‘The patient described presents with a history most consistent with a recent, resolved transient ischemic attack (ILA). This was most likely due to an embolus related to her airial fibrillation. Her risk for a recurrent neurologic event (ILA or stroke) is high. Long-term anticoagulation with warfarin reduces this risk. The use of antiplatelet agents such as clopidogrel to reduce TLAs has not been studied. Lowering blood pressure and lipid levels can reduce sisks over the long term, but do not require immediate intervention. Cardioversion for patients with atrial fibrillation of uncertain or long duration may be appropriate but should aot be attempted before several weeks of anticoagulation in the stable patient, ‘Mark this question & => Question Td : Question 6 of 30 A 66-year-old woman for the first time had a 20 minute episode of dificulty speaking and weakness of fight side face and right arm. Her pulse is 90/min iregulasly regular. ECG shows atrial Sbrilation with left ventricular hypertrophy but no Q waves or ST elevation, CT brain without coutrast is negative. Appropriate immediate step is a) Lowering blood pressure +b) Antiplatelet therapy with clopidogrel Plavix) Y © ©) Antcoagulation with warfarin (Coumadin) 4) Blectrical or chemical cardioversion e) An MRI scan of the brain with contrast Ancwor [UEIABWBIAN) other Users Explanation Report An Evan Question Explanation: ‘The patient described presents with a history most consistent with a recent, resolved transient ischemic attack (ILA). This was most likely due to an embolus related to her airial fibrillation. Her risk for a recurrent neurologic event (ILA or stroke) is high. Long-term anticoagulation with warfarin reduces this risk. The use of antiplatelet agents such as clopidogrel to reduce TLAs has not been studied. Lowering blood pressure and lipid levels can reduce sisks over the long term, but do not require immediate intervention. Cardioversion for patients with atrial fibrillation of uncertain or long duration may be appropriate but should aot be attempted before several weeks of anticoagulation in the stable patient, 22472014 1:17:01 PM “Mak this question & => Question 7 of 30 An elderly man with a history of unresected cancer now has decreased level of consciousness. He is found te have large cerebral metastasis with some edema and midline shift on a magnetic resonance imaging MRI of the brain. The finding consistent with the dlagnosis would be a) Unlateral headache ) Increased heart rate ©) Increased respiratory rate 4d) Reduced blood pressure €) Reduced heart rate Anewor UERUNERY) oer tisrs Explanation Repost An Err Question Explanation: ‘Metastatic brain tumor commonly occurs in patients with primary cancers such as lung, breast, GI end melanoma, Patient can present vith severe headache and vomiting, Physical exam will show bradycardia, mid hypertension paplledema due to increased intracranial pressure. Diagnosis is by CT or MRI. 22472014 1:17:01 PM “Mak this question & => Question 7 of 30 An elderly man with a history of unresected cancer now has decreased level of consciousness. He is found te have large cerebral metastasis with some edema and midline shift on a magnetic resonance imaging MRI of the brain. The finding consistent with the dlagnosis would be a) Unlateral headache b) Increased heart rate ) Increased respiratory rate ) Reduced blood pressure v © ¢) Reduced heart rate Anewor EINER) oer tisers Explanation Repost An Err Question Explanation: ‘Metastatic brain tumor commonly occurs in patients with primary cancers such as lung, breast, GI end melanoma, Patient can present vith severe headache and vomiting, Physical exam will show bradycardia, mid hypertension paplledema due to increased intracranial pressure. Diagnosis is by CT or MRI. 2/24/2014 PM ‘Marie this question & => Question Id : 51185 Question 8 of 30 Gilostazol Pletal) has been found to be useful drug for the treatment of intermittent claudication This drug is contraindicated in patients with a) Congestive heart failure ) A past history of stroke ©) Diabetes mellitus 9) Thid degree heart block €) Hyperipidemia newer (UEINRER) tne sors xplanaton — RepertAnEror Question Explanation: Cilostazol is a drug with phosphodiesterase inhibitor activity introduced for the symptomatic treatment of arterial occlusive disease and intermittent claudication, Cilostazol should be avoided in patients with congestive heart failure, There are no limitations on its use in patients with previous stroke of a history of diabetes. It has been found to have beneficial effects on HDL cholesterol levels and in the treatment of third degree heart block. 2/24/2014 PM ‘Marie this question & => Question Id : 51185 Question 8 of 30 Cilostazol Pletal) has been found to be useful drug for the treatment of intermitient claudication This drug is contraindicated in patients with Y © a) Congestive heart failure 'b) A past history of stroke c) Diabetes mellitus d) Third degree heart block e) Hyperlipidemia newer (UEINRER) tne sors xplanaton — RepertAnEror Question Explanation: Cilostazol is a drug with phosphodiesterase inhibitor activity introduced for the symptomatic treatment of arterial occlusive disease and intermittent claudication, Cilostazol should be avoided in patients with congestive heart failure, There are no limitations on its use in patients with previous stroke of a history of diabetes. It has been found to have beneficial effects on HDL cholesterol levels and in the treatment of third degree heart block. 2/24/2014 1:17:26 PM ‘Mark this question & => Question Ta : 54541 Question 9 of 30 In which clinical stuation would be most appropnate to use a Beta-blocker that has intrinsic sympathomimetic actinty such as acebutolol or pindolol? a) As a cardioprotective agent post myocardial infarction bb) In a hypertensive patient with symptomatic Bradycarcia while talking Metoprolol (Lopressor) c) In a hypertensive patient with diabetes melliras 4) In 2 hypertensive patient with asthma @) To maintain sinus rhythm in @ patient with chronic atrial fibrillation Answer | Explanation Other User's Explanation Report An Error Question Explanation: Beta-Blockers with intrinsic sympathomimetic activity (S.A) are less beneficial in reducing mortality post myecardial infarction, and for this reason are not recommended for ischemic heart disease. They have a potential advantage in orly one clinical situation. Since they tend to lower heart rates less, they mey be beneficial in patients with symptomatic bradycardia while taking other Beta-blockers. ‘All Beta-blockers should be used cautiously in patients with diabetes os asthma, Orly sotalol, which delays ventsicular depolarization, hhas been shown to be effective for maintenance of sinus rhythm in patients with chronic attial fibrillation 2/24/2014 1:17:26 PM ‘Mark this question & => Question Ta : 54541 Question 9 of 30 In which clinical situction would be most appropnate to use a Beta-blocker that has intrinsic sympathomimetic activty such as acebutolol or pindolol? a) As a cardioprotective agent post myocardial infarction Y © b) Ina hypertensive patient with symptomatic Bradycardia while taking Metoprolol (Lopressor) c) Ih a hypemtensive patient with diabetes mellius 4d) In a hypertensive patient with asthma @) To maintain sinus rhythm in @ patient with chronic atrial fibrillation Answer | Explanation Other User's Explanation Report An Error Question Explanation: Beta-Blockers with intrinsic sympathomimetic activity (S.A) are less beneficial in reducing mortality post myecardial infarction, and for this reason are not recommended for ischemic heart disease. They have a potential advantage in orly one clinical situation. Since they tend to lower heart rates less, they mey be beneficial in patients with symptomatic bradycardia while taking other Beta-blockers. ‘All Beta-blockers should be used cautiously in patients with diabetes os asthma, Orly sotalol, which delays ventsicular depolarization, hhas been shown to be effective for maintenance of sinus rhythm in patients with chronic attial fibrillation 2/24/2014 1:17:40 PM. ‘Mark this question ez Question 10 of 30 An 81-year-old has palpitations for five hours. A month before, he had right arm weakness and speech problem that resclved within four hours, Being stable with a BP=110/82 smmiFig and pulse=137 bpm, ie is confirmed to have atrial Gbrillation. He was euthyroid and on no medicines. In an hour, he reverted to sinus rhythm spontaneously. A 24 hour ECG revedled three episodes of atrial fibsillaion each of ten minutes. What is the Management? a) Amiodarone ) Warfarin ©) Aspirin 4d) Metoprolol @) Digoxin Answer [MESSNER] omer Users xpanas Question Explanatio ‘The most appropnate intial therapy initial for this patient who has a high tisk of thrombo-embolic siroke is anticoagulation with warferin mainteining an intemational normalized ratio (INR) between 2-3. This should be the inital prionty as he has already had one episode of transient ischemia attack (ILA) ‘The maintenance of smus rhythm would be the next step and amiodarone or sotalol are options. n Report An Error 2/24/2014 1:17:40 PM. ‘Mark this question ez Question 10 of 30 An 81-year-old has palpitations for five hours. A month before, he had right arm weakness and speech problem that resolved within four hours. Being stable with a BP=110/82 mmHg and pulse=137 bpm, he is confirmed to have atrial fibrillaion He was euthyroid and on no medicines. In an hour, he reverted to sirus rhythm spontaneously. A 24 hour ECG revealed three episodes of atrial fibrillation each of ten minutes. What is the Management? a) Amiodarone ¥ © b) Warfarin ©) Aspirin d) Metoprolol e) Digoxin Answer [MESSNER] omer Users xpanas Question Explanatio ‘The most appropnate intial therapy initial for this patient who has a high tisk of thrombo-embolic siroke is anticoagulation with warferin mainteining an intemational normalized ratio (INR) between 2-3. This should be the inital prionty as he has already had one episode of transient ischemia attack (ILA) ‘The maintenance of smus rhythm would be the next step and amiodarone or sotalol are options. n Report An Error 2242014 1:17:55 PM. “Matt this question & => Question 11 of 30 A.45- year-old patient with enterococcal endocarditis should be treated firs with a) Gentamicin, 6) Mothicilin c) Penicilin 4) Ceftriaxone ¢) Trimethoprin- sulfamethoxazole Question Explanation: Gentamicin is the dmg of choice for the treatment of enterococcal endocarditis, mostly because of the higher incidence of streptomycin resistence (over gentamicin resistence). Bactericidal drugs such as gentamicin are the drugs of choice for the majority of bacterial endocarditis, 2242014 1:17:55 PM. “Matt this question & => Question 11 of 30 ‘A.45- year-old patient with enterococcal endocarditis should be treated frst with Y © a) Gentamicin, b) Methicillin c) Penicillin 4) Ceftriaxone. ¢) Trimethoprim sulfamethoxazole. Question Explanation: Gentamicin is the dmg of choice for the treatment of enterococcal endocarditis, mostly because of the higher incidence of streptomycin resistence (over gentamicin resistence). Bactericidal drugs such as gentamicin are the drugs of choice for the majority of bacterial endocarditis, ‘Mark this question ec ‘Question Id: 82086 Question 12 of 30 A. 67-year-old woman shows ischemic changes on her EKG in leads I, II, and avF. The coronary vessel most likely to be affected is a) Left reain b) Let circumflex c) Left anterior descending 4) Right coronary Answer | Bxpianation | Other User's Explanation Report An Error Question Explanation: ‘The right coronary artery supplies the inferior wall of the left ventricle. Ischemic changes (ST segment depression/elevation) will therefore manifest in the inferior ECG leads (leads TL, II, and avF), Ischemia in the left circumflex territory (B) will show changes in the lateral leads (L, avL, V5-\'6) The left anterior descending territory (C) is mapped by the anterior precordial leads (V2- V4iV'5), ‘Mark this question ec ‘Question Id: 82086 Question 12 of 30 A. 67-year-old woman shows ischemic changes on her EKG in leads I, II, and avF. The coronary vessel most likely to be affected is a) Left reain b) Let circumflex c) Left anterior descending SM © d) Right coronary Answer | Bxpianation | Other User's Explanation Report An Error Question Explanation: ‘The right coronary artery supplies the inferior wall of the left ventricle. Ischemic changes (ST segment depression/elevation) will therefore manifest in the inferior ECG leads (leads TL, II, and avF), Ischemia in the left circumflex territory (B) will show changes in the lateral leads (L, avL, V5-\'6) The left anterior descending territory (C) is mapped by the anterior precordial leads (V2- V4iV'5), Mark this question ez Question Td : 85789 Question 13 of 30 37 year old Japanese woman has a history of myalgias and fever for the past 3 weeks. She also reports increasing fatigue, weight loss and abdominal pain over the past 2 months. She has 2 20 pack year smoking history and admits to unprotected sex with multiple sexual partners in the past several months. Physical examination reveals a blood pressure of 15 mm Hg, higher in the left acm. Abdominal bruit is heard on auscultation, Racial pulse on her right arm is weak and it cannot be palpated on the left arm. The erythrocyte sedimentation rate (ESR) is 60 mmfhour. The most eppropriats diagnosis is a) Atherosclerosis b) Buerger disease ©) Giant cell arteritis 4) Takayasu arteritis, €) Syphilis 8) Thromnboohlebitis Question Explanation: TTaleayasu arteritis is an uncommon diserder that is most prevalent in young Asian women, In Takayasn arteritis, an inflammatory process produces fibrous thickening ofthe acrtic arch, causing narrowing or near obliteration of the origins of arteries that branch from the arch, Reduced blood pressure in one of beth arms is common a differential of mere than 10 malig between the armas is typically prosent. Abdominal pain, darthea, and gactrointestinal hemorchage may result from mesenteric agtery ischemia. The classic sign leading to the diagnosis is the abseace of pulses in the upper extremities (pulseless disease). Bruits are not common findings butif the disease is severe enough, they can be present due to aproncunced arterial narrowing, Ocular (visual loss, retinal hemorrhages) znd neurologic abnormalities can also be present While atherosclerosis can cercainly affect all ofthe vessels involved in this vignette, it does not adequately explain this patient's presentation, Atherosclerosis typically presents with chest pain on exertion. Buerger disease (taromboangitis obliterans) is a disease of young to middle-aged adult males with a history of heavy cigarette smoking. Itis cheractenzed by segmental thrombosis of arteries end veins, involving the extremities. Clinical presentation is significant for severe pain (claudication), digtal thrombophlebitis associated with microabscesses, ulceration, and gangrene. Giant cell arteritis, whose best known form is temporal arteritis, causes inflammation with giant cel formation that tends to affect medum-sized arteries of the head, such as the temporal and ophthalmic attesies, Tertiary syphils can cause aortic root diation, and sometimes secondarily involves the vessels originating in the aortic arch. ‘The dilation can compromise the aortic valve, causing insufficiency. Furthermore, the regurgitated blood increases cardiac contraction, which is represented by bounding pulses in the extremities. Tertiary syphilis is now very rarely diagnosed in the United States. probably because most people have had several courses of antibiotics of some sort during their lifetimes, and so people with undiagnosed syphilis are treated, Thrombophiebitis usually involves the deep veins of the legs and often occurs in association with eee cera CH ier eigen aes aCe Ce CC eS Mark this question ez Question Td : 85789 Question 13 of 30 A.37 year old Japanese woman has a history of myalgias and fever for the past 3 weeks. She also reports increasing fatigue, weight lose and abdominal pain over the pact 2 monthe. She has 2 20 pack year smoking history and admits te urpratected cex with multiple coxval partners ix the past ceveral months. Physical examination eveale a blood pressure of 15 mm He, higher in the leit acm. Abdominal bmuitis heard on auscultation. Racial pulse on her right arm is weale and it cannot be palpated on the left arma. The erythrocyte sedimentation rate (ESR) is 60 ma/kour. The most eppropriats dagnosis is a) Atherosclerosis ) Buerger disease ©) Giant cell artertie Y © d) Takayasu arteritis ©) Syphilis 8) Thromnboohlebitis Question Explanation: TTaleayasu arteritis is an uncommon diserder that is most prevalent in young Asian women, In Takayasn arteritis, an inflammatory process produces fibrous thickening ofthe acrtic arch, causing narrowing or near obliteration of the origins of arteries that branch from the arch, Reduced blood pressure in one of beth arms is common a differential of mere than 10 malig between the armas is typically prosent. Abdominal pain, darthea, and gactrointestinal hemorchage may result from mesenteric agtery ischemia. The classic sign leading to the diagnosis is the abseace of pulses in the upper extremities (pulseless disease). Bruits are not common findings butif the disease is severe enough, they can be present due to aproncunced arterial narrowing, Ocular (visual loss, retinal hemorrhages) znd neurologic abnormalities can also be present While atherosclerosis can cercainly affect all ofthe vessels involved in this vignette, it does not adequately explain this patient's presentation, Atherosclerosis typically presents with chest pain on exertion. Buerger disease (taromboangitis obliterans) is a disease of young to middle-aged adult males with a history of heavy cigarette smoking. Itis cheractenzed by segmental thrombosis of arteries end veins, involving the extremities. Clinical presentation is significant for severe pain (claudication), digtal thrombophlebitis associated with microabscesses, ulceration, and gangrene. Giant cell arteritis, whose best known form is temporal arteritis, causes inflammation with giant cel formation that tends to affect medum-sized arteries of the head, such as the temporal and ophthalmic attesies, Tertiary syphils can cause aortic root diation, and sometimes secondarily involves the vessels originating in the aortic arch. ‘The dilation can compromise the aortic valve, causing insufficiency. Furthermore, the regurgitated blood increases cardiac contraction, which is represented by bounding pulses in the extremities. Tertiary syphilis is now very rarely diagnosed in the United States. probably because most people have had several courses of antibiotics of some sort during their lifetimes, and so people with undiagnosed syphilis are treated, Thrombophiebitis usually involves the deep veins of the legs and often occurs in association with eee cera CH ier eigen aes aCe Ce CC eS ‘Mark this question = => (Question Id : 87859 Question 14 of 30 Aprevicusly healthy 21 year boy presents with recurrent episodes of syncope. Physical examination shows late systolic ejection murmur. ECG demonstrates atrial btilation. Echocardiography is performed and reveals left ventncular hypertrophy, impaired ventricular relazation, and an ejection fraction that is zbove normal The most likely diagnosis is a) Aortic stenosis b) Cardiac amyloidosis ©) Endocardial Sbroelestosis 4) Idiopathic hyopertrophic subaortic stenosis ¢) Loefiler endocarditis Question Explanation: The lesion describedis typertrophic cardiomyopathy, more specifically known is idiopathic hypertrophic subaortic stenosis (HSS). This lesion is usually seen in young adults, and a genetic predispostion (autosomal dominant) may be present Some patients with this condition may experience episodes of syncope, dyspnea, argina, dizeness, or congestive heart failure. Other patients are asymptomatic unti they undergo sudden death, usually during siremons exercise, possibly becanse the acrtic outlet becomes completely occluded is a result ofmmscle contraction. Left ventricular compliance is reduced (diastolic dyefunction) as a result of the hypertrophy, but systolic performance is not depressed. The heart hypercontractile and systole occurs with striking rapidity. Bjestion finctionis often increased and the left ventricle may be virtually obliterated in systole. On physical examination, systolic ejection, ‘marmur and till are characteristic. Infective endocarditis of the adjacent (damaged) mitral valve and a tril brillation may also occur. (On macroscopic examination, there is thickening of the interventricular septum at the level of the miral valve, and microscopically the myocytes are hiypertrophied and arranged in haphazard patter Aortic stenosis commonly presents in the elderly population. Tt presents with dyspnea, engina, syncope, and a systolis rourmur in the aortic area that radiates to the carotids Echocardiography shows thick aortic valve leaflets and left ventricular hypertroohy. Severe cardiac amyloidosis, endocardial fbroelastosis and Loeffler endocarditis can all produce a restrictive cardicmyopathy which is characterized by impaired relaxation with a normal ejection traction. There is no associated hypertrophic muscle, and atvial fibrillation is not a commen finding. ‘Mark this question = => (Question Id : 87859 Question 14 of 30 Aprevicusly healthy 21 year boy presents with recurrent episodes of syncope. Physical examination shows late systolic ejection murmur. ECG demonstrates atrial btilation. Echocardiography is performed and reveals left ventncular hypertrophy, impaired ventricular relazation, and an ejection fraction that is zbove normal The most likely diagnosis is a) Aortic stenosis b) Cardiac amyloidosis ©) Endocardial Sbroelestosis Y © 4) Idiopathic hyopertrophic subaortic stenosis ¢) Loefiler endocarditis Question Explanation: The lesion describedis typertrophic cardiomyopathy, more specifically known is idiopathic hypertrophic subaortic stenosis (HSS). This lesion is usually seen in young adults, and a genetic predispostion (autosomal dominant) may be present Some patients with this condition may experience episodes of syncope, dyspnea, argina, dizeness, or congestive heart failure. Other patients are asymptomatic unti they undergo sudden death, usually during siremons exercise, possibly becanse the acrtic outlet becomes completely occluded is a result ofmmscle contraction. Left ventricular compliance is reduced (diastolic dyefunction) as a result of the hypertrophy, but systolic performance is not depressed. The heart hypercontractile and systole occurs with striking rapidity. Bjestion finctionis often increased and the left ventricle may be virtually obliterated in systole. On physical examination, systolic ejection, ‘marmur and till are characteristic. Infective endocarditis of the adjacent (damaged) mitral valve and a tril brillation may also occur. (On macroscopic examination, there is thickening of the interventricular septum at the level of the miral valve, and microscopically the myocytes are hiypertrophied and arranged in haphazard patter Aortic stenosis commonly presents in the elderly population. Tt presents with dyspnea, engina, syncope, and a systolis rourmur in the aortic area that radiates to the carotids Echocardiography shows thick aortic valve leaflets and left ventricular hypertroohy. Severe cardiac amyloidosis, endocardial fbroelastosis and Loeffler endocarditis can all produce a restrictive cardicmyopathy which is characterized by impaired relaxation with a normal ejection traction. There is no associated hypertrophic muscle, and atvial fibrillation is not a commen finding. ‘Mark this question & => Question Id : 89739 Question 15 of 30 INCORRECT statement regarding management of congestive heart failare is: a) Dyspnea can be helped by decreasing let ventricular filing pressure ) Fatigue can be relieved by afterload reduction, €) Vasodilators can relieve symptoms by reducing preload, but they also increase afterload. 4) Hycralicine exerts its effects by afferload reduction ) Nitrates exert their effect by preload reduction. Answer | Explanation Other User's Explanation Report An Error Question Explanation: ‘Vasodilators reduce preload by decreasing left ventricular filing pressure, The nitrates are a class of drugs that exert their beneficial effects by decreasing left ventricular filing pressure. Dyspnea results from elevated left ventricular filling pressure, Other vasodilators, such as hydralisine, work by decreasing afterload. This improves cardizc output, Fatgue in these patients is a resut of decreased cardiac output.

You might also like